Gamle spørsmål Internal

Download as pdf or txt
Download as pdf or txt
You are on page 1of 219

INTERNAL EXAM

Na+ 135 - 140 mEq/L

K+ 3.5 - 5.0 mEq/L

Ca2+ 8.5 - 10.5 mg/dL


4.3 - 5.3 mEq/L

Mg 1.5 - 2.5 mg/dL

HCO3 21 - 27 mEq/L

Cl- 96 - 106 mEq/L

pCo2 35-45 mmHg


7.7 - 6.0 kPa

H+ 40 nEq/L
GI TRACT
On physical examination of a young patient after motorcycle accident in a serious state you
have noticed bruising of the epigastric region. On percussion the liver dullness is not
present, and you are not able to perform deep palpation doe tu the muscle guarding and
severe pain. indicate the most probable diagnosis
a)perforation of the GI tract
b)cholecystitis
c)aortic aneurysm rupture
d)hepatitis
e)right sided pneumothorax
a

Indicate the true statement about bowel sounds


a)they are described as click and gurgles
b)they occur in regularity 5-33 / minute
c) decreased bowel sounds occur with peritonitis and paralytic ileus.
d) a, b and c are correct
e) only a and b are correct
- D

Tension of the abdominal wall muscles to protect inflamed organs within…. from
pressure upon them (detected during palpation)is known as
a)abdominal guarding
b)rebound tenderness sign
c)blomberg sign
d)goldflam sign
e)none of the above
a

GI bleeding might be manifested by (choose incorrect)


a)hemoptysis
b)melena
c)chock
d)skin pallor if chronic or severe
e)hematemesis
a → hemoptysis is coughing up blood. Would rather vomit blood (hematemesis).

Peptic ulcer disease


Indicate the FALSE answer about peptic ulcer disease:
a. The most common causes are infection with H. pylori and nonsteroidal anti- inflammatory
drugs
b. Pain usually locates in epigastrial region, may be dull, sharp or burning in character
c. Pain is influenced by ingestion of meals
d. It is the least prevalent cause of gastrointestinal bleeding
e. Peptic ulcer disease may mimic for example pancreatitis, cholesystitis, gastroesophageal
reflux
- D

In april a 37 year old man presented to your office for an increase in frequency and severity
of burning epigastric pain. He has experienced the pain occasionally for more than 2 years.
For 6 weeks pain occurs three or four times per week, usually when he has an empty
stomach, and it often awakens him at night. The pain usually is relieved within minutes by
food or over the counter antacids but then recurs within 2-3 hours. There is no marked
weight or appetite change. What is the most probable cause?
a)gastric cancer
b)peptic ulcer disease
c)colon cancer
d)hiatal hernia
e)ulcerative colitis
B

Choose the false statement for peptic ulcer disease


a)risk factors are colonication with H pylori, smoking, NSAIDS, stress
b)all stomach ulcers are benign
c)endoscopic biopsy confirms colonization with H pylori and histological nature of the ulcer
d)in zollinger ellison syndrome ulcers are more difficult to cure with standard
pharmacotherapy and tend to be recurrent
e)among common complications are anemia and bleeding from the GI tractco
B

Where are the peptic ulcers located when it occurs on an empty stomach and can be
relieved by food and antacids?
- Duodenum.

Where are the peptic ulcers located when it occurs after eating and can be relieved by
vomiting?
- Stomach.
Gastric or duodenal ulceration might be caused by
a)chronic treatment with nonsteroidal anti inflammatory drugs
b)chronic treatment with nonsteroidal anti inflammatory drugs in combination with
corticosteroids
c)helicobacter pylori infection
d)a and c are correct
e) a b and c are correct
e

Abdominal Signs

Blumberg’s sign is most closely associated with:


a. Peritonitis
b. Paralytic ileus
c. Appendicitis
d. Pancreatitis
e. Only a and c
- E
Which of the following signs is NOT typical for appendicitis?
a. Jaworskis sign
b. Rowsings sign
c. Blumbergs sign
d. Murphys sign
e. Aaron sign
- D

Positive Chelmonski’s sign occurs in:


a. Pancreatitis
b. Appendicitis
c. Cholecystitis
d. Ulcer perforation
e. Pneumonia
- C

Cullens sign is:


a. Bluish discoloration around the umbilicus
b. Reddish discoloration along the flanks
c. Pain during inspiration
d. Characteristic for pyloric stenosis
e. Sign of liver cirrhosis
- A (pancreatitis)

Ganske blå i huden han derre Edward Cullen da

Match the following signs with their causes


1.rovsing
2.psoas
3.murphy
4.kernig
5.cullens

a)1-appendices, 2-appendices, 3-renal …, 4-meningitis, 5-acute hepatitis


b)1-appendicitis, 2-appendicitis, 3-acute pancreatitis, 4-cholecystitis, 5-acute pancreatitis
c)1-peritonitis, 2-cholecystitis, 3-meningitis, 4-acute hepatitis, 5-appendicitis
d)1-peritonitis, 2-peritonitis, 3-renal…4-acute hepatitis, 5-acute pancreatitis
e)1-appendices, 2-appendices, 3-gallstones, 4-meningitis, 5-acute pancreatitis
e
- 1: Rovsing = peritonitis + appendicitis
- 2: psoas = appendicitis
- 3: Murphy = cholecystitis
- 4: kernig = meningitis
- 5: cullens = pancreatitis

Skin symptoms of acute pancreatitis are


a)cullen sign
b)grey turner sign
c)loeffler sign
d)a and b
e) a b and c
d
- a: Cullen = ecchymosis around umbilicus
- b: Grey-Turner = ecchymosis around flanks
- c: Loeffler sign = eosinophils accumulate in the lung in response to a parasitic
infection.

The best description of Courvoisier sign is:


a) Enlarged not tender gallbladder, usually due to carcinoma of pancreas or lower biliary tree
b) Enlarged tender gallbladder, usually due to gallstones
c) Pain on palpation of the RUQ when the patient inhales might indicate acute cholecystitis
d) Abdominal pain radiating to the left shoulder due to spleen rupture
e) None of mentioned above
- A

Courvoisier sign
a)may indicate cancer of the pancreatic head
b)is characterized by a painful palpable gallbladder
c)occurs in acute cholecystitis
d)correct a b and c
e)none of the answers are correct
a

Courvoisier sign, indicate FALSE answer:


a. Gallbladder is large and tender
b. Patient has jaundice
c. Most typical for ampullary carcinoma
d. Gallbladder is not tender
e. All statements are false
- A
RLQ pain intensified by LLQ abdominal palpation is characteristic for:
a) Jaworski sign
b) Rowsing sign
c) McBurney sign
d) Murphy sign
e) Chelmonski sign
- B

Where is McBurnery’s point located?


- ⅓ distance between umbilicus and right anterior superior iliac spine.

Appendicitis
First common location of abdominal pain in acute appendicitis would be
a)right shoulder
b)right iliac fossa ⅓ distance between right anterior superior iliac spine and umbilicus
c)left iliac fossa ⅓ distance between left anterior superior iliac spine and umbilicus
d)right epigastric area
e)left scapula
B

Ulcerative Colitis and Crohns Disease


- what differentiates ulcerative colitis and crohns?
- ulcerative colitis vs crohn's disease.
A cardinal symptom of ulcerative colitis is:
a) constipation
b) diarrhea with blood and mucus in the stool
c) ribbon-shaped stools
d) periumbilical pain
e) none of the above
- B
- c: Ribbon-shaped is the same as pencil-shaped. Often seen in colon cancer.

a young patient comes to the ER with fever, cramping abdominal pain, painful bloody
diarrhea (8 times a day), weight loss lasting for more than 3 weeks. indicate the
possible differential diagnosis
1)toxic megacolon
2)crohn disease
3)mechanical bowel obstruction
4)appendicitis
5)ulcerative colitis
a)1, 2, 3
b)2, 3, 4
c)1, 3, 5
d)2, 3, 5
e)2, 5
E
- 1: No, toxic megacolon would be more of an acute situation, not last for more than 3
weeks.
- 3: No, mechanical bowel obstruction would not cause diarrhea, rather constipation,
bloating and vomiting.
- 4: No, appendicitis rarely causes bloody diarrhea and would be more acute.

Colon Cancer

A 68 year old caucasian female presented to her primary care provider with complaints of
weakness and fatigue over the past weeks. She experienced recent weight loss of 10kg, not
attributable to diet or exercise. When questioned about her stool, she reported chronic
constipation, bright red blood in her stool and a smaller diameter of stool over the past 2
weeks. What is the most probable cause of given signs and symptoms
a)appendicitis
b)peptic ulcer disease
c)colon cancer
d)diverticulitis
e)ulcerative colitis
c
In 80 years old male patient with a history of 10 kg weight loss, poor appetite,
abdominal pain, intermittent diarrhea and constipation. In blood test, severe
microcytic anemia, thrombocytosis, …. white blood cell count. The most probable
diagnosis is
a)colorectal cancer
b)addison biermer disease
c)acute myeloid leukemia
d)bleeding peptic ulcer
e)all above are mentioned answers are correct
a
- Colorectal cancer can cause microcytic anemia, due to loss of iron in the bloody
stools.

The most common malignant neoplasm of the GI tract is:


a. Esophageal cancer
b. Colorectal cancer
c. Pancreatic cancer
d. Gastric cancer
e. None of them are common
- B

Pancreas
Pancreatitis
Steady epigastric pain, which may be partly relieved by sitting up and leaning forward
radiating to the back is characteristic for:
a) PUD (Peptic Ulcer Disease)
b) Biliary pain
c) Pancreatic pain
d) Renal colic
e) Bowel obstruction
- C

In acute pancreatitis you will observe all of the following symptoms except
a)incessant nausea and vomiting
b)diminished or lack of peristalsis
c)diminished muscle guarding
d)grey-turner sign
e)pleural effusion - more common the the left side
- C
Acute pancreatitis may be caused by:
a) gallstones
b) alcohol abuse
c) superior vena cava syndrome
d) A and B true
e) A,B,C true
- D

Acute Pancreatitis:
a. Main causes are alcohol abuse and biliary tract disease
b. May present with Grey-Turner’s sign and/or Cullen’s sign.
c. The Ranson score can be used as an indicator of severity of disease.
d. Diagnosis may include X-ray as well as increased amylase levels.
e. All are true
- E

The most common cause of chronic pancreatitis is:


a) connective tissue disorders
b) metabolic disorders, stress,
c) trauma
d) alcohol abuse
e) drugs, viral infections
- D

Show the most characteristic features of acute pancreatitis:


a) dull, strong pain radiating to the right hand side, increasing while fasting
b) pain “colic” in nature in epigastric region, with increased bilirubin level
c) severe, constant epigastric pain that radiates through to the back, along with an elevated
blood amylase level
d) pain in the right upper quadrant, radiating to the back, increasing after fatty meal,
with
increased transaminase levels
e) migrating pain, in the right lower abdominal quadrant, with anorexia, and fever
- C
Indicate the FALSE answer about pancreatitis:
a. The alcohol abuse does not have any influence of incidence of pancreatitis
b. Acute pancreatitis usually presents with severe pain, radiating to the back, usually worse
when supine, nausea, vomiting, and fever
c. The cause of acute pancreatitis may be: gallstones, hypertriglyceridemia, trauma, alcohol
ingestion
d. The consequence of chronic pancreatitis may be: malabsorption syndromes, diabetes
mellitus, weight loss, pseudocysts formation
e. Chronic pancreatitis may present with: pain episodic or continuous, diarrhea and weight
loss
- A

What are some complications of chronic pancreatitis?


- malabsorption syndromes
- diabetes mellitus
- weight loss
- pseudocysts formation
- splenic venous thrombosis
- pancreatic cancer
- duodenal or biliary obstruction

39. disappearance of liver dullness due to pancreatitis? perforation of gi tract? cholecystitis?


intestinal obstruction? ascites?

4)which of the below can not be a cause of transudat


a) hypoalbuminema
b) congestive heart failure
c) pancreatitis
d) chirrosis
- C → exudate
Pancreatic Cance r
Painless jaundice, without gallstones substantial weight loss, increased ESR followed
by gradually increasing pain in epigastrium, radiating to the back, what disease
manifests like this
a)pancreatic cancer
b)chronic pancreatitis
c)hepatic dysfunction in liver cirrhosis
d)chronic cholecystitis
e)portal hypertension
a

A 63 year old man presents with significant weight loss and circumflexing back pain. The
back pain is constant and keeps him awake at night. Over the past 2 months he has
developed hyperglycemia and nocturia. On examination he appears cachectic and pale,
without jaundice. Laboratory studies reveal a mild normochromic anemia. Liver and kidney
function studies are normal. What do you think may be a preliminary diagnosis
a)acute pancreatitis
b)spondyloarthrosis of the lumbar part of the vertebral column
c)pancreatic neoplasm
d)diabetes mellitus type 1
e) none of the diagnosis
c
(I) jaundice never occurs in the course of pancreatic cancer, because (II) this cancer
often causes the compression of bile duct
a)both parts are true, correlated
b)both parts are true, no correlation
c)I is true, II false
d) I is false, II true
e) both are false
d

Diabetes

A 65 year old male randomly used his wife’s glucometer and obtained a blood glucose
level of 245 mg/dl. Next day he visits his family doctors, has no symptoms
/complaints. Can you diagnose the patient with diabetes type 2
a)yes, a single random blood glucose measurement >200 mg/dl is sufficient to make this
diagnosis
b)yes, but only if on the following day the patient’s fasting blood glucose level is 126 mg/dl
c)yes but only if on the following day the patient’s fasting blood glucose is 100mg/dl
d)answers b and c are correct
e)none of the above
E

A patient with fasting plasma glucose level of 111 mg/dl and 114 mg/dL was performed
75 g oral glucose tolerance test. Glucose level after 2 hours was 225 mg/dl. Diagnosis
based on these result ´s in
a) normoglycemia
b)impaired fasting glucose
c)impaired glucose tolerance
d)diabetes mellitus
e)b and d are true
D
The diabetes can be diagnosed if:
a) random glucose sampling exceeds 200 mg/dl, twice with accompanying symptoms
b) fasting glucose level exceeds 126 mg/dl, at least twice
c) 2-hour plasma glucose level in glucose tolerance test exceeds 140 mg/dl
d) 2-hour plasma glucose level in glucose tolerance test lies between 140 -200 mg/dl
e) answers A and B are correct
- E

Diabetes is diagnosed om the basis of


a)plasma glucose from venous blood equal to >200 mg/dl
b)fasting glucose > 100 mg/dl
c)glucose above 140 mg/dl in the second hour of the OGTT
d)correct a b and c
e)none of the above
a

Which of the following is a CORRECT criterion for diabetes mellitus?


a. Random glycemia <140 mg / dl ,with symptoms of hyperglycemia
b. One different fasting plasma glucose levels of ≥126 mg / dl
c. Two different fasting plasma glucose levels of ≥126 mg / dl
d. Glycemia in 2nd hour of oral glucose tolerance test ≥ 126 mg /dl
e. Glycemia in 1st hour of oral glucose tolerance test ≤ 100 mg /dl
- C

In oral glucose tolerance test the first glucose measurement (before oral glucose
administration) was 99 mg% and 2 hours after glucose intake - glucemia was 225 mg.
That indicates
a)impaired glucose tolerance
b)impaired fasting glucose
c)diabetes mellitus
d)diabetes insipidus
e)reactive hypoglycemia
c
Which sign is not related to diabetes mellitus?
a. Polydipsia (increased thirst)
b. Polyphagia
c. Polyuria
d. Polyopia
e. Hyperglycemia
- D

Classic signs of hyperglycemia are


a)oliguria, decreased thirst, dehydration, weight loss
b=oliguria, increased thirst, overhydration, weight gain
c)polyuria, decreased thirst, overhydration, weight loss
d)polyuria, increased thirst, dehydration, weight loss
e)none of the above
d

Show the characteristic signs and symptoms of diabetes:


a) obesity, high hemoglobin levels, nycturia
b) high cholesterol and triglycerides levels, obesity, family history
c) polidypsia, poliuria, polyphagia, decrease in body weight, sleepiness
d) metabolic syndrome features
e) none of the above
- C

Weight loss, increased thirst, poliuria, fatigue, sight disturbances are the indicative
symptoms of :
a. hyperthyroidism
b. pancreatitis
c. diabetes mellitus
d. coronary artery disease
e. cholecystitis
- C

Which of the following i an acute complication of diabetes


a)iatrogenic hypoglycemia
b)diabetic nephropathy
c)diabetic foot syndrome
d)diabetic retinopathy
e)peripheral polyneuropathy
A

Indicate the chronic complications of diabetes


a)hypoglycemia
b)autonomic neuropathy
c)diabetic ketoacidosis
d) all above answers are correct
e)correct answers are a and c
B
Type 1 diabetes is caused by
a)chronic pancreatitis
b)insulin resistance
c) autoimmune reaction
d)destruction of alpha cells in pancreas
e) none of the above
c

Which condition(s) is/are most specific for ketoacidosis?


a. Diabetes Type 1
b. Diabetes Type 2
c. Diabetes Type 3
d. Diabetes Type 4
e. All of the above3
- A

A 23 year old woman is admitted to the emergency department. On examination a breathing


pattern with increased breath amplitude and rate is evident, rotten fruit smell is present, skin
turgor is diminished, heart rate is 140 bpm, BP 100/56 mmHg. Which chronic disease may
present in a form of an acute complication with the mentioned symptoms
a)
b)hemochromatosis
c)hyperparathyroidism
d)type 1 diabetes mellitus
e)
D

Type 2 diabetes is characterized by


a)insulin resistance
b)insulin deficiency due to autoimmune destruction of beta cells
c)frequent association with hypertension
d)all of above
e) a and c are true
e

Indicate true sentence


a)type 2 diabetes mellitus most often has an acute onset
b)diabetes mellitus type 2 - the most common complication is ketoacidosis
c)diabetes mellitus might be a complication of chronic pancreatitis
d)diabetes mellitus type 2 in all cases must be treated with insulin
e)hypoglycemia is the benign complication of diabetes mellitus treatment and has no impact
on patient overall risk
c

(I) there is insulin resistance in diabetes mellitus type 2, therefore (II) insulin is not
used in tis treatment
a) I and II are true, correlation
b) I and II are true, no correlation
c) I is true, II false
d) I is false, II true
e) I and II false
- C

A 76 year old female with diabetes mellitus type 2 and nephrolithiasis is admitted to hospital
due to severe fever, dehydration and pyuria. BP 80/50 mmHg, HR 120/min, bruises and
petechiae are visible on the skin. White blood cell count is severely increased. Haemoglobin
10 &, platelets 40 000/mm3, CRP 300, procalcitonin 20 x above upper norm limit, INR 1.5
APTT 80, fibrinogen not detectable. The clinical picture describes
a)cardiogenic shock with reactive anergy of a bone marrow
b)lower urinary tract infection
c)septic shock with disseminated intravascular coagulation
d)severe urinary tract infection with reactive anemia and thrombocytosis
e)anaphylactic shock due to the bacteria toxins
- C - tror vi - H&I

A 65 year old woman comes to your clinic with a history of coughing up 1-2 tablespoons of
bright blood on 2 occasions. She has smoked heavily for the past 30 year and reports recent
weight loss of 10-15 kg. Her past medical history includes diabetes, hypertension and
rheumatoid arthritis. Her medication include insulin, atenelol and ibuprofen. What are the
alarming factors in this patient?
a)quantity of hemoptysis
b)advanced age, weight loss and history of smoking
c)history of diabetes, hypertension and rheumatoid arthritis
d) current medication
e)none of the above symptoms is an alarming
B
The cause of hyperglycemia as a result of increased insulin demand (insulin
resistance) may be
a)obesity
b)acute and chronic infections
c)hyperthyroidism
d)correct a and b
e) correct a b and c
e

Possible signs of autonomic neuropathy are


a)orthostatic hypotension
b)bradycardia
c)excessive sweating
d) a and b are true
e) a and c are true
e

6. The most common metabolic complication of insulin therapy is:


a) Hypoglycemia
b) Hyperglycemia
c) Ketoacidosis
d) Diabetic coma
e) Metabolic alkalosis
- A

Cause of hypoglycemia might be


1.prediabetic condition
2.acromegaly
3.liver disease
4.pheochromocytoma
5.adrenal insufficiency
6.insulinoma
7.excessive alcohol consumption
a)1, 3, 5, 6, 7
b)1, 2, 3, 6
c)4, 3, 5, 7
d)1, 2, 3, 7
e)only 6
a

30) The most common metabolic complication of insulin therapy is:


a.Hypoglycemia
b. Hyperglycemia
c. Ketoacidosis
d. Diabetic coma
e. Metabolic alkalosis
- A
The cause of insulin resistance may be
a)obesity
b)acute and chronic infections
c)hypothyroidism
d) a and b
e)all of the above
d

Liver
The most sensitive laboratory parameter for liver damage is
a)presence of urobilinogen in urine
b)prothrombin time
c)platelets level
d)albumin concentration
e)bilirubin concentration
c?
- usikker, kanskje det er b?

No dullness of liver indicates:


a. Enlarged liver
b. Perforation of GI tract
c. Anemia
d. Liver cirrhosis
e. Right heart failure
- B

No dullness of liver can be due to:


a) Perforation of GI tract
b) Hepatitis
c) Cholecystitis
d) Enterocolitis
e) Ascites
- A

Dullness over liver area


a)is pathologic
b)is present only in very obese patients
c)is a sign of liver disease
d)is a sign of right ventricle heart failure
e)none of above
E → dullness is physiological.
Liver Disease

What are the important risk factors of liver disease?


a) alcohol abuse
b) chronic viral hepatitis B and C
c) a1-antitrypsin (a1AT) deficiency
d) drug abuse
e) all of the above
- E

Show the characteristic signs and symptoms for liver disease:


a) vomiting, weight loss, appetite for meat
b) pain on fasting, weakness, jaundice
c) jaundice, dark urine, light stools, itching, abdominal pain
d) oedema, ascites, tachycardia, fatigue
e) none of the above
- C

Liver disease which not happen:


a) caput medusa
b) gynecomastia
c) ascities
d) clubbed fingers happens
e) cyanosis
- E

Acute Hepatitis
Severe pain in right upper abdominal quadrant is most typical for all EXCEPT:
a) gallbladder disease
b) acute hepatitis
c) liver abscess
d) severe venoocclusive disease
e) acute cholecystitis
- B

Liver cirrhosis
Which is not a laboratory finding of liver failure
a)increased INR
b)decreased albumin level
c)increased ALT and AST level
d)increased bilirubin level
e)increased white blood cells level
e

Laboratory finding of liver failure:


● Increased ALT and AST
● Low albumin
● High bilirubin
● Increased INR → takes longer time for blood to clot.

Which of the following is not a complication of liver cirrhosis:


(a) Esophageal varices
(b) Hepatic coma
(c) Hyperglycemia
(d) Hemorrhagic diatheses
(e) Ascites
- C

Which of following are symptoms of liver cirrhosis?


a) Edema
b) Spider angioma
c) Caput medusae
d) All are correct
e) None are correct
- D

Which of below mentioned symptoms might be present in liver cirrhosis


a)jaundice, palmar erythema, hyperalbuminaemia
b)esophageal varices, splenomegaly, thrombocytosis
c)jaundice, thrombocytosis, spider angioma
d)ascites, hypoalbuminemia, thrombocytopenia
e)cyanosis, anemia, hyperalbuminaemia
D

Choose the correct statement referring to liver cirrhosis


a)it results in a decrease in hepatocellular mass, and thus function, but does not alter blood
flow
b)patients with cirrhosis early manifest with liver dysfunction
c)portal hypertension is a significant complicating feature of cirrhosis
d)the complications of cirrhosis closely depend on the etiology of liver disease
e)cardiac failure is the most prevalent cause of liver cirrhosis
c

Which statement concerning acute viral hepatitis A is FALSE:


a) usually fecal-oral transmission
b) incubation period is 2-6 weeks
c) usually leads to chronic hepatitis and cirrhosis
d) may start with prodromal symptoms including fatigue, nausea and myalgia
e) splenomegaly may be present in some cases
- C

Causes of hepatic cirrhosis might be:


1. alcohol abuse
2. viral hepatitis
3. hepatoma (=HCC)
4. autoimmune chronic active hepatitis
5. duodenal ulcer

a)1,2,4 b)3,4,5 c) 5 d)1,3 e)1,2,3,4


- E

In patients with liver cirrhosis you will observe symptoms such as


a)jaundice
b)telangiectasias
c)leukonychia
d)xanthelasma
e)all answers are correct
e

Choose the correct symptoms of liver cirrhosis


1.cyanosis
2.jaundice
3.enlarged veins on the … side of the abdomen
4.varicose veins on the lower limbs
5.palmar and foot erythema
6.nail pitting
7.leuco…?
8.loss of axillary and pubic hair in man
9. alopecia areata (spot baldness)

a)1, 2, 3, 6, 7
b)1, 3, 5, 7, 8
c)1, 3, 5, 6, 9
d)2, 3, 4, 5, 6
e)2, 3, 5, 7, 8
e
- 4: esophageal varicose veins ***
- 6: nail pitting occurs in psoriasis.

(I)thrombocytopenia is present in liver cirrhosis (II) because due to portal


hypertension and ….. platelets are being destroyed
a)I and II are true, connected
b)I and II are true, no connection
c)true, false
d)false, true
e)false, false
C (previous answer was E)

A 58 year old male with a long standing cirrhosis resulting from hepatitis C develops vague
right upper quadrant pain and weight loss. A right upper quadrant mass is palpable. Serum
alkaline phosphatase is elevated. What would you choose as first line diagnostic tool
a)gastroscopy
b)liver biopsy
c)urography
d)abdominal CT with contrast
e)colonoscopy
d?

Gallbladder

Cholecystitis / Gallbladder inflammation


Clinical features of acute cholecystitis are:
a) acute onset of the pain
b) the pain localizes in the epigastrium and/or right hypochondrium
c) nausea and vomiting are common
d) A and B are true
e) A,B and C are true
- E

Rebound tenderness over lower liver margin may be a symptom of:


a. Colitis ulcerosa
b. Cholecystitis
c. Peptic ulcer disease
d. Urinary tract infection
e. Pneumonia
- B

Cholelithiasis / Gallstones
What are the risk factors for cholelithiasis?
a. Female
b. Fat
c. Fertile
d. Oral contraceptives
e. All answers are correct
- E

Which features increases the risk of gallstones formation


a)male sex, early twenties, smoking
b)vegetarian diet, smoking, severe periods
c)hemolytic anemia, splenomegaly, history of pancreatitis
d)female sex, middle age, obesity
e)liver cirrhosis, varicose veins, alcohol consumption
D

Increased pain at the peak of inspiration during palpation of the right costal margin is
characteristic for
a)gallbladder cancer
b)liver cancer
c)cholelithiasis
d)chronic pancreatitis
e)acute cholelithiasis
E?
- Hadde gått for E, men riktig er vell cholecystitis?

Which of the following DON’T cause Jaundice?


a. Hemolytic anemia
b. Ampullary carcinoma
c. Cholelithiasis
d. Choledocholithiasis
e. Liver cirrhosis
- C?
Biliary Colic
Indicate FALSE answer for biliary colic:
a. Epigastrium is the most common site of pain
b. Pain is colicky
c. Relieved by NSAIDs ( nonsteroidal anti inflammatory drugs)
d. Radiate to right scapula and right arm
e. Pain often follows a few hours after meals
- B

THYROID GLAND
The cause of thyroid enlargement (goiter) might be:
a) Hypothyroidism
b) Thyroiditis
c) Thyroid gland tumors
d) Iod deficiency
e) All of the above
- E

“Goitre” is a term used for:


1. neck asymmetry
2. thyroid enlargement
3. salivary gland enlargement
4. hyperthyreoidismus
5. hyperparathyreoidismus
- 2

Which of the conditions can be the cause of goiter


a)grave’s disease
b)iodine deficiency
c)thyroid cancer
d)thyroiditis
e)all above
E

Choose the cause of hypertension


a)grave’s disease
b)de Quervain thyroiditis
c)radiation thyroiditis
d) can’t see
e)all of the above
e

Hypothyroidism
Causes of decreased body weight does not include
a)hypothyroidism
b)hyperthyroidism
c)neoplasm
d)eating disorder
e)malabsorption
- A

Which of the following is NOT a sign of hypothyroidism?


a) anosmia
b) lethargy
c) moist skin (correct)
d) irritability
e) a & d
- C → dry skin

Fever may be a coexisting sign in listed diseases except one


a)infectiour
b)malignancies
c)tissue injury
d)hypothyroidism
e)connective tissue disorders (ex lupus)
D-

In which condition the rate of breathing will not increase?


a)acidosis
b)anxiety
c)pan
d)aspirin poisoning
e)hypothyroidism
E → hypothyroidism causes decreased breathing rate

Indicate the proper name for the face features given below; dry, thickened skin, dry,
brittle hair and startled face expression without mimic
a)cushing syndrome
b)hypothyroidism
c)hippocratic face
d)grave’s disease
e)down syndrome
B

Cold intolerance, weight gain, bradycardia, myxedema all are features of :


a. Hyperthyroidism
b. Hypothyroidism
c. Cushing disease
d. Cushing syndrome
e. Addisons disease
- B

13.Which one does not cause facies rubra


a)Mitral face
b)alcoholism
c)erythema
d)hypothyroidism
e)Hypertension
- D
Hashimoto Disease
Which antibodies are usually NOT found in Hashimoto’s disease?
a. Tyrosine phosphatase autoantibodies
b. Antithyroid peroxidase
c. Antithyroglobulin
d. TSH receptor blocking antibodies
e. Thyroid stimulating antibody
- A
- Tyrosine phosphatase autoantibodies er i diabetes type 1

Hyperthyroidism
Exophthalmos is common in
a)Hyperthyroidism
b)hypothyroidism
c)Grave´s disease
d)Answers a & c
e)Only c
- D

Weight loss is most characteristic for one of below mentioned diseases:


a) Anaemia
b) Diabetes mellitus type 2
c) Hyperthyroidism
d) Cirrhosis
e) Hypothyroidism
- C

1. A young (30 years old) female patient comes to the outpatient clinic complaining of heart
palpitations, weight loss, vision disturbances, and leg swelling, she had also noticed that her
favorite necklace became too tight for her. What might be the preliminary diagnosis?
a) acromegaly
b) mital valve disease
c) chronic heart failure
d) hyperthyroidism
e) ulcerative colitis
- D

A lady arrives at the hospital complaining of sudden weight loss. She is sweating
excessively and is irritable. Her eyes also appear to be bulging. An expected lab
finding is:
a. High ACTH
b. High Cortisol
c. High Thyroxine
d. High Growth Factor
e. High Prolactin
- C
Indicate the FALSE answer. Decrease skin moisture:
a) Elders
b) Scleroderma
c) Hyperthyroidism
d) Vomiting
e) Diarrhea
- C

The vascular murmur heard over thyroid gland may be a feature of:
a. Hyperthyroidism
b. Hypothyroidism
c. Internal carotid artery stenosis
d. a) and c)
e. All of the above
- A

Hyperthyroidism – indicate the FALSE statement:


a. the reasons might be, among others, Graves-Basedow disease and multinodular goiter
b. is characterized by tachycardia, increased SBP, anxiousness, weight loss, palpitation,
sweating
c. constipation, weight gain, dry skin, bradycardia, sleepiness
d. there might be some ocular sings like Kocher, Moebius sings
e. there might be murmur over thyroid gland
- C

Constant feeling of heat, excessive sweating, loss of weight despite good appetite,
hand tremor, palpitations are clinical symptoms
a)hypothyroidism
b)diabetes insipidus
c)hypoparathyroidism
d)acromegaly
e)overactive thyroid gland
E

Patient comes to the ER presenting tachycardia, increased sweating and problems


with concentration. Upon physical examination you notice the presence of sclera
above the iris while patient looks down. What do you suspect?
a)Hyperthyroidism in graves disease
b)hashimoto disease
c)hypothyroidism in graves disease
d)iron deficiency anemia
e)viral hepatitis
A

The most common causes of hyperthyroidism are:


a) Graves diseases
b) Toxic multinodular goitre
c) Solitary toxic nodule/adenoma
d) A and B
e) all of the above
- E

Hyperthyroidism may occur due to


a)hashimoto disease
b)thyroiditis
c)thyroid cancer
d)all mentioned are true
e)all mentioned are false
- D

Indicate the symptoms that is NOT characteristic for thyrotoxicosis


a)nervousness/anxiety
b)increased sweating
c)heart palpitation
d)weight gain
e)diarrhea
d

Graves Disease
Bruit over the thyroid is a typical finding in patient with
a)multinodular goiter
b)grave’s disease
c)hashimoto thyroiditis
d)simple goiter
e)thyroid cancer
B
Indicate the characteristic features of Graves-Basedow disease:
a) tachycardia, skin thickening over tibia, weight loss, diffuse goiter
b) proximal muscle weakness, increase in body weight, ophtalmopatia, hand tremor
c) palpitation, oedema, increase in body weight, constipation
d) goiter, sleepiness, dry skin, coarse voice
e) none of the above
- A

9. Graves disease: multinodular goiter and weight loss

PARATHYROID GLAND
Hyperparathyroidism
A 72 year old female with history of bilateral renal stones since 2010 recurrent gastric ulcers
since 2020 and osteoporosis. Chronic supplementation of calcium (calperos 500 mg/ p.o)
and vitamin D3 (vigantoletten 1000 UI p.i). Her rheumatologist ordered a blood test which
revealed calcium 13,6 mg/dl (norm 8.9-10.0), phosphate 1.6 mg/dl (2.3-4.7) creatinine 1.4
mg/dl (0.73-1.18). Based on the above history and results, the most likely diagnosis is
a)hypercalcemia due to a vitamin D3 overdose
b)hypoparathyroidism secondary to neoplastic disease
c)primary hyperparathyroidism
d)hyperparathyroidism secondary to renal failure
e)primary hypoparathyroidism
- C → primary because it’s high calcium and low phosphate.

What is not a symptom of hypercalcemia


a)polyuria and polydipsia
b)urolithiasis
c)peptic ulcer disease
d)hypotension
e)pancreatitis
d?
Patient with long term hypertension, joint pain and history of… stones for the second time.
What do you suspect?
a)hypothyroidism
b)chronic kidney disease
c)cushing’s disease
d)primary hyperparathyroidism
e)primary hyperaldosteronism
D

Hypoparathyroidism

PITUITARY GLAND
Each of the following is a hormone contained in the anterior pituitary gland except:
a) Follicle-stimulating hormone (FSH)
b) Prolactin
c) Thyroid-stimulating hormone (TSH)
d) Oxytocin
- D
Each of the following is a cause of hyperprolactinemia except:
a) Pituitary adenoma
b) Chest wall trauma
c) Pregnancy
d) Hyperthyroidism
- D → hypothyroidism

Pituitary Tumor

Which of the pathologies below may result from a pituitary adenoma?


a) Bitemporal hemianopsia
b) Acromegaly
c) Hyperthyroidism
d) Galactorrhea
e) All of the above
- E
- A→ same as tunnel vision

Each of the following clinical presentations is consistent with a pituitary adenoma


except:
a) Headaches
b) Galactorrhea
c) Central visual defect
d) Amenorrhea
e) all of the above
- E

Acromegaly

Acromegaly:
a. Is most often due to a benign GH-secreting pituitary adenoma
b. A major sign is atrophy of the tongue
c. IGF-1 level is not important in diagnosis
d. Is the name given to excess growth hormone in children
e. None are true
A
b: no, they can have macroglossia.
c: no, IGF-1/GH over-secretion is the cause.
d: No, acromegaly typically occurs in 40-year old people.

Which symptom is absent in acromegaly


1)headaches
2)excessive sweating
3)exacerbation of carbohydrates control
4)hypertension
5)kardiomegali
6)libido disturbances
7)carpal tunnel syndrome
8)hypoalbuminemia
a)only 8
b)1, 2, 3, 4
c)3, 6, 8
d)7, 8
e)1, 3, 5, 8
A

On the basis of symptoms like: enlargement of the facial cranium, hands and feet,
voice change and gaining weight and night sweats, you are able to diagnose
a)diabetes mellitus
b)adrenal insufficiency
c)liver failure
d)hyperthyroidism
e)acromegaly
e

Acromegaly displays these symptoms:


a) bad teeth
b) sweaty palms
c) deeper vocal cords
d) all of the above
e) none of the above
- D
Choose sign or symptom typical for acromegaly
a)tongue enlargement
b)purple striae
c)buffalo hump
d)feeling of cold
e)exopahtalmus
a

ADRENAL GLAND
Hypoglycemia is not a common feature observed in
a)insulin secreting tumors
b)insulin overdose
c)adrenal insufficiency
d)anterior pituitary insufficiency
e)in the treatment with glucocorticoids
e

Cushing Syndrome
22) Hypertension, central obesity, “buffalo hump”, full-moon face with red cheeks, acne, red
striae are the characteristic features of:
a. Severe obesity
b. Hypothyroidism
c. Addison’s disease
d. Conn’s syndrome
e. Increased level of glucocorticoids
- E

Indicate the FALSE statement about the Cushing's syndrome:


a) may be iatrogenic
b) is caused by the overproduction of corticosteroids
c) is caused by the overproduction of ACTH
d) the characteristic features include: moon face, bull hump, central obesity
e) may cause the secondary form of hypertension
- C
a) yes (is called ICS)
b) yes (either by too much endogenous corticosteroid production or too much
exogenous corticosteroid medication)
c) no (not necessarily, this is typical for Cushing DISEASE)
d) yes
e) yes

Moon-face,bufallo hump,abdominal obesity are typical for


a)Addisons disease
b)Cushing's syndrome
c)Giantism
d)Hypothyroidsm
e)Hemiballismus
- B

Involuntary weight loss might be the manifestation of different diseases. choose false
a)severe chronic COPD
b)malignancy
c)cushing disease
d)addison disease
e)chronic heart failure
c

Among given signs and symptoms indicate those characteristic for cushing’s
syndrome
1.lunar face
2.exophthalmos
3.buffalo hump
4.excessive sweating
5.red purple striae
6.diffuse goiter
a)1, 2, 3, 4, 5
b)1, 3, 5
c)1, 3
d)2, 4, 6
e)2, 6
B
- Cushing syndrome = striae , buffalo hump , lunar face, weight gain

Indicate the set of signs describing patient with Cushing syndrome, the best
a)menstrual irregularities, hirsutism, hypoglycemia
b)hypotension, purple striae, hirsutism
c)diabetes mellitus, acne, respiratory alkalosis
d)edema, hypotension, hirsutism
e)menstrual irregularities, diabetes mellitus, hypertension
e
- hirsutism = excessive body hair. can be caused by excess ACTH in cushing disease.

Addison Disease
Addison's disease is
a)secondary adrenal insufficiency
b)primary adrenal insufficiency
c)primary adrenal hyperfunction
d)secondary adrenal hyperfunction
e)primary parathyroid insufficiency
b

A 40 year old white male complains of weakness, weight loss and abdominal pain. On
examination the patients has diffuse hyperpigmentation and is dehydrated. His blood sugar
is 65 mg/dL and blood pressure 108/64 mmHg. Which of the following is the most likely
diagnosis
a)insulin dependent diabetes mellitus
b)pancreatic carcinoma
c)addison disease
d)hemochromatosis
e)metabolic syndrome
c

ANEMIA
41. Which of the following is a sign of anemia?
a) peripheral cyanosis
b) alopecia
c) bradycardia
d) A & B is correct
e) none of the above
- E

24) Tachycardia, pallor, systolic flow murmur, breathlessness, fatigue, palpitations may be
symptoms of:
a. Asthma
b. Coronary artery disease
c. Peripheral vessel disease
d. Anemia
e. Gastritis
- D

Among listed signs and symptoms of anemia, choose false one


a)skin and mucous tissue discoloration (pallor)
b)dizziness and fatigue on exertion
c)tachycardia
d)systolic murmur over heart and palpitations
e)cyanosis
e: cyanosis is NOT a symptoms of anemia.

Interpret the lab test of patient with chronic infection (diabetic foot) hemoglobin 9.0 g/dL,
hematocrit 28%, mean corpuscular volume 87 fl (low), platelets count of 600 000/ul (high),
elevated serum ferritin, serum iron concentration low normal range
a)megaloblastic anemia
b)anemia of chronic diseases
c)macrocytic anemia
d)anemia with thrombocytopenia
e)none of the above is possible
B

- Ferritin is low in iron deficiency anemia, while high in anemia of chronic disease due
to acting as inflammatory acute phase protein, so serum iron levels are still low.

Which of the following might be a symptom or sign of chronic blood loss anemia
1)angina pectoris
2)dyspnea
3)microcytosis
4)anisocytosis
5)thrombocytopenia
6)hair loss
7)fatigue
a)1, 2, 3, 4, 5, 6, 7
b)1, 2, 3, 4, 6, 7
c)1, 2, 5, 6, 7
d)3, 4, 5, 6
e)1, 2, 5, 6, 7
- B

5. hemolytic hypochromic anemias

15. anemia questions

44. anemia is measured by what Hb, MCV, Ht???

26. megaloblastic anemia - all of the above


Normocytic Anemia

What can cause normocytic anemia


a)renal insufficiency
b)neoplasm
c)acute bleeding
d)a b and c
e)b and c
- D

Microcytic Anemia

Microcytic anemia (MCV<80fl) may be caused by:


a. iron deficiency
b. folate deficiency
c. Vitamin B12 deficiency
d. A and B true
e. A, B and C true
- A

Hypochromic microcytic anemia is characteristic for deficiency of which of those


elements
a)vitamin B12
b)folic acid
c)iron
d)b and c
e)a and b
C

Microcytic anemia is characteristic for


a)iron deficiency
b)myelodysplastic syndrome
c)folic acid defic iency
d)b and c are true
a)b and c are true
A
In anemia caused by chronic bleeding you will find
a)low hemoglobin, iron and ferritin level with low MVC
b) low hemoglobin, with correct iron and ferritin level and high MVC
c)low hemoglobin, iron and ferritin level with macrocytosis
d)low hemoglobin and iron level, high ferritin level with microcytosis
e)high hemoglobin level, with correct iron and ferritin level and low MCV
a → chronic bleeding causes iron deficiency.

Iron deficiency anemia may occur in the course of


1)gastro intestinal tract bleeding
2)pregnancy
3)primary hemochromatosis
4)vegetarian diet
a)1, 3
b)1, 2, 3, 4
c)1, 2, 4
d)only 1
e)2, 3, 4
- C
- 3: primary hemochromatosis is when you absorb too much iron.
A 38 year old woman in a vegetarian diet has been feeling fatigue, has fissures of
mouth zones, spooning of nails, pale skin. A possible cause is
a)gilbert syndrome
b)vitamin B12 deficiency
c)aortic regurgitation
d)hyperprolactinemia
e)-...anemia
E?
Macrocytic Anemia

Vitamin B12 deficiency may be suspected in all EXCEPT:


a. Macrocytic anemia (MCV > 100fl)
b. Autoimmune atrophic gastritis (pernicious anemia)
c. In patients after gastrectomy
d. Microcytic anemia (MCV < 80fl)
e. In patients after small intestine resection
- D

Intrinsic factor deficiency might be the cause of


a)iron deficiency anemia
b)folic acid deficiency anemia
c)vitamin b12 deficiency anemia
d)hemolytic anemia
e)aplastic anemia
c
Patient presenting with signs of anemia with lemon yellow skin color, glossitis, neurologic
abnormalities, atrophic gastritis and low hemoglobin level with macrocytic erythrocytes may
suffer from
a)iron deficiency
b)chronic diseases anemia
c)pernicious anemia
d)folate deficiency anemia
e)aplastic anemia
- C

RED BLOOD CELL DISORDERS


Most common hereditary coagulation abnormality is
a)hemophilia a
b)hemophilia B
c)schönlein henoch disease
d)von willebrand disease
e)disseminated intravascular coagulation
d
Indicate the factor lacking in hemophilia A
a)von willebrand factor
b)activated factor XII
c)factor VIII
d) factor IX
e)vitamin K dependent factors ( II, VII, IX, X)
c

Indicate the states which may predispose to increased bleedings


a)liver cirrhosis
b)hypersplenism
c)sepsis
d)hemophilia
e)all of the above
e

WHITE BLOOD CELL DISORDERS


Chronic leukocytic leukemia:
a) may be associated with hepatomegaly
b) may be associated with thrombocytopenia
c) over 90% of cases are diagnosed in young people younger than 30 years
d) A and B true
e) A, B and C true
- B
- a: splenomegaly
- c: CLL never occurs in young patients.

Mark the FALSE answer about thrombocytopenia


a)might be a complication of heparin use
b)might be a complication of a massive blood transfusion (dilutional thrombocytopenia)
c)signs of bleeding diathesis usually appear at blood count <100000/ul
d)might be a consequence of an autoimmune process
e)thrombocytopenia is defined as platelet count <150000/ul
c
- a: yes, remember we have something called heparin-induced thrombocytopenia
(HIT)
- b: Post-transfusion purpura (PTP) is a rare yet serious disease characterized
by severe thrombocytopenia occurring after a blood transfusion.
- c:
- d: yes, e.g. lupus can destroy platelets leading to thrombocytopenia.
- e: yes. Less platelets than 150’000 = thrombocytopenia.

Agranulocytosis may occur due to


a)genetic disorder
b)some bacterial infection like tuberculosis
c)some viral infections like EBV
d)after chemotherapy
e)all above are correct
e

In the peripheral blood morphology laboratory test you found neutrophil leukocytosis.
In which clinical situation does it occur most typically?
1)HIV infection
2)treatment with cytostatic agents
3)trauma
4)within treatment with glucocorticosteroids
5)in aplastic anemia
6)in acute appendicitis
7)in bacterial pneumonia
8)in addison’s disease
a)correct is 2, 3, 4, 7
b)1, 3, 4, 5, 7
c)3, 4, 6, 7
d)all are correct
e)correct answers 3, 6, 7, 8
c

SKIN + LYMPH NODES


Thick skin on a physical examination could indicate:
a. Scleroderma
b. Myxedema
c. Ichthyosis
d. All of the above
- D

Which pair of sentences about skin colour is false


a)bluish - pulmonary diseases
b)white - albinism
c)brownish - cushing disease
d)redish - lupus erythematosus
e)yellowish - haemolytic anemia
- C

Petechiae - find the correct


a)are---- disappear on applied light pressure
b) are purple-red discoloration of an area less than 0.5 cm .
c)more commonly appear after severe trauma
d) all are localized on the scalp
e)cause discomfort to the patient, because they itch.
- B
Match the systemic diseases with generalized skin colour changes
I.adrenal gland disease, II-polycythaemia, III-liver disease, IV-albinism
1.brown, 2.white,3.yellow, 4.red
a)I-2, II-1, III-3, IV-4
b)I-1. II-4, III-3, IV-2
c)I-1, II-2, III-3, IV-4
d)I-3, II-4, III2, IV-1
e)I-4, II-3, III-1, IV-2
B
- Adrenal gland disease = brown
- Polycythemia = red
- Liver disease = yellow
- Albinism = white
Décolletage symptom is characteristic for:
a)neck skin melanoma
b)chronic obstructive pulmonary disease
c)sailors
d)systemic lupus erythematosus
e)there is no such symptom
- E
- assa … ? stefanski whats up

When can you observe terrys nails?


a. Iron deficincy
b. Liver cirrhosis
c. DM
d. Psoriasis
e. b and c is correct
- E

Clubbing fingers may occur in


a)diabetes type 2
b)atelectasis
c)liver cirrhosis
d)b and c
e)all of above
- d

Clubbing fingers are associated with:


a) Respiratory disease
b) Rheumatoid arthritis
c) Dupuytren's contracture
d) Cardiovascular diseases
e) a and d are correct
- E
Dehydration

Signs of dehydration
a)dense salive
b)hypotonia
c)bradycardia
d)elastic skin
e)confusion (especially in elderly)
- a,b,e??

Which of the following are the symptoms of dehydration


1 hypotonia
2. tachycardia
3. dry mucosa
4. paleness
5. bradycardia
a)2 and 4
b)1 ,3, 5
c)1, 4
d)1, 2, 3, 4
e)1, 2, 3
e

Cyanosis

When looking for cyanosis one should inspect?


a) Large body parts
b) Body sites containing minimal melanotic pigments
c) Body sites that have a capillary bed close to the skin surface
d) b and c are correct
e) All are correct
- D

Central cyanosis
a)is caused by insufficient oxygenation of the blood in the lungs
b) the best way is assessment of the skin around the umbilicus
c)is more intense in the case of a severe anemia (hemoglobin <5 g/dl)
d) all answers are correct
e) only answers a and b are correct
a

Where should you look for cyanosis?


a) lips
b) nail beds
c) Eyes
d) A and B is correct
e) A, B and C is correct
- D

Wich of these may result in increased skin temperature?


a) Inflammation
b)Hyperthyroidism
c) Connective tissue disease
d) a and b are correct
e) All are correct
- E

T/F: thickened skin occurs in people with myxdema.


- T

Find the false statement about petechiae


a)they lose their color (blanch) after pressing on them
b)they might be a sign of hemorrhagic diathesis
c)they are caused by extravasation of blood into the skin or mucosa
d)they are red, brown or purple
e)they might look similar to a rash
A

Raynaud sign is
a)paroxysmal blanching of fingers and toes due to exposure to cold
b)redness and warmth of limbs particularly toes, accompanied by intense burning pain
c)blue punk mosaic pattern on the skin of the limbs
d)linear redness of the skin that between the infected site with nearby lymph nodes
e)none of the above
A

Ascites
Ascites can be caused by
a)liver cirrhosis with portal hypertension
b)malignant tumor within the abdominal cavity with the metastases to the peritoneum
c)right ventricular failure
d)hypoproteinaemias
e)all above answ
ers are correct
- E

Choose disorders which may lead to ascites


1.hepatic cirrhosis
2.metastases of ovary cancer in peritoneum
3)right ventricular heart failure
4)intestinal obstruction

a)1, 3
b)1, 4
c)1, 2, 3
d)1, 3, 4
e)all answers are corret
c

Indicate the possible causes of ascites


1)left ventricular failure
2)severe nephrotic syndrome
3)portal hypertension in liver cirrhosis
4)peritonitis?
a) 1, 2, 3
b) 2, 3, 4
c) 1, 3, 4
d) all of above
b

Patient with massive ascites might present


a)increased abdomen circumflex
b)dull sound upon percussion of abdomen
c)sign of fluid wave
d) decreased peristalsis during auscultation
e)all of above
e

Lymph Nodes
During lymph nodes examination a node which is enlarged, soft , painful, single and
movable under the skin with red, warm skin over the node area suggests
a)lymphatic system malignancy
b)metastatic changes in lymph node with unknown primary point
c)infection of the node or in the lymph node area
d)old -- cured TB infection
e)none of above
c

Which is not characteristic for an inflamed lymph node


a)lymph node is painful upon palpation
b)lymph node is unmovable
c)....
d)skin upon the lymph node is changed
e)lymph nodes does not make ….
B

Lymph nodes in cancerous disease are


a)painless
b)firmly fixed to underlying tissue
c)hard
d)skin over them is not changed
e)all of above
e

During palpation, lymph nodes that are enlarged due to an infection can be
a)painless, fixe, firm
b)painful, movable, soft
c)painful movable, firm
d)painless, fixed soft
e)painless movable, firm
b

HEART
An 80 year old with chronic heart failure is admitted due to worsening dyspnea. Chest x ray
revealed congestion in pulmonary vasculature. While looking for the cause of the worsening
dyspnea you will consider
a)infection
b)acute coronary syndrome
c)reduced doses of diuretics
d)aortic dissection
e) a b and c are correct
e?

Indicate false answer regarding pulse examination


a)pulsus parvus et tardus is present in aortic stenosis
b)hyperkinetic pulse is characteristic for ventricular fibrillation
c) x
d)weak pulse is characteristically seen in patients with shock
e)pulsus paradoxus occurs in cardiac tamponade
B

- a:

- b: No
- c:
- d: Yes, a weak pulse can be due to low CO, seen in shock.
- e: Yes, pulsus paradoxus (drop in BP during inspiration) can be seen in cardiac
tamponade.

Match the following valve defects with corresponding auscultation findings


1)aortic stenosis
2)aortic regurgitation
3)mitral stenosis
4)mitral regurgitation

a)opening snap
b)mural …
c)pulse… water hammer pulse
d)systolic murmur radiating to the left axilla

1)1a, 2b, 3c, 4d


b)1b, 2c, 3a, 4d
c)1c, 2b, 3d, 4c
d)1d, 2b, 3a, 4c
e)1b, 2a, 3d, 4b
b
- Aortic stenosis =
- Aortic regurgitation = water hammer pulse
- Mitral stenosis = opening snap
- Mitral regurgitation = systolic murmur radiation to left axilla

Aortic valve is auscultated in the


a)left 2nd intercostal space
b)right 5th intercostal space
c)left 5th intercostal space
d)left 5th intercostal space
e)right 2nd intercostal space
e

Area for auscultation of the pulmonic valve is located at


a)2nd right intercostal space at right sternal border
b)3rd left intercostal space at left sternal border
c)2nd left intercostal space at left sternal border
d)4th left intercostal space at sternal border
e)2nd right intercostal space at the midclavicular line
c

Match the percussion tones with example there they might be heart
I-tympanic, II-dulll, III-resonant, IV-hyper resonant
1-muscle, 2-gastric bubble, 3-healthy lung, 4-emphysematous lung
a)I-1, II-4, III-3, IV-3
b)I-1, II-2, III-3, IV-4
c)I-2, II-1, III-3, IV-4
d)I-3, II-4, III-2, IV-1
e)I-2, II-3, III-4, IV-1
c
- Muscle = dull
- Gastric bubble = tympanic
- Healthy lung = resonant
- Emphysematous lung = hyper-resonant

The 2nd heart sound (S2) – choose FALSE sentence:


a) is the result of closure of the semilunar valves
b) indicates the end systole
c) is best heard in the aortic and pulmonic areas
d) splitting is greatest at the peak of expiration
e) is increased in systemic hypertension
- D
- 2nd heart sound = closure of aortic + pulmonary valve (semilunar valves) at the end
of systole. The difference in timing between the closure of these 2 valves, creates a
split second heart sound. This can be heard during inspiration.
89 year old woman with hypertension, known acquired valve defect is presenting with
angina, dyspnea, as well as episodes of consciousness, loss on exertion. Based on the data
above point the signs that would typically be present in the valve defect the patient most
probably suffers from
a)systolic murmur over aorta and apex radiating to carotids, with diminished S2
b)systolic murmur over pulmonary trunk and diastolic over mitral valve with shift of apical
impulse to the right
c)end systolic murmur over aorta with augmented S2 and water hammer pulse
d)loud diastolic murmur over mitral valve with loud first tone and opening click
e)austin flint murmur and systolic murmur over aorta
a?

Optimal first line no pharmacological treatment of cardiovascular diseases should


include
a)cessation of tobacco use
b)change of diet
c)weight loss
d)systemic physical exercise
e)all of the above
e

Select the true answer


a)nausea and vomiting may accompany inferior wall myocardial infarction
b)anginal pain is relieved with nitroglycerin
c)exacerbation of angina is dependent upon body position
d)the correct is a and c
e)the correct is a and b
e

Systolic murmur is present during auscultation of heart valves in the case of


1)aortic stenosis
2)mitral stenosis
3)aortic regurgitation
4)mitral regurgitation
5)tricuspid valve stenosis
a)2, 5
b)1, 4
c)3, 4
d)1, 2
e)2, 3
b
- Systolic murmur = aortic stenosis, mitral regurgitation, patent ductus arteriosus
- Diastolic murmur = aortic regurgitation, mitral stenosis, patent ductus arteriosus
Indicate the proper places of heart valves auscultation:
1. mitral valve
2. tricuspid valve
3. aortic valve
4. pulmonic valve
a. second right intercostal space at the right sternal border
b. second left intercostal space at the left sternal border
c. fourth left intercostal space along the lower left sternal border
d. apex of the heart, usually in the fifth left intercostal space at the midclavicular line

1. 1- A, 2-C, 3-D, 4-B


2. 1-B, 2-C, 3-A, 4-D
3. 1-D, 2-C, 3-A, 4-B
4. 1-D, 2-B, 3-A, 4-C
5. 1-C, 2-A, 3- B, 4-D
- 3
- Aortic = right 2nd
- Pulmonary = left 2nd
- Tricuspid = left 4th
- Mitral = left 5th

Heart Failure

7. 50-year-old patient complains of: nycturia, ankle region edema particularly in the
evenings, pain in the right subcostal area, worsening of physical activity tolerance. Most
probable reason of his condition is:
a) viral hepatitis B
b) hemochromatosis
c) right sided heart failure
d) hypertension
e) chronic renal failure
- C

Which symptom is less likely to appear in right-side heart failure?


a. Peripheral edema
b. Hepatomegaly
c. Anorexia
d. Orthop Nea
- D

Indicate the signs of right ventricular failure:


a. weight gain, ankle edema, hepatomegaly, hydrothorax, carotid veins distention
b. dyspnea, fast heart rate, hemoptoe, dizziness on exertion
c. atrial fibrillation, systolic murmur best heard on the apex, carotid veins distention
d. left ventricle enlargement, aortic distention, systolic murmur best heard over renal
artery
e. high blood pressure, systolic murmur over apex, tachycardia
- A

Which is NOT a sign of left congestive heart failure


a) pulmonary hypertension
b) left ventricle hypertrophy
c) dyspnea
d) pulmonary edema
e) peripheral edema
- E

Among the signs of right sided heart failure are


a)peripheral edema
b)chest pain
c)jugular vein distension
d)a and c are correct
e)all of the above
- E

Which is not a typical symptom of (chronic?) right side heart failure


a)lower limbs edema
b)ascites
c)hepatomegaly
d)dyspnea
e)nocturia
d

The main symptom in left ventricle heart failure is


a)peripheral edema
b)ascites
c)dyspnea
d)chest pain

e)all above symptoms are typical for left ventricle heart failure
- C

Indicate the symptoms not characteristic for right ventricle heart failure
a)edema
b)hepatosplenomegaly
c)pulmonary edema
d)ascites
e)weight gain
c
Acute left ventricular heart failure is manifested by:
a. liver enlargement
b. ankle oedema
c. pulmonary oedema
d. A and B are true
e. A,B,C are true
- C

Common signs of RS and LS heart failure is/are:


a. Nocturia
b. S3 gallop
c. hydrothorax
d. All of the above
- D

Neck vein distension, pulmonary edema, positive hepatojugular reflux, S3gallop and
rales on auscultation might be the symptoms of the following state:
1. Chronic Obstructive Pulmonary Disease
2. Heart failure
3. Arterial hypertension
4. Isolated pulmonary hypertension
5. Arteriosclerosis obliterans
- 2

In oliver Cardarelle sign the is no diagnos for


a. Mediastinal tumor
b. Aortic regurgitation
c. Aortic aneurysm
d. Heart Failure
- D

What is the most likely reason for a transudative pleural effusion?


a) Typical pneumonia
b) Atypical pneumonia
c) Tuberculosis
d) Left heart failure
e) Pulmonary embolism
- D

Paroxysmal nocturnal dyspnea is a frequent manifestation of:


1. Arterial hypertension
2. Lung cancer
3. Pleural mesothelioma
4. Heart Failure
5. Emphysema
- 4
person who has heart failure can have peripheral, centralized edema from what?

50 year old patient complains of nocturia, ankle region edema particularly in the evenings,
pain the the right subcostal area, dyspnea on walking up to the first floor and during the
night. Most probable reasons of his condition is
a)viral hepatitis B
b)hemochromatosis
c)congestive heart failure
d)hypertension
e)chronic renal failure
c

Which of the following is NOT a cause of secondary hypertension


a)renal artery stenosis
b)heart failure
c)acromegaly
d)aortic coarctation
e)primary hyperaldosteronism
B
NYHA

Can walk 20-100 meters, NYHA = 1,2,3,4 or lol they should use the Canadian CCS
scale instead of NYHA.
- I would go for class 3 CORRECT!

According to NYHA classification, in which class would a patient with marked


limitation of physical activity be in?
a. NYHA klass I
b. NYHA class II
c. NYHA class III
d. NYHA class IV
- C

NYHA scale is describing?


- Heart failure classification

Coronary Artery Disease


Which of the following symptoms is not associated with Coronary Artery Disease?
A) Shortness of breath
B) Radiation to back and/or jaw
C) Pulsus Paradoxus
D) Nausea
E) Weakness
- C → Drop in blood pressure during inhalation.

Which of the following states IS NOT a risk factor for coronary artery disease:
1. hypertension
2. high LDL cholesterol
3. high HDL cholesterol
4. diabetes
5. smoking
- 3

Coronary artery disease (CAD) main symptom is:


a. claudicatio intermittens
b. retrosternal pain
c. transient ischaemic attack (TIA)
d. A and B are true
e. A,B,C are true
- B

Chest discomfort on exertion radiating to the jaw, neck, left arm, dyspnea, nausea and
palpitation are characteristic features of:
a. left ventricular failure
b. left ventricular hypertrophy
c. myocarditis
d. coronary artery disease
e. mitral valve stenosis
- D

A 70 year old male smoker with hypercholesterolemia and hypertension, reports a few
months history of retrosternal, burning chest pain induced by exertion (climbing II floor,
walking distance around 1000m) with shortness of breath, which symptoms subside at rest.
The most probable diagnosis is
a)dissection of aorta
b)acute myocardial infarction
c)heart failure NYHA IV
d)angina pectoris CCS II
e)angina pectoris CCS IV
D
Angina pectoris
a)is caused by disproportion between the demand and the supply of the heart’s muscle the
oxygen
b)it is not connected with the breathing any body position
c)most commonly it is retrosternal
d)usually it appears after physical activity, psychological stress or it might be exacerbated by
cold air
e)all answers are correct
e

Variant (prinzmetal)angina
a)is always induced by physical exertion and stress
b)always cause ST depression
c)is caused by the rupture of the plaque in the coronary artery
d)can appear at rest
e)none of the above is correct
D
- b: no, will rather cause ST elevation.

Myocardial Infarction
Earliest serum marker of myocardial infarction to increase is?
a.CK-MB
b.Troponin
c.Myoglobin
d.Glutamic oxaloacetic transaminase
e.Lactic dehydrogenas
- C

Patient comes to the ER with retrosternal pain radiating to the jaw and left upper limb.
Which test should be the first that you perform
a)troponin level
b)full blood count
c)chest angio CT
d)D-dimer level
e)chest X ray
a

Indicate characteristic features of myocardial infarction


a)severe retrosternal pain radiating to jaw, not relieved with sublingual nitroglycerin
b)
c)retrosternal pain with new left bundle branch block
d)answers a and b are correct
e) all of above mentioned are correct
E

One of the complications of myocardial infarction is


a)ventricular septal rupture
b)papillary muscle rupture
c)acute mitral regurgitation
d)ventricular fibrillation
e)all of the above are correct
e

ST segment elevation in leads I, aVL and V… most likely indicates


a)lateral wall STEMI infarction
b)inferior wall STEMI infarction
c)pericarditis
d)inferior wall NSTEMI infarction
e)anteroseptal STEMI infarction
A
Aortic Stenosis
Murmur in aortic stenosis is best heard:
a. During systole in IV intercostal space on left sternum border
b. During systole in II right intercostal space on the sternum border
c. During diastole in II left intercostal space on the sternum border
d. During diastole in V left intercostal space in midclavicular line
e. During systole in IV left intercostal space on the sternum border
- B

Symptoms of the aortic stenosis are


a)systolic murmur that radiates to the neck
b)apex beat spilled, moved to the left side and inferiorly
c)small and lazy pulse
d) correct answer a and b
e)correct a, b, c
e

A typical symptom of aortic stenosis is


a)chest pain radiating along the sternum to the jaw
b)... dyspnea
c)syncope
d)correct b and c
e)all are correct
e

60 year old woman is seen in ER due to recurrent episodes of collapse. She has a long
history of hypertension, hyperlipidemia. On physical examination blood pressure is 120/85,
HR 110 irregular, weak pulse on the radial arteries. On auscultation there is a systolic
murmur present over the apex and second, right intercostal space. There is a murmur
present over the carotid arteries. The most probable diagnosis
a)atrial fibrillation caused by mitral stenosis
b)aortic valve stenosis
c)hyperthyroidism
d)ventricular tachycardia
e)pulmonary edema
b

5)Aortic Stenosis
a) creates a systolic murmur
b) creates a diastolic murmur
c) best heard at the in 2nd right intercostal space
d) A&C are correct
e) all are correct
- D

Coarctation of Aorta
Choose correct findings for coarctation of the aorta
a)hypertension, reduced BP in the upper extremities, increase BP in the lower extremities,
systolic murmur, left ventricle hypertrophy
b)hypotension, increased BP in the upper extremities, reduced BP in the lower extremities,
systolic murmur, left ventricle hypertrophy
c)hypertension, reduced BP in the upper extremities, increasing BP in the lower extremities,
diastolic murmur, left ventricle hypertrophy
d)hypotension, increasing BP in the upper extremities, reduced BP in the lower extremities,
diastolic murmur, left ventricle hypertrophy
e)hypertension, increased BP in the upper extremities, reduced BP in the lower extremities,
systolic murmur, right ventricle hypertrophy
e
- CoA → Hypertension, high BP in arms, low BP in legs
Pericarditis
Sharp retrosternal pain, getting stronger when the patient lies in supine position and
softens when sitting up and leaning forward, with fever and pericardial friction rub
upon auscultation is characteristic for
a)myocardial infarction
b)unstable angina
c)pericarditis
d)gallstones
e)lobar pneumonia
c → Pericarditis’ pain is often relived when leaning forward and you can hear this
pericardial friction rub.

4)A woman presents to the hospital with excess fluid in her pericardium (pericardial
effusion) which of the following options is not a possible mechanism of her condition:
a) Inflammatory connective tissue disease
b) Cancer
c) Left ventricular hypertrophy
d) Viral infection
e) None of the above
- C

2)Sharp pain in precordial area, increases during respiratory movement or left side
position is an indication for:
a) angina pectoris
b) prinzmetals angina
c) myocardial infarct
d) percarditis
- D

Bacterial Endocarditis
Newly diagnosed heart murmur, high fever and shivers, hemiparesis may suggest
a)acute pancreatitis
b)bacterial endocarditis
c)mitral valve prolapse
d)acute glomerulonephritis
e)exudative pericarditis
b

Subacute bacterial endocarditis in a susceptible host may be prevented by:


1. replacement of diseased valves
2. use of antibiotics before and after dental surgery
3. antibiotic prophylaxis throughout adolescence
4. antibiotic prophylaxis for genitourinary tract instrumentation
The correct answer is:
a) only 2 b) 1, 2 c)2, 4 d) 3, 4 e) none of the above
- E → think it’s none of the above bc best prevention (more effective than antibiotic
prophylaxis) is to just have good oral hygiene everyday.

A new murmur of the heart may be suggestive of


a)endocarditis
b)complication after myocardial infarction
c)valve degenerative disease - quite common among elderly
d)hyperkinetic circulation state (pregnancy, fever etc)
e)all above correct
E

A 83 year old male for the last 3 months reports having fever up to 40 degrees, weight
change (loss 6kg, current weight 66kg, height 173 cm). Which disease may cause the
symptoms
a)infective endocarditis
b)tuberculosis
c)non-hodgkin lymphoma
d) a and b
e) a , b, and c are correct
e?
Which of the following findings may suggest bacterial endocarditis
1)new heart murmur
2)osler nodules
3)erythrocyturia (presence of red blood cells in urine)
4)janeway lesion
5)pulsus paradoxus
a)1, 3
b)1, 4
c)1, 2, 3
d)1, 2, 3, 4
e)all of above
d

Aortic Aneurysm

An elderly man has an abnormal pupillary response (argyll robertson pupil). Select
the cardiovascular disorder with which this clinical scenario is most likely to be
associated
a)massive tricuspid regurgitation
b)aortic aneurysm and aortic valve regurgitation
c)coarctation of the aorta
d)severe renal artery stenosis
e)myocarditis
b

The argyll Robertson pupils are caused by syphilis:

Aortic Dissection
37: Q - Pain radiates to the back typically in which of the following conditions?
A) Aortic dissection
B) Pleurisy
C) Angina
D) Myocardial Infarction
E) Endocarditis
- A

In a patient with dissecting aortic aneurysm (thoracic…..abdominal)indicate the possible


clinical presentations
1)pre.. acute renal failure
2)pain in the chest
3)pericardial effusion
4)differences of arterial blood pressure measured on both arms
5)TIA
6)pleural effusion
1)1, 2, 4
b)2, 4, 5
c)1, 2,5, 6
d)1, 2, 3, 4, 5, 6
e)1, 2, 6
- D

Aortic Regurgitation
1)Pulsus disferiens is seen in:
a) aortic stenosis
b) Pericarditis
c) aortic regurgitation
d) Tricuspid regurgitation
- CC

Indicate false statement about physical examination of a patient with regurgitation


a)holosystolic, plateau shaped intensity, high pitch murmur
b)murmur transmitted into left axilla
c)murmur heard best at apex
d)rumbling murmur at apex (austin flint murmur)
e)s1 intensity diminished
d?

What are the signs of aortic regurgitation


a)corrigan pulse
b)de mussat sign
c)Quincke sign
d)high pulse pressure
e)all mentioned above
e

Mitral Regurgitation + Mitral Valve Disease


Tetrology of Fallot characterized by all the following except:
a.Ventricular septal defect
b.Mitral regurgitation
c.Pulmonary stenosis
d.Aorta overriding
e.Right ventricular hypertrophy
- B

In mitral regurgitation radiates to


a)right groin
b)left groin
c)carotid arteries
d)right armpit
e)left armpit
- E

5. What are the typical physical examination findings in mitral stenosis:


a) loud, high pitch systolic murmur, radiating to apex and left axilla, S1 intensity diminished,
S3 often present
b) early diastolic, rumbling murmur at apex (Austin - Flint), apical impulse shifted down and
to the left, pulse pressure water-hammer
c) quiet diastolic murmur, with increase in late diastole, opening snap, best heard at apex,
d) midsystolic murmur, diamond shaped, radiates down sternal border and to carotid with
palpable thrill
e) none of the above
- C
mitral stenosis:
- mid-diastolic murmur
- best heard at apex of heart when patient is rolled over on their left side
- characterized by “opening snap”
a: systolic murmur radiating to left axilla → mitral regurgitation
b: rumbling = austin-flint murmur, water-hammer pulse → aortic regurgitation
d: diamond shape + radiates to carotid aa → aortic stenosis
57 year old patient was admitted due to significant ST elevation myocardial infarction. On
the 3rd day of the hospital stay he developed acute pulmonary edema with hypotension,
upon heart auscultation a loud systolic murmur over the apex is audible which was not
present the day before. You suspect
a)acute mitral regurgitation due to the rupture of papillary muscle as a complication of MI
b)acute mitral regurgitation due to the rupture of chordae tendineae as a complication of
endocarditis he must have had developed
c)massive aortic regurgitation as a complication of MI
d)aortic dissection
e)pulmonary embolism
a → mitral regurgitation causes pulmonary edema, orthostatic hypotension,
ST-elevation + mitral valve is located in the apex area.

The chest X-Ray with dominant left ventricle is a common finding in the following
abnormalities except for:
1. Severe mitral valve prolapse
2. Arterial hypertension
3. Aortic stenosis
4. Aortic regurgitation
5. Mitral stenosis
- 5

Facies mitralis is characteristic for:


1. acromegaly
2. chronic heart failure
3. mitral valve disease
4. Cushing disease
5. aortic valve disease
- 3

A patient with advanced mitral valve disease may present with:


a. dyspnoe
b. atrial fibrillation
c. facies mitralis
d. A and B are true
e. A, B and C are true.
- E
6)Mitral Stenosis
a) creates a systolic murmur
b) creates a diastolic murmur
c) best heard at the in 5th left intercostal space (apex)
d) A&C are correct
e) B&C are correct
- E

33.) Rheumatic fever most often causes which valvular defect?


a.Mitral stenosis
b.Pulmonary stenosis
c.Pulmonary regurgitation
d.Subaortic stenosis
e.Tricuspid regurgitation
- A

*47. Redness of face is seen in:


a) Alcoholism
b) Pregnancy chloasma
c) Mitral stenosis
d) a and c are correct
e) a, b and c are correct
- D

55.Indicate the correct statement?


a) Mitral stenosis leads to mild cyanosis of lips, cheeks and malar prominences.
b) Does not lead to clubbing of the fingers.
c) Oftens seen in patients with mitral stenosis.
d) a and c are correct
e) All are correc*
Indicate false statement
a)mitral stenosis is generally rheumatic origin
b)⅔ of all patients with mitral stenosis are male
c)in mitral stenosis s2 is accentuated and snapping
d)mitral regurgitation may be a result of myocardial infarction
e)the graham steell murmur is a systolic murmur of pulmonary regurgitation

a) BCE
b) BC
c) A, D
d) A, E
e) E
- A
- b→ more common in women ⅔
- c→ S1
- e→ a diastolic murmur in pulmonary regurgitation

41. Mitral stenosis murmur can be heard after s1 before s2 and is systolic/diastolic, after s2
before s1 and its systolic/diastolic

28. Apical impulse in mitral valve stenosis is:


1. strong and lifting
2. Displaced to the left and downwards
3. Displaced to right and upwards
4. A and B are true
5. None of the above is true X

Pulmonary Embolism
Indicate the true statement (s) of pulmonary embolism?
a) Usually arises from the thrombi from lower extremities
b) One of the respiratory consequences is Hypoxemia
c) Most common clinical sign is Tachycardia
d) a and b are correct
e) All are correct
- D

Pulmonary embolism usually arise from thrombi originating from:


a. Lower extremity deep veins
b. Pelvic veins
c. Upper extremity veins
d. Renal veins
- A

Shortness of breath, pleuritic pain, tachypnoe, tachycardia, swollen, red painful lower
limb might suggest:
a. Broken leg
b. Pneumonia
c. Hydrothorax
d. Pulmonary embolism
e. Acute coronary syndrome
- D

A man after knee surgery at bed rest stands up with chest pain. Orthostatic
hypotension. What is it?
- pulmonary embolism

A… year old female went to ER due to haemoptysis and strong pain in upper gastric region.
She had a history of coronary disease and hormonal replacement therapy. Upon physical
examination she had central cyanosis. Most probable diagnosis is
a)perforation of gastric ulcer
b)pulmonary embolism
c)pneumonia
d)myocardial infarction
e)acute pancreatitis
b

55 yo man without any chronic disease in the medical history, on day after long journey from
the other continent, was transferred by ambulance to emergency department because of
sudden onset of dyspnea and central cyanosis. In the physical examination after oxygen
therapy with no sign of cyanosis, tachycardia 115/min, RR 95/55 mmHg upon auscultation of
the lungs symmetrical vesicular sounds, upon auscultation of the heart - heart sounds loud,
clear and properly accentuated. What is the most probable reason of the patient’s state
a)acute thrombus pulmonary embolism
b)anaphylactic shock
c)myocardial infarction
d)heart failure de novo
e)viral infection causing acute respiratory insufficiency
a

If you suspect pulmonary embolism most important diagnostic tool should be


a)chest angio ct scan
b)level of C-reactive protein
c)level of D dimmers
d)lungs ultrasound
e)chest X-ray
c

The laboratory marker… pulmonary embolism


a)D-dimer
b)FEV1
c)creatinine
d)hemoglobin
e)blood culture
a

Differential diagnosis of sudden onset dyspnea, chest pain, hypotension and…. will
include
a)pulmonary embolism
b)myocardial infarction
c)massive pneumothorax
d)a and b
e)all answers correct
e

79 year old woman with a history of deep vein thrombosis has experienced fracture of pelvis
last month. Now she has pleural chest pain and dyspnea, blood pressure is 90/65, pulse
regular 130/min (no pulse deficit) respiration 26/min. What condition should be considered
first
a)asthma
b)ischemic stroke
c)acute pulmonary embolism
d)left ventricle failure
e)atrial fibrillation
c

Deep Vein Thrombosis

Deep vein thrombosis risk factors are:


a. immobilisation
b. oral contraceptive use
c. malignancy
d. A and B true
e. A,B,C true
- E

Arrhythmia
49 year old female patient, with a history of asthma, went to ER because of the feeling of
heart palpitations lasting few hours. Upon ECG she was diagnosed with atrial fibrillation with
heart rate of 140/min. The cause for arrhythmia might be
a)hyperthyroidism
b)overdose of beta mimetic medication
c)enlargement of left atrium
d)hypokalemia
e)all of the above
E

Atrial Fibrillation
Patient comes with sudden dyspnea, heart palpitations, stabbing chest pain. In ECG
heart rate is irregular, about 120/min, with no p waves, narrow qrs complexes and no
changes in ST segment. What is the probable diagnosis?
a)exacerbation of asthma
b)acute coronary syndrome
c)infective endocarditis
d)episode of atrial fibrillation
e)cardiac tamponade
d → No P-waves and irregular heart rhythm = atrial fibrillation.

Choose the best sentence to describe atrial fibrillation


a)tachycardia, regular irregularity, ECG tracing with regular negative p waves in I, aVR, and
V1 with and isoelectric line between p waves
b)the absence of clearly distinguishable p waves and irregular RR intervals
c)clearly defined P waves of similar configuration with variable RR intervals
d)RR intervals with slow phasic variations with respiration, PQ interval elongation
e)none of the above
- B

Atrial fibrillation can be caused by


a)hyperthyroidism
b)coronary artery disease
c)electrolyte imbalance
d)heart valve dysfunction
e)all of the above are correct
e

Possible causes for atrial fibrillation are


a)mitral valve diseases
b)pheochromocytoma
c)hypothyroidism
d)a and b
e) a b and c
d

Pulse deficit is characteristic for


a)cardiac tamponade
b)aortic coarctation
c)atrial fibrillation
d)subclavian steal syndrome
e)buerger syndrome
- C

Chest Pain
Differential diagnosis of chest pain might include:
a) Mitral valve prolapse
b) Hiatal hernia
c) Pneumothorax
d) a &c
e) all of the above
- E

Chest pain intensifying during breathing occurs in:


a)myocardial infarct
b)pleurisy
c)left ventricle failure
d)onset of asthma
e)herpes zoster
- B

in a patient with sudden chest pain and dizziness and history of severe hypertension,
indicate most important differential diagnosis
1)dissection of aorta
2)bicuspid aortic valve
3)esophageal rupture
4)myocardial infarction
5)acute pneumthorax
a) 1, 2, 3
b) 1, 3, 5
c) 1, 3
d) 2, 4
e) 1, 4
- 1,

Cardiac Tamponade

Upon physical examination you find a triad of symptoms:distended neck veins,


hypotension, muffled heart sounds. It appears in the case of
a)left side heart failure
b)portal hypertension
c)acute abdomen
d)cardiac tamponade
e)upper vena cava syndrom
d

Paradoxal pulse is seen


a) cardiac tamponade
b) pulmonary embolism
c) asthma
d) all of the above
- D
Characteristic symptoms for cardiac tamponade include
a)paradoxical pulse
b)increased BP
c)bradycardia
d)collapsing of jugular veins
e)a and c are correct
- A

A 55 year old man presents to the ER with marked dyspnea. On physical examination;
tachycardia, paradoxical pulse, fast heart tones (?) and a bP of 80/50 mmHg. Key ecg
findings are low voltage and beat-by-beat R wave amplitude changes. The most probable
diagnosis
a)cardiac tamponade
b)aortic dissection
c)acute coronary syndrome (ST elevation)
d)acute pericarditis
e)myocardi
- A

AV Block

Atrioventricular block first degree, choose correct answer


a)ventricles are placed by escape pacemaker
b)causes no subjective symptoms
c)cannot be identified in ecg
d) a and b
e)b and c
B
The most probable cause for very slow regular heart rhythm in a patient who experienced
sudden consciousness loss after mitral valve replacement surgery is
a))ventricular fibrillation
b)right bundle branch block
c)IIIrd degree AV block
d)atrial fibrillation
e)first degree atrioventricular block
e?

ECG
Find the false statement
a)by checking the patient’s skin turgor you can look for signs of dehydration
b)positive goldflam sign might indicate the presence of pyelonephritis
c)tachycardia can be secondary to fever
d)fine crackles/rales can be heard over the bases of lungs in illnesses such as heart failure
e)on an ecg the presence of negative P waves in lead aVF confirms that the heart’s rhythm
is the sinus rhythm.
e

10. Name the pathology in the ECG below (lead III):

11. Show the ECG features characteristic for left atrial hypertrophy :
a) M shaped P wave in lead II
b) P wave duration > 0.11 seconds
c) terminal negative component to the P wave in lead V1
d) only a and b are correct
e) all of the above are correct
- E?

Electrocardiogram ECG nr 2 below shows


a)normal ecg
b)left bundle branch block
c)atrial fibrillation
d)right bundle branch block
e)atrioventricular block second degree
a?

ECG nr 3 shows
a)left bundle branch block
b)atrial fibrillation with rapid ventricular rate
c)normal sinus rhythm
d)ventricular fibrillation
e)atrial fibrillation with bradycardia
b?

ECG (nr 1) shows


1)evolution of anterior myocardial infarction
2)posterior myocardial infarction
3)supraventricular extrasystole
4)digoxin poisoning
5)left bundle branch block
a)2, 3
b)1
c)2, 5
d)4
e)1, 3
e?
*Cant see ECG but learn how third degree av block looks like*

Indicate the correct diagnosis on the basis of ECG tracing


a)RBBB
b)LBBB
c)inferior wall of myocardial infarction
d)II degree AV block
e)atrial fibrillation

Picture of ECG is not in test bu learn about: Afib, LBBB, RBBB, lateral wall infarction, sinus
arrhythmia, Identify them

Widening of QRS complexes of 0.12s can occur in


1)left bundle branch block
2)right bundle branch block
3)... degree atrioventricular block
4)right ventricular hypertrophy
5)ventricular tachycardia
a)1, 2, 3
b)1, 2, 5
c)1, 2, 3, 5
d)1, 2, 4, 5
e) none of above
- B
HYPERTENSION
Hypertension may be diagnosed if a patient with repeated measurements has blood
pressure over:
1. 120/75
2. 160/70
3. 170/100
4. 115/95
5. 2, 3, 4 are true.
- 5

What are the routine tests in hypertensive patients


a)electrocardiogram
b)serum creatinine
c)ankle brachial BP index
d)echocardiogram
e)glucose tolerance test
f)fasting plasma glucose
1)a, b, f
2)a, b, e
3)a, c, f
4)b, d, e
5)b, c, d
1?

Values of systolic blood pressure 120-129 mmHg and diastolic blood pressure 80-84
mmHg are
a)optimal pressure
b)high correct pressure
c)correct pressure
d)first degree arterial hypertension
e)isolated systolic hypertension
c
Indicate the false answer about causes of secondary hypertension :
1. renal artery stenosis
2. hyperthyroidisms
3. genetic predisposition
4. parenchymal renal diseases
5. Cushing’s syndrome and disease
- 3

Secondary hypertension can occur in all diseases except


a)kidney diseases
b)liver diseases
c)suprarenal gland disease
d)pituitary gland disease
e)aortic diseases
b

Indicate the false answer about possible causes of secondary hypertension


a)hypothyroidism
b)hyperthyroidism
c)hypercortisolism
d)acromegaly
e)primary hypoaldosteronism
e

All of the following statements about hypertension are true EXCEPT:


a. In the absence of target organ damage, patients with severe hypertension do not require
immediate blood pressure reduction
b. Severely elevated blood pressure is often discovered incidentally without symptoms of
specific physical findings
c. In case of kidney disease the blood pressure should not fall below 140/90 mmHg
d. Hypertension may be diagnosed if blood pressure level exceeds SBP 140 or/ and DBP
90 mmHg
e. Aortic dissection may be the presenting manifestation of a hypertensive emergency
- C → if you have kidney disease it should be under 140/90

Hypertension can be caused by


1)cushing syndrom
2)hypothyroidism
3)renal diseases
4)adrenal insufficiency
5)hyperthyroidism
a)1, 2, 3, 5
b)1, 3, 5
c)1, 2, 3, 4, 5
d)1, 2, 4
e)3, 5
a
- 4: adrenal insufficiency = too little cortisol = the opposite of cushing = low BP.
Resistant hypertension might be diagnosed when
a)you cannot achieve correct blood pressure using one drug in maximum dose
b)you cannot achieve correct blood pressure using two drugs in maximum dose
c)you cannot achieve correct blood pressure using 2 drugs in maximum doses including
diuretic
d) you cannot achieve correct blood pressure using any 3 drugs in maximum doses
e) you cannot achieve correct blood pressure using any 3 drugs in maximum doses including
diuretics
- E

SVC + JUGULAR

The shortness of breath,lightheadedness, facial edema with venous distention in the


neck and distended veins in the upper chest with a lip, mouth and finger cyanosis
may be a consequence of
a)stenosis of carotid artery
b)severe obstructive pulmonary disease
c)obstruction of the superior vena cava
d)right ventricle heart failure
e) left supraclavicular lymph node enlargement
c

Enlarged neck veins may indicate what pathological condition?


a) Tumor of cavernous sinus
b) Pulmonary hypertension
c) Infer...ior vena caval obstruction
d) Tension pneumothorax
e) b and d
- E

Elevated jugular pressure without visible pulsation, peripheral edema and


hepatomegaly can be sign of
a)mediastinal tumor
b)pulmonary embolism
c)decompensated heart failure
d)aortic aneurysm
e)b and c are correct
a

Hepatojugular reflux is a sign of:


a. Increased venous pressure when applied pressure on the abdomen.
b. Decreased venous pressure when applied pressure on the abdomen.
c. Decreased atrial pressure when applied pressure on the abdomen.
d. Increased atrial pressure when applied pressure on the abdomen.
- A

2011
Seminar group 1 questions
What signs and symptoms are NOT characteristic for gastrointestinal hemorrhage?
a) epigastic pain while fastingX
b) nausea
c) occult blood in stool
d) “ground coffee” vomits e) maelena

2. Which of the following conditions do not produce a symptom of “wet” cough`?


a. COPD
b. Tuberculosis
c. Atelectasis
d. Tobacco abuse

ANSWER: e) Peripheral edema

9)Name the option preferred for palpation of the apex beat:


a) Left lateral recumbent position
b) Sitting position
c) Supine position
d) Answers A & B
e) None of the above

ANSWER: d) A&B

Seminar group 3 questions


5)In case of unbalanced respiratory acidosis arterial blood analysis show:
a) increased PH, decreased CO2, increased HCO3
b) decreased PH, increased CO2, increased HCO3-correct
c9 decreased PH, decreased CO2, inc HCO3
d) decreased PH, INcreased CO2, decreased HCO3

Seminar group 4 questions

3. kaiser fischer ring is characteristic for


a. Addisons
b. Wilsons correct
c. Horners
d. corneal arcus

a. Horner’s Syndrome is characterized by:


a. Endopthalmos
b. Miosis
c. Ptosis
d. Hyperhidrosis
e. Only a, b, and c*

4. Pancoast tumor can cause the following


a. Superior vena cava syndrome
b. Atelectasis
c. Horner's syndrome correct
d. Enlarged virchow's node

5. Unilateral exophthalmos is found in following


a. Ortibal tumor
b. Aneurysm
c.Orbital hematoma
d. All of the above correct

6. Which malignancy is the most common in the virchow's node


a. Lung
b. GI Tract correct
c. Lung and GI tract
d. Glioblastoma

7. Which one is not a sign of Horner's syndrome


a. Endopthalmus
b. Anhydrosis
c. Miosis
d. Exophthalmos correct

8. Not a sign of dyspne is


a. Nasal flaring
b. Retraction of ribs
c. Use of accessory muscle
d. Decubitus position correct
2006
3. According to Adult Treatment Panel III (ATP III) the components of the metabolic
syndrome are:
a) central obesity, high triglycerides, low HDL-cholesterol, hypertension and fasting
hyperglycemia.
b) central obesity, high triglycerides, high LDL-cholesterol, hypertension and fasting
hyperglycemia.
c) central obesity, total-cholesterol, hypertension and fasting hyperglycemia.
d) central obesity, low triglycerides, low HDL-cholesterol, hypertension and fasting
hyperglycemia.
e) triglycerides, HDL-cholesterol, hypertension and fasting hyperglycemia.

8. A murmur heard during neck auscultation may be caused by:


1) toxic goiter
2) narrowing of carotid artery
3) mitral valve stenosis
4) aortic valve insufficiency
a) 1, 2, 3 b) 2, 3, 4 c) 3, 4 d) 1, 2, 4 e)1, 3, 4
systolic murmur radiates to NECK and carotid aa.
1: toxic goiter causes a systolic murmur.
2. carotid bruit can be detected on neck auscultation indicating carotid artery stenosis
unsure about 3 and 4 bc both cause diastolic murmurs and usually don’t radiate to neck. A)
would be correct if 3 was mitral regurgitation and D) would be correct if 4 was aortic
stenosis.
9. Blood pressure measurement should be performed after 5 minutes rest BECAUSE blood
pressure may fall after standing up.
a) first and second statement is correct, and there is a logical link between them
b) first and second statement is correct, and there is no logical link between them
c) first statement is true, the second statement is false
d) first statement is false, the second statement is true
e) both statements are false

20. Increase of blood urea nitrogen (BUN) may be caused by:


a) Liver disease (decreases BUN)
b) Myocardial infarction
c) Chronic renal failure
d) Low protein diet (decreases BUN)
e) None of the above are true

Questions from classes:

1. Increased bowel sounds are found in:


A. Gastroenteritis
B. Hunger
C. Peritonitis DECREASED
D. Paralytic Ileus DECREASED
E. Diarrhea
Correct answer is:
a) A,B,C
b) A,B,D
c) B,C,E
d) A,B,E
e) A,B,C,E

5. Which parameter is best for monitoring renal function


a) BUN
b) GFR
c) Plasma creatinine concentration
d) Creatinine clearance
e) All mentioned above

17) Colicky pain:


A. Comes from obstruction of the bowel
B. Comes from obstruction of ureters
C. Comes and goes in waves
D. Is not related to peristaltic movement

Correct answer is:


a) A,B,D
b) A,C,D
c) A,B
d) A,D
e) A,B,C,D
COLICKY PAIN: biliary, renal, intestinal colic

18) Osmotic diarrhea is characterized by:


A. Large volume of stools
B. It persist with fasting
C. May be due to lactose intolerance (=lactase deficiency)
D. It disappear with fasting

Correct answer is:


a) A,B,C
b) A,C,D
c) A,B
d) A,D
e) A,C

19) Melena is
a) Vomiting blood
b) Passage of jet black stool
c) Passage of bright red blood per rectum
d) Erythrocytes in urine
e) None of above

Hyperventilation can be provoked by:


1)fever
2.metabolic acidosis
3.deviation of nasal septum (causes HYPOventilation)
4.severe anemia
5.hypothyroidism (HYPERthyroidism causes hyperventilation)

Correct answer is:


a)1, 2
b)1, 2, 3
c)1, 2, 4
d)2
e)each

High location of lower lungs border is related to:


1.meteorism (tympanites)
2.diaphragmatic nerve palsy
3.pleural tumors
4.pleural adhesion

Correct answer is:


a)3, 4
b)2, 4
c)1, 2, 4
d)each
e)1, 2, 3

Correct answer is:


a)1, 2, 3
b)3, 4
c)1, 3, 4
d)each
e)none

Normal movement of lower lung border is approximately:


a)2 cm
b)4 cm
c)6-8 cm
d)12 cm
e)20 cm

Fluid in pleural space can occur in:


1.congestive heart failure
2.lung neoplasm
3.pleurisy

Correct answer is:


a)1
b)1, 2
c)3
d)2, 3
e)each

Inspiratory dyspnea doesn’t occur in case:


a)pulmonary fibrosis
b)subglottic stenosis
d)passive lung congestion
e)foreign body in trachea
f)asthma (expiratory dyspnea)

Inspiratory dyspnea doesn’t occur in case:


a)pulmonary fibrosis
b)pleural adhesions
c)pleural effusion
d)bronchial asthma EXPIRATORY
e)congestive heart failure

Apart from respiratory and cardiovascular system diseases cause of shortness of breath can
be:
1.anemia
2.obesity
3.uremia
4.emotion
5.diabetes

Correct answer is:


a)1, 3, 4
b)2, 3, 4
c)1, 2, 4
d)each
e)none

Atelectasis is – choose incorrect answer:


a)is condition where alveoli have no gas, and their walls are collapsed
b)is condition where alveoli have no gas, and their walls are not collapsed
c)sometimes is due to internal bronchial obstruction
D)can be caused by external compression
e)affected area is airless

In case of a large volume of fluid in pleural cavity we recognize:


a)dull percussion tone and increased tactile fremitus
b)dull percussion tone and decreased tactile fremitus
c)tympanic percussion tone and increased tactile fremitus
d)resonant percussion tone and decreased tactile fremitus
e)none of above mentioned
Which description related to decreased tactile fremitus is incorrect:
a)occurs when there is fluid between lung and chest wall TRUE
b)occurs above lung without air, but with open it’s bronchus
c) occurs above lung without air, but with closed it’s bronchus
d) occurs in obese people TRUE
e)occurs when there is air between lung and chest wall TRUE

Unilateral retraction of chest wall occurs in:


1. pneumonia
2. pneumothorax
3. pleural effusion
4. pleural adhesion

Correct answer is:


1, 3
2, 4
2, 3
each
none

Dull percussion tone is:


a)soft, long, high
b)soft, short, low,
c)occurs when there is fluid in pleural cavity
d)occurs above organs containing air
e)soft, long, low

Seminar 4

Cause of hyperventilation can be:


a)stroke
b)metabolic acidosis
c)hyperthyroidism
d)each
e)b, c
Breathe in:
a)is passive
b)diaphragm descends during contraction
c)chest volume decreases
d)each
e)a, c

Percussion tone in right midclavicular line in second intercostal space in right-handed person
is louder, BECAUSE OF on the left side is muted (duller) due to large vessels.

Upper borders of lungs – apex area – choose incorrect answer:


a)we percuss supraclavicular and suprascapular regions to compare percussion tones
b)medial border goes from sternoclavicular joint to spinous process Th1
c)dull percussion tone is normal tone for this area
d)resonant tone narrowing or disappearance in this area testify to pathology
e)lateral border is about 3-5cm distant from medial border

Correct lower left lung borders (midclavicular line, midaxillar line, vertebral line) are:
a)VI rib, VIII rib, X rib, XI thoracic vertebra
b)IV r, VIII r, XI r, XII r
c)IV r, VIII r, X r, XI r
d)IV r, VI r, VIII r, X r
e)no correct answer

Breath in is an active process, BECAUSE OF requires diaphragm and intercostal muscles


activity.

Right-handed people have loudest percussion tone in left mid-clavicular line in second
intercostal space, BECAUSE OF their right pectoral muscle is better developed than left.

Cough can occur due to alveoli irritation, BECAUSE OF coughing is a reflex action started
by mucosa stimulation.

Normal active diaphragmatic excursion (distance between lower lung border position during
forced maximal inspiration and expiration) is approximately:
a)2 cm
b)4 cm
c)6-8 cm
d)12 cm
e)20 cm

Normal passive diaphragmatic excursion (distance between lower lung border position
during horizontal and vertical posture) is approximately:
a)2 cm
b)4 cm
c)6-8 cm
d)12 cm
e)20 cm

Large pleural effusion may be cause of atelectasis, BECAUSE OF in this condition air
pressure in lungs is bigger than outside the chest and leads to alveoli collapse.

Choose incorrect answer related to cough:


a)is result of throat, larynx, trachea, bronchi irritation
b)selective alveoli irritation doesn’t leads to coughing
c)irritation of pleura, stomach can lead to coughing
d)elderly people have bigger sensitivity of respiratory mucosa to irritation
e)brain irritation cal lead to coughing

In atelectasis due to bronchus occlusion there is lack of air in lung connected with this
bronchus, BECAUSE OF air diffuses to blood.

Which answer related to dyspnea is incorrect:


a)orthopnoe is dyspnea, where patient has to keep vertical position with chest stabilization
(to use accessory inspiratory muscles) to breathe
b)brain tumor can cause dyspnea
c)paroxysmal dyspnea in bronchial asthma is inspiratory dyspnea
d)dyspnea occurs when carbon dioxide plasma level rises, oxygen plasma level falls or there
is acidosis
e)dyspnea in metabolic disturbances is due to acidosis

In chest examination:
a)during percussion the loudest tone is in second left intercostal space, so called basic tone
b)lowering of lower longs border can occur in emphysema, pulmonary edema, meteorism
c)increased tactile fremitus occurs above lung without air
d)dull percussion tone is soft, long, low
e)kyphosis is forward curvature of the spine

Correct is:
a)1, 3
b)2, 4
c)1, 2
d)1, 2, 3, 4
e)1, 2, 3, 4, 5

Unilateral chest retraction occurs in:


a)pneumonia
b)pneumothorax
c)pleural effusion
d)pleural adhesion
e)each

Decreasing of tactile fremitus, choose incorrect:


a)occurs when there is foreign body between lung and chest
b)occurs over lung without air, but with open bronchus
c)occurs over lung without air, but with open bronchus
d)occurs in fat people
e)can occur physiologically in women population

Cause of expiratory dyspnea is:


a)lungs fibrosis
b)pleural adhesion
c)pleural effusion
d)bronchial asthma
e)myocardial infarct

The most frequent sign of respiratory tract diseases is:


a)dyspnea
b)fever
c)pain in chest
d)cough
e)hemoptysis

1.Most common reason of syncope


a)unknown
b)vasovagal
c)orthostatic
d)bradycardia
e)none of the above

2.Stokes-Adamas attack are a type/cause of:


a)Myocardial infarction
b)Examination maneuver
c)syncope
d)Breathing disorder
e)neurological disorder

4.Lordosis,kyphosis and scoliosis concerns


a)posture
b)neurologic disorders
c)speech
d)Vision
e)Hearing

5.A person with pyknic body type has/is


a)muscular,narrow pelvis,slim legs
b)big head,short limbs,large amount of subcutaneous tissue
c)narrow shoulders,long limbs,long thorax
d)square face,slim legs,strong
e)none of the above

6.Dwarfism may be caused by


a)rickets
b)kidney disease
c)hypothyroidism
d)hypogonadism
e)all of the above

9.Normal BMI values range from


a)16-25
b)18.5-24.9
c)25-29.9
d)20.9-35.6
e)18-26

10.Gait problems include


a)Spastic hemiparesis
b)Ataxia
c)Tabetic
d)Spastic diplegia
e)all of the above

11.When checking a mole one should think about


a)asymmetry
b)border
c)diameter
d)color
e)all of the above

12.Tophi are structures related with


a)Gout
b)Athritis
c)heart failure
d)COPD
e)Diabetes

15.Endophtalmus is common for


a)Grave's disease
b)Horner's syndrome
c)Diabetes Mellitus
d)Heart Failure
e)Parkinson's disease

16.Graefe,Kocher,Moebius and Stelwag's signs concernes the


a)ears
b)eyes
c)heart
d)brain
e)limbs
18.following is not true for dyspnea
a) difficult and laboured breathing with shortness of breath
b)it is a symptom
c)it is a sign
d)it is related with Heart failure
e) involves changes in partial pressures of oxygen and carbon dioxide

19.Dyspnea at rest is typical for which degree


a)second
b)fourth
c)fifth
d)third
e)first

20.Dyspnea increasing when patient lies down is called


a)paroxysmal nocturnal dyspnea
b) orthopnea
c)platypnea
d)tachypnea
e)bradypnea

21.sudden onset of shortness of breath after a period of sleep


a)paroxysmal nocturnal dyspnea
b)orthopnea
c)platypnea
d)tachypnea
e)bradypnea

22. Strong hypercarbic drive is seen in


a)"pink puffers"
b)"blue bloaters"
c)"red smoakers"
d)Answers a&b
e)Answers c & a
23.Tachypnea involves more than____ breaths per minute
a)25
b)12
c)30
d)20
e)none of the above

24.Bradypnea involves less than____ breaths per minute


a)10
b)13
c)20
d)12
e)25

25.incresed rate and depth of breathing may be caused by


a) acidosis
b)aspirin poisoning
c)alkalosis
d) Answers a & b
e)answers b & c

26.Decreased rate and depth of breathing may be caused by


a)acidosis
b)myasthenia gravis
c)alkalosis
d)Answers a & b
e)Answers b& c

27. Another name for non-productive cough is


a)dry
b)pleural
c)COPD
d)sputum
e)none of the above

30.vesical breathing sound ?


a) Can be heard over most of healthy lung area
b) Expiration is shorter than inspiration
c) No pause between inspiration and expiration
d) Diminished in emphysema, pneumothorax and hydrothorax
e) All are true

31.During your physical examination you observed diminished lung expansion, percussed
dullness over intercostals spaces 8-10 on the right side, vesicular sounds where absent over
this area and tactile fremitus over this area was diminished. What would you suspect?
A) An underlying tumor
B) Accumulation of f...luid in pleural space*
C) Heart failure
D) May be pneumothorax
E) Nothing this is normal

32. Q- Indicate the false answer concerning palpation?


a) Can be used to assess tenderness
b) you should ask if patient feel any pain even before you start palpation
c) cannot be used to assess abnormalities*
d) include tactile fremitus
e) false answers are b and d

36.Increased amplitude of the apical impulse may indicate:


A. Hyperthyroidism
B. severe anemia
C. Mitral regurgitation
D. A+B are correct
E. All answers are correct

39. Choose correct statement(s)


:a). In fibrosis, residual volume decreases*
b). In emphysema, alveolar walls are destroyed, leading to decrease in total lung capacity
(TLC)
c) In chronic bronchitis but not in emphysema, there is an increase in residual volume
d) all are true
e) a and c are true

46. Alarming locations of lymph nodes are:


a) Supraclavicular
b) Axilarry
c) Supratrochlear
d) Popliteal
e) All are correct

51.When can you observe curschmanns spirals?


a. Bronchiectasis
b. Tuberculosis
c. COPD *
d. Bronchial asthma
e. All statements are correct

53 Indicate the correct statement(s) about blue bloater


a. Intense dyspnea and obese
b. Cyanotic and little sputum
c. May develop cor pulmonale*
d. Strong hypercarbic drive
e. All statements are correct

58. Indicate false statement:


a) hyperresonant sound – asthma
b) flat tone – pneumonia
c) dull – pleural effusion
d) tympanic – gastric bubble
e) resonant – lung fibrosis*
63. Signs of leftsided heartfailure is all except:
a)pulmonary edema
b)cerebral symptoms
c)dyspnea
d)nocturia
e) edema *

64. Skintemperature will rise in case of:


a)inflammation
b)shock
c)hyperthyrodism
d) a and c are correct*
e) all are correct

65. Hirsutism:
a) is another name for anemia
b) is lack of male hair
c) is male pattern och hair in female *
d) means that you have an bacterial infection
e) is loss of feeling in hands and toes

69. During physical examination of a patient, you


note that he has a high fever with shaking chills (rapid onset), cough, and
dyspnea. You also note dullness on
percussion, bronchial breath sounds, increased fremitus, and the presence of
crackles.

70.What can you generalize regarding his condition?


a) Patient most probably h...as bacterial pneumonia
b) Hemoptysis might be a sign of this disease
c) Central cyanosis might be present
d) a and b are true
e) all of the above are true*

1. Horner’s Syndrome is characterized by:


a. Endopthalmos
b. Miosis
c. Ptosis
d. Hyperhidrosis
e. Only a, b, and c*

1. Dalrymple’s Sign is:


a. Rare blinking
b. Miosis
c. Scared eyes
d. May be seen in thyrotoxicosis
e. Only c and d are correct*
f.

1. Dalrymple’s Sign is:


a. Rare blinking
b. Miosis
c. Scared eyes
d. May be seen in thyrotoxicosis
e. Only c and d are correct*
f.

1. Melena (black, tarry stool) can occur in:


a. Peptic ulcer disease
b. Mallory-Weiss syndrome
c. Rectal varices
d. Only a and b are correct*
e. All of the above

1. An infant born with decreased thyroid function will be at risk for:


a. Tachycardia
b. Fine muscle tremors
c. Impaired mental development*
d. Hyperhidrosis
e. Increased metabolic rate

1. What is the main discriminator between primary and secondary hyperaldosteronism?


a. Plasma aldosterone level
b. Plasma sodium level
c. Plasma renin level*
d. Plasma potassium level
e. A and C are correct

a.

1.

ANOTHER SET:

1. What is haematemesis?
a. Fresh Blood in stool
b. Old blood in stool
c. Vomiting blood
d. Blood in urine
e. All answers are correct
1. Which of the following is/ are not malabsorption syndromes signs?
a. Diarrhea
b. Weight loss
c. Edema
d. Anemia
e. Acromegaly *

f.

1. What causes decreased or absent bowel sounds ?


a. Gastroenteritis
b. Diarrhea
c. Hunger
d. Paralytic ileus *
e. Mechanical ileus

f.

f.

f.

f.

f.

1. You have a female patient that complains of: dull, recurrent pain that is radiating to back. Pain is
increased after meal and decreases with vomiting. What does this patient suffer from?
a. Duodenal ulcer with perforation
b. Duodenal ulcer with no perforation
c. Gastric ulcer with perforation *
d. Gastric ulcer with no perforation
e. Gastric carcinoma

f.

f.
f.

1. Reversible airway obstruction is distinctive symptom of:


1. Pneumothorax
2. Pneumonia
3. Asthma X
4. Chronic Obstructive Pulmonary Disease
5. Bronchiectases

3. Orthopnea is described as:


1. Dyspnea on exertion
2. Dyspnea caused by bronchospasm
3. Dyspnea that begins or increases in the lying position X
4. Nocturnal paroxysmal dyspnea
5. Dyspnea that arises at upright position

6. Hemoptysis often accompanies following disorders except for:


1. Pulmonary embolism
2. Lung tuberculosis
3. Lung cancer
4. Asthma X
5. Mitral stenosis

8. True cyanosis may result from the following, except for:


1. Hemoglobin with low affinity to oxygen
2. Pulmonary arterio-venous fistulas
3. High altitude
4. Arterial obstruction X
5. Hypoventilation syndromes
11. Jaundice is a:
1. A viral liver disease spreading with food and water
2. A yellowish colour of skin, mucous membrane and sclera accompanying
hyperbilirubinemia X
3. A synonym for bile duct obstruction
4. Intolerance of tropical fruit juices
5. None of the above

14. Identify true statement regarding carotid palpation:


1. carotid sinus massage can cause an exaggerated decrease in sinus rhythm X
2. carotid sinus massage can cause an exaggerated increase in sinus rhythm
3. both sides should always be palpated simultaneously
4. should be performed solely in patients with valvular disease
5. none of the above

15. Show the definition of paradoxic pulse:


1. the pulse is characterized by alternation of pulsation of small amplitude with the pulse
wave of large amplitude, with the regular rhythm
2. the pulse wave is large and bounding, not easily obliterated by the examining fingers
3. the pulse is characterized by an exaggerated decrease (>10mmHg) in the amplitude of
pulsation during inspiration and increased amplitude
during expiration X
4. the pulse is characterized by a large amplitude, rapid rise, narrow summit and sudden
descent
5. the pulse has two main peaks

16. Dull percussion sound over pulmonary area may be present in:
1. pleural effusion X
2. pulmonary emphysema
3. pneumothorax
4. A and B are true
5. A,B and C are true.

17. Decreased respiratory sounds are typical for:


1. pulmonary emphysema
2. obesity
3. cachexia
4. A and B are true X
5. A,B and C are true
22. “Miosis” is a term used for:
1. uneven pupils
2. wide pupils
3. narrow pupils X
4. irregular pupils
5. none of the above is true

23. A typical sign of the 7th cranial nerve paresis is:


1. face asymmetry X
2. miosis
3. exophthalmus
4. none of the above is true
5. facial oedema

27. Cor pulmonale may be caused by:


1. Chronic obstructive pulmonary disease (COPD) X
2. Aortic valve insufficiency
3. Systemic arterial hypertension
4. A and B are true.
5. None of the above is true

1. Reversible airway obstruction is distinctive symptom of:


1. Pneumothorax
2. Pneumonia
3. Asthma X
4. Chronic Obstructive Pulmonary Disease
5. Bronchiectases

3. Orthopnea is described as:


1. Dyspnea on exertion
2. Dyspnea caused by bronchospasm
3. Dyspnea that begins or increases in the lying position X
4. Nocturnal paroxysmal dyspnea
5. Dyspnea that arises at upright position

6. Hemoptysis often accompanies following disorders except for:


1. Pulmonary embolism
2. Lung tuberculosis
3. Lung cancer
4. Asthma X
5. Mitral stenosis

8. True cyanosis may result from the following, except for:


1. Hemoglobin with low affinity to oxygen
2. Pulmonary arterio-venous fistulas
3. High altitude
4. Arterial obstruction X
5. Hypoventilation syndromes

11. Jaundice is a:
1. A viral liver disease spreading with food and water
2. A yellowish colour of skin, mucous membrane and sclera accompanying
hyperbilirubinemia X
3. A synonym for bile duct obstruction
4. Intolerance of tropical fruit juices
5. None of the above

14. Identify true statement regarding carotid palpation:


1. carotid sinus massage can cause an exaggerated decrease in sinus rhythm X
2. carotid sinus massage can cause an exaggerated increase in sinus rhythm
3. both sides should always be palpated simultaneously
4. should be performed solely in patients with valvular disease
5. none of the above

15. Show the definition of paradoxic pulse:


1. the pulse is characterized by alternation of pulsation of small amplitude with the pulse
wave of large amplitude, with the regular rhythm
2. the pulse wave is large and bounding, not easily obliterated by the examining fingers
3. the pulse is characterized by an exaggerated decrease (>10mmHg) in the amplitude of
pulsation during inspiration and increased amplitude
during expiration X
4. the pulse is characterized by a large amplitude, rapid rise, narrow summit and sudden
descent
5. the pulse has two main peaks

16. Dull percussion sound over pulmonary area may be present in:
1. pleural effusion X
2. pulmonary emphysema
3. pneumothorax
4. A and B are true
5. A,B and C are true.

17. Decreased respiratory sounds are typical for:


1. pulmonary emphysema
2. obesity
3. cachexia
4. A and B are true X
5. A,B and C are true

19. Lung cancer risk factors – show the correct answer:


1. cigarette smoking is the most important risk factor of lung cancer X
2. the risk of lung cancer does not increase with dose of tobacco
3. the risk is greater in persons who begin smoking in older age.
4. Only A and B true
5. A,B,C true

22. “Miosis” is a term used for:


1. uneven pupils
2. wide pupils
3. narrow pupils X
4. irregular pupils
5. none of the above is true

23. A typical sign of the 7th cranial nerve paresis is:


1. face asymmetry X
2. miosis
3. exophthalmus
4. none of the above is true
5. facial oedema
27. Cor pulmonale may be caused by:
1. Chronic obstructive pulmonary disease (COPD) X
2. Aortic valve insufficiency
3. Systemic arterial hypertension
4. A and B are true.
5. None of the above is true

2. A 42 years old male presents to the emergency room with sharp epigastric pain, nausea,
vomiting and anorexia. BP 108/60, HR 124/’, T 38,2°C. What is differrential diagnosis?
I) ruptured aortic aneurysm
II) myocardial infarction
III) renal colic
IV) pancreatitis a) I,III
b) I, II, III
c) II, IV
d) only IV
e) all of the above X

11. Causes of the spleen enlargement are:


a) haematologic diseases (leukemia, lymphoma)
b) portal hypertension c) infectious diseases d) infiltrative diseases
e) all of the above X

12. The possible causes of hyperglycemia are:


a) diabetes type 1 and 2 b) chronic pancreatitis
c) steroid usage d) acromegaly
e) all of the above X

15. Rebound tenderness over the abdominal area may be present in:
a) acute pancreatitis
b) acute cholecystitis
c) rupture of the peptic ulcer
d) infection of the ovary
e) all of the above X

17. What are the predisposing factors of gall stone formation:


1) obesity 2)contraceptive pill usage 3) female sex 4) male sex
5) low body mass 6) alcohol abuse 7) paracetamol usage

a) 1,4, 6, 7 b) 1, 2, 7
c) 1, 2, 3 X
d) 5, 6, 7 e) 4, 5, 6

20. What are the features of metabolic syndrome:


a) hypercholesterolemia, obesity, diabetes
b) hypertension, central obesity, hypertrigycerydemia, low HDL levels, hyperglycemia
X
c) hyperglycemia, low HDL, high LDL, obesity d) hypertension, diabetes, low HDL, high LDL
e) high total cholesterol, low HDL, high TG, obesity

5) Indicate the sins and symptoms of “acute abdomen”:


a. Chronic pain in RUQ, jaundice, vomiting
b. Strong abdominal pain with rebound tenderness, involuntary abdominal muscles defense,
changed bowel sounds characteristic X
c. Epigastric pain, radiating to the back, relieved by meals and antiacids d. Pain the lower
regions of abdomen with dysuria, and fever
e. Constipation, flatulence with occult blood in stools

7) In what case would you suspect the gastrointestinal bleeding?


a. A patient presents with abdominal, severe pain in RUQ, jaundice and light stools b. A
patient presents with pain and rebound tenderness in lower right quadrant,
anorexia, constipation, fever
c. A patient presents with weakness, dizziness, tachycardia, low blood pressure, cramping
abdominal pain, hematemesis, melena X
d. A patient presents with diffuse abdominal pain with decrease in bowel sounds and
constipation
e. None of the above

11) Typical place of auscultation for pulmonary artery valve is:


a. IInd intercostal space on the left border of sternum X
b. IInd intercostal space on the right border of sternum
c. Vth intercostal space medialy to the left midclavicular line d. IVth intercostal space on
the right border of the sternum
e. IV intercostal space on the left border of the sternum

15) Dull sound during chest percussion may be a sing of:


a. pleural effusion X
b. bronchitis
c. pneumothorax d. emphysema
e. sarcoidosis

16) During chest auscultation which sound is NOT adventitious?


a. Wheeze b. Ronchi c. Stridor
d. vesicular sound X
e. crepitations
17) Which of the following are the clinical features of chronic bronchitis:
a. history of smoking b. sputum
c. chronic cough d. A and B true
e. A,B,C true X

19) Disability to raise eyebrows, close eyelids, smile widely is a feature seen in:
a. paresis of cervical autonomic plexus b. paresis of the IIIrd cranial nerve
c. paresis of the VIIth cranial nerve X
d. Horner’s syndrome e. hypothyroidims

20) Indicate the characteristic features of asthma:


a. Wheeze, chronic episodic dyspnea, chronic cough, reversible airway obstruction X
b. Dyspnea, productive cough, not reversible airway obstruction
c. Acute onset, severe dyspnea with accompanying pain in the chest, cyanosis
d. Chronic productive cough, especially in the morning, dyspnea at exertion, hemoptysis e.
None of the above

23) Hirsutism is the term used to describe:


a. Hair loss at the top of head
b. Increased hair growth in females in typically men’s places as breast, abdominal midline,
face, pubic and thigh area X
c. Changes in feeling of smooth surface like in diabetic neuropathy d. Disturbances in
fertility
e. None of the above

25) Orthopnoe is a term used to describe:


a. Need to go to bathroom during the night
b. Waking up at night with the feeling of shortness of breath
c. Dyspnea on lying flat X
d. Alternate episodes of hyperventilation and apnea
e. During heart auscultation: lack of pulse wave while heart beats are audible

29) Indicate the proper order of thorax physical examination:


1. auscultation
2. percussion
3. palpation
4. observation
a. 1, 2, 3, 4
b.4,3,2,1 X
c. 2,1, 4, 3
d. 1, 3, 4, 2
e. 4, 2, 1, 3

SOME RANDOM NOTES FROM A TEST


study blood, diseases with blood, anemias, ecg

1.nephrotic syndrome - protein uria at leaset 3.5/day

2. ansocoria picture

3. lung picture lower quadrant has tectile frematus. what does that mean? lobar pneumonia?
pneumothorax? atelectasis? Pleural effusion? all the above?

4. jaundice cause

6. normal bmi

7. 70F mumur @ apex and @ 2nd right intercostal space. A fib b/c of Mitral stenosis?

8. ascultation of pulmonary valve @ left intercostal space?

10. chronic cough = false for copd??


11. Ekg = RBBB? had no wide complexes and 1 + 3 werent inverted
13. Woman with bleeding decreased vwF

14. hypertension = criteria all of the above?

17. prolonged QT,

19. hypocalcemia = facial twitch

20. man has flank pain = renal colic

22. decreaesd bowel sounds = ileus

23. patient with increased bowel sounds and movements, constipation, vomiting soon after
eating

25. acute pain

28. anuria characterized by 24hrs/less than 100ml?


29. wheezing: called wheezus maximus? occurs due to stridor
30. angina = helped after nitroglycerin? stabbing pain?

32. normal acommodation: mydriasis and myosis

34. woman with pain upon upper border what diagnostic test do u use? xray?

36. question with thyrotoxicosis

37. goetsches disease

38. Parathyroidism = because of what?

40. paradoxial pulse

43. renal artery stenosis is heard thru ascultation of abdomen? case question

45. light colored stools?


46. woman comes in with voice changes, bad teeth etc. > acrogmegaly

47. absent bowel sounds? paralytic ileus

48. acute dm1? signs = ketoacidosis?


49. right side of face is not working. Lesion in Cn 5,7,horners syndrome,all the above?

50. cor pulmonale found when?

51. Copd is found when?

52. how long to check the HbA1C level in diabetics

53. most important clinical thing to do (like take history and physical exam, or blood count, or
urinalysis).

54. Signs of upper GI tract bleeding.

55. EKG = Rbbb? or something else, narrow qrs, bunny ears in v2,v3,v4 no hypertrophy/
axis deviation

56. CHF symptoms

58. man has one arm 180 bp other arm 100 bp what is the problem?

59. diverticultis

60. reasons for kidney failure outside kidney. uti?

61. GN - like nephrotic/phritic syndrome?

63. bone loss? PTH? parathryoidism

64. what is a neoplastic infection? cancer cells in body cause infection? or cancer cells
cause infection
66. normocytic>hypochromic>normocytic>microcytic

67. Vit. K

68. T/F questions patient has asthma breathing difficult b/c lungs are overinflated patient
with asthma breathing out thru smaller opening both true?

69. which of the following is not a vital sign? bp, breathing, pulse, temperature are the 4
main vitals

70. person abdominal pain, constipation, diarrhea etc what is the disease?

99. The most common malignant neoplasm of the GI tract is:


(a)Esophageal cancer
(b)Colorectal cancer*
(c)Pancreatic cancer
(d)Gastric cancer
(e)None of them are common

100. What is the correct order of physical examination of the lungs?


(a)Auscultation, percussion, palpation, inspection
(b)Inspection, palpation, auscultation, percussion
(c)Auscultation, percussion, inspection, palpation
(d)Inspection, auscultation, palpation, percussion
(e) Inspection, palpation, percussion, auscultation*

- plural effusion?

- DM1

1. Consequences of Hypertension (ie: renal failure)

2. What to look for when checking sinus rhythm.


3. Abdominal exam method. From first - last
5. Difference between UC and CD

6. The different types of Pulses

7. Symptoms of CHF

8. When you hear tactile fremitus (increased/diminished)

9. Nephrotic/Nephritic syndrome

10.No dullness over liver = GI perforation.

miosis,
anisocoria,
symptoms of acromegaly,
the value of anuria,
how long to check the
HbA1C level in diabetics,
all the heart stuff (mitral stenosis, regurgitation etc.),

2018

Dullness on percussion is present in the case of


1.pneumonia
2.pneumothorax
3.fluid in the pleural cavity
4.physiologically over heart
5. physiologically over lungs
correct answers are
a)1, 3,5
b)2, 3, 4
c), 2, 4, 5
d)1, 2, 4
e), 1, 3, 4

21. A 35 year old visits the local emergency department due to acute dyspnea. He had
episodes in the past, however no diagnostics were done. No history of chronic illness. He
smoke… cigarettes (15 pack years). Opin auscultation: diminished vesicular sounds and
bilateral…..Normal chest x-ray. The most reasonable diagnostic test is
a)serum copper level
b)pulsoximeter
c)chest CT scan
d)CRP level
e)spirometry
COPD
- history of smoking
- decreased vesicular sounds
- dyspnea
- will show abnormal spirometry

The causes of transudate in the pleural cavity include


1)circulatory insufficiency
2)tuberculosis EXUDATE
3)pneumonia and pleura EXUDATE
4)nephrotic syndrome
a)1, 2, 3, 4,
b)only 3
c) 1, 2, 3
d)only 1
e)1, and 4
e

(I) A patient with symptoms of hemorrhagic diathesis in the form of haemorrhages to


muscles and joints, as well as prolonged bleeding time after extraction of teeth, occurring
several hour after surgery, requires assessment of plasma coagulation factors, because (II)
on the basis of the clinical picture we suspect thrombophilia
a)sentence I and II are true, there is a relation between them
b) I and II are true, no correlation
c)I false, II true
e) I and II are false
C?

During physical examination of a patient with pneumothorax, all symptoms might be present
EXCEPT
a)signs of dyspnea
b)decreased breathing sounds
c)increased vocal fremitus (DECREASED vocal fremitus)
d)hyper resonant sound upon percussion
e)deviation of trachea
c
(I) vomiting and diarrhea are early symptoms of chronic renal failure, because (II) a small (i.e
about 10%) reduction in creatinine clearance results in serum electrolyte abnormalities and
an increase in serum urea and creatinine
a)both parts are true, there is a causative relationship between them
b)both parts are true, there is no correlation
c)part I is true, part II is false
d)part I is false, part II is true
e)both parts are false
e?

Intermittent claudication
a)typically pain is present in the lower part of the lower extremities
b)it is a symptoms connected with ischemia, causing pain of the lower extremities while
walking
c)it is the most common symptom of the peripheral arteries disease
d)pain is relieved upon rest
e)all the answers are correct
e

(I) renal artery stenosis may cause hypertension, because (II) the most common cause of
renal artery stenosis in atherosclerosis is atherosclerosis
a)sentence I and II are true, correlated
b) I and II true, not correlated
c) I is true, II is false
d) I is false, II is true
e) I and II are false
b?

Exudate in pleural cavity might be caused by


a)nephrotic syndrome
b)hypoalbuminemia
c)heart failure
e)pneumonia and pleuritis
e?
Bronchial obturation appears in
1)asthma
2)pneumonia
3)pulmonary tuberculosis
4)chronic obstructive pulmonary disease
a)all answers are correct
b)1, 4
c)2, 3
d)1, 2
e)1, 3

Patient complains about having exercise induced dyspnea that increases for years. He also
claims to have productive cough with expectoration of sputum (especially in the morning)
patient used to be an active smoker. Which tests should you perform to confirm the most
probable cause of the above symptoms
a)plethysmography
b)corona rography
c)chest x ray
d)bronchofiberscope
e)spirometry
e?

Laboratory tests assessing kidney function are all of the following except
a)prothrombin time
b)arterial blood gas test
c)creatinine
d)electrolytes
e)BUN
b?
23 year old male comes to an ER. Patient is conscious but presents dyspnea. He reports
that 1 hour before he noticed chest pain that occurs while he was watching TV. Since that he
suffers from escalating dyspnea. He does not have any chronic diseases and refuses to take
any medications. Upon physical examination patient presents cyanosis, tachypnea, BP
150%90 mmHg, HR 120/min, upon auscultation of the lungs vesicular sound is absent on
the right side. Upon percussion you find resonant sound on the left side and tympanic on the
right side. Most probable diagnosis
a)fracture of the ribs
b)spontaneous pneumothorax
c)myocardial infarction
d)aortic dissection
e)pleuritis
b?

Indicate possible reasons of fever


a)CMV infection
b)lupus erythematosus
c)Hodgkin lymphoma
d)tuberculosis
e)all above mentioned answers are correct
e?

Upon physical examination you diagnosed patient having symptoms of the Horner’s
syndrome. Which of the additional examination you will perform in this patient as first choice
procedure
a)head CT scan
b)gastroscopy
c)thorax CT scan
d)colonoscopy
e)visual field examination
c?

In a case of acute diarrhea regardless the etiology, correct proceedings is are


a)patient's hydration per os or intravascular to prevent dehydration
b)stopping the defecation with some medication, optimal up to 3 hours from the beginning of
symptoms
c)administering high caloric nutrition to prevent cachexia
d)transferring the patient to a specialist facility
e)all the above are correct
a?

The test which unequivocally confirms pulmonary tuberculosis is


a)typical changes on chest x ray
b)TB skin test
c)sputum culture positive for mycobacterium tuberculosis
d)a and b
e) a b c
c?

In which condition d we observe an elevation of ammonia levels in the blood


a)uremic coma
b)hypoglycemic coma
c)ketonic coma
d)hyperosmotisk coma
e)hepatic coma
e?

What is the most probable diagnosis when you encounter: loss of consciousness, seizures,
tachycardia, increased sweating, normal body temperature
a)uremic coma
b)hypercalcemia crisis
c)ketonic coma
d)hypoglycemic coma
e)thyrotoxic crisis
d?

Choose correct statements


1)hepatojugular reflux is a sign of right sided heart failure
2)pardee wave is a ST elevation in an acute phase of myocardial infarction
3)creatinine kinase concentration is the most sensitive marker of myocardial necrosis
4)hyperkalemia causes ST segment depression in ECG
1)1, 2, 3
b)2, 3, 4
c)2, 4
d)1, 2,
e) all above;
- 1: well congestive HF can cause hepatojugular reflux sign.
- 2: Yes, pardee waves show ST elevation in response to mycardial infarction.
- 3: ..I thought that would be troponon
- 4: Yes, hyperkalemia causes ST segment depression.
Microbes that cause atypical pneumonia are
a)mycoplasma pneumoniae, legionella pneumophila, chlamydia pneumoniae
b)haemophilus influenzae, bordetella pertussis, candida species
c)E.coli, streptococcus pneumoniae, Staph. A
d)mycoplasma pneumoniae, type B flu, staph A
e)type A flu, haemophilus influenzae, moraxella catarrhalis
a?

What do we observe in uncompensated respiratory acidosis


1)elevated pH
2)decreased pH
3)elevated pCO2
4)decreased pCO2
5)elevated HCO3-
6)decreased HCO3-
7)elevated base excess
8)decreased base excess
a)2, 3, 6, 8
b)2, 4, 6, 8
c)2, 3, 5, 7
d)2, 4, 6, 7
e)2, 4, 6, 8
c?

The cause of prerenal acute renal failure include


1)ureteral cancer
2)acute left ventricular failure in the course of infarction
3)anaphylactic shock
4)obstruction of the urethra
5)hemorrhage
a)1, 3 5
b)1, 2, 4
c)2, 3, 5
d)3, 4, 5
e)2, 3, 4
c?

Your primary recommendation for a patient with chronic obstructive pulmonary disease
should be
a)high doses of vitamin C
b)weight loss
c)chronic treatment with antibiotics
d)discontinuation of smoking
e)high protein diet
d?

On physical examination in a case of purulent meningit you might find


a)neck stiffness
b)headache
c)fever
d)nausea and vomiting
e)all of the above
e?

Upon physical examination of the patient’s chest you find decreased vocal fremitus on left
side, dull sound upon percussion on left side and decreased vesicular sound on left side.
What could be the reason and which test would you perform in first place?
a)right sided pneumonia + chest CT scan
b)right side emphysema + chest X ray
c)left side penumothorach + chest x ray
d)left side hydrothorax + chest CT scan
e)left side hydrothorax + chest X ray
e?

2014
What are not possible causes of acute chest discomfort
a)hepatomegaly
b)aortic dissection
c)peptic ulcer
d)peripheral arterial disease
e)pulmonary embolism
1)a, d
2)b, c,
3)b, e
4)a, c, d
5)a, c

Polycythaemia (erythrocytosis) may be due to - indicate false


a)chronic lung disease
b)polycythaemia vera
c)renal carcinoma
d)chronic kidney failure
e)dehydration

Indicate FALSE statement about the possible causes of high eosinophils count
a) hay fever
b) parasitic infestations
c) bronchial asthma
d) bacterial infection
e) eosinophilic leukemia

Prerenal renal failure may be due to the severe hemorrhage because of possible renal
function impairment caused by urinary flow obstruction in a patient with nephrolithiasis
a)the first and the second is true, correlation
b) first and second statement true, no correlation
c)first is true, second is false
d)first is false, second is true
e)first and second are false
b?

Peripheral facial palsy is characterized by - indicate the correct answer


a)central and peripheral facial nerve palsy cannot be distinguished on physical examination
b)inability to close eyes tightly and whistle with saved function of the forehead muscles
c)inability to feel the touch on forehead, cheeks and chin and under the orbits, under lower
lip
e)inability to rise eyebrows, close eyes tightly and whistle
e?

A patient complaining of productive cough over 3 months, during a year, exertional dyspnea
with wheezes, periodical exacerbation of the symptoms with fever, who is a heavy smoker
over many years may suffer from
a)bronchial asthma
b)congestive heart failure
c)paroxysmal atrial fibrillation
d)COPD
e)lobar pneumonia

Sinus bradycardia is diagnosed when the heart rate is


a)between 60-100 bpm, with irregular RR intervals
b)between 60-100 bpm and the pacemaker is in the sinus node
c)less than 60 bpm with the pacemaker in the av node
d)between 40 and 60 bpm with the pacemaker in HIS purkinje fibers
e)less than 60 beats per minute and the pacemaker is in the sinus node
e?

Lymph nodes examination consists o nodes


a)size
b)tenderness
c)clusters
d)movability
5) all above

e?

On physical examination you see more than 5 spider angiomas on a female chest, indicate
the investigation helpful in differential diagnosis
a)ECG, bnp levels, creatinine, myocardial necrosis markers
b)bilirubin level, INR and clotting status of the patient, full blood count, protein and albumin
levels, abdominal ultrasound
c)abdominal ultrasound, bilirubin, GGT and alkaline phosphatase levels
d)abdominal CT, amylase levels in urine and serum , stool general examination
e)gastroscopy helicobacter pylori test, pH metry of the esophagus
b?
Soft eye balls may be a finding on PE of patient with
a)dehydration
b)hyperosmotic state
c)acute glomerulonephritis
d) a and b are true
e) a b and c are true
d?

Skin yellow discoloration together with stool discoloration and dark urine can result from
1.gallstones
2.pelvic inflammatory disease
3)tumor of the Oddi’s sphincter
4)Gilbert’s syndrome
5)hepatitis
a)1, 4
b)1, 2, 4,
c)1, 3,
d)1, 4, 5,
e)only 1
c?

INdicate possible reasons for gross hematuria


1)glomerulonephritis
2)urinary tract infections
3)kidney stones
4)urinary bladder tumors
5)hemophilia A
a)2, 3, 5
b)2, 3, 4
c)1, 3, 5
d)1, 2, 4
e)2, 3, 4, 5
e?

The amount of air expelled when one takes maximally deep breath and then forcefully
exhales maximally is called
a)forced expiratory volume in 1 sec (FEV1)
b)forced vital capacity
c)vital capacity
d)tidal volume
e)total lung capacity
b?

Wjat diseases should not be in differential diagnosis when nocturia occurs? choose false
a)renal artery stenosis
b)obstructive sleep apnea
c)congestive heart failyre
d)diabetes mellitus
e)urinary tract infection
a?

In patient presenting with intense flank pain extending to groin and genitals, accompanied by
fever and hematuria the best investigation to set the diagnosis is
1)abdominal CT
2)creatinine and sodium plasma levels, chest X ray
3)abdominal ultrasound, full blood count and creatinine levels
4)abdominal x ray, stool examination
5)mikroalbumineamia assessement, abdominal examination

28. Lower extremity ischemia usually manifests as… chose the false
1)leg pains racially aggravated by exertion
b)calf knee that might occur in Humans (with the flexed knee, forcibly dorsiflexes the ankle )
maneuver is performed
c) barely palpable or absent peripheral lower extremity arteries pulsation
4)intermittent claudication 5. decreased temperature and pallor of lower sweating
f)

2017

The following signs and symptoms of chronic kidney disease EXCEPT FOR
a)weakness, fatigue
b)acidosis, breathing (kussmaul)
c)pale, dry skin and brownish color
d)uremic frost
e)goldflam sign
e?
Symptomatic bacteriuria must always be treated, because it leads to chronic
complication….. treatment reduced the occurrence of symptomatic UTI
a)true,true, with a relationship
b)true, true , without relationship
c)true,false
d) false, )true
e)false, false

Lowering of inferior lung border might be present in


a)pleural effusion
b)phrenic nerve palsy
c)emphysema
d)pleural effusions
e)correct a and c
c?

Pain in the upper abdomen may occur


a)food poisoning
b)duodenal ulcer disease
c)myocardial infarction
d)correct a and b
e)correct a b and c
e?

Anisacaria edema is not typical in


a)heart failure
b)liver cirrhosis
c)nephrotic syndrome
d)femoral vein thrombosis
e)malnutrition
D?

Which is useful indicator of kidney function


a)....
b)urea nitrogen level
c)serum creatinine level
d)estimated gfr
e)24 hour urine collection
d?

Asthma and copd are … by


a)chest CT
b)chest x-ray in the AP projection
c)angiography
d)spirometry
e)pulse oximetry
D?
tis
a?

Weight loss might be present in


a)neoplasm
b)chronic pancreatitis
c)uncontrolled type 1 diabetes
d)correct a and b
e)all correct
e?

In which pathological states fever is one of the major symptoms


1.tumor of a kidney
2.pneumonia
3.lymphoma
4.infectious endocarditis
5.lupus erythematosus
a)all are correct
b)2, 3, 4
c)2, 4, 5
d)only 2
e)correct, 2, …..
a?
In the course of glomerulonephritis we may identify
a)elevated titers of anti streptolysin O
b)proteinuria
c)erythrocyte
d)correct a and b
e)all above are correct
e?

Neurological symptom most common..


a)folic acid deficiency
b)vitamin b12 define icny
c)iron (?) deficiency
d) a and c
e) b and c
b?

Symptoms of chronic kidney failure?

What is not a characteristic for neoplastic lymph node


a)tender
b)non removable
c)...1 cm in diameter
d)found in …
e)a and c

Hematuria can be caused by


a)tuberculosis
b)neoplastic process in urinary bladder
c)thrombocytopenia
d)a and c
e)all correct

Moebius sign is positive when..?

Cockermouth and gaudi formula can be used to


a)calculate creatinine clearance
b)calculate … in urine
c)calculate pulse oximetry
d)calculate FEV/FVC index
e)none of the above
a?

Exudate in pleural cavity can be caused by


a)pneumonia
b)nephrotic syndrome
c)ovarian cancer
d) a and b are correct
e) .. and c are correct
e?

Upon physical examination you find decreased vocal fremitus, dull sound upon percussion
and decreased vesicular sound upon .. of right lung. What do you suspect?
a)pneumonia
b)pneumothroac
c)elevated diaphragm
d)hydrothorax
e)... and d are correct
e?

Find the true statement


a)gynecomastia can occur in both women and men
b)gynecomastia affects only boys and men over 15 years of age
c)gynecomastia is a growth of tissue other than glandular tissue
d)gynecomastia occurs in boys and men, regardless of age
e)gynecomastia affects only one breast
d?

Acute glomerulonephritis is most commonly caused by infection with


a)staph a MRSA
b)group Ab-hemolytic streptococci
c)E.coli
d)pseudomonas
e)adenovirus
b?
Patient reports dyspnea and hemoptysis. Upon physical examination you find cyanosis,
increased breathing effort, correct vesicular sound
a)pulmonary embolism
b)exacerbation of COPD
c)pneumoniae
d)asthma
e)server anemia
c?

Increased waist circumflex might be caused by


a)heart failure
b)liver cirrhosis
c)ovarian cancer
d) a and b
e)a b and c
E?

(I) central cyanosis is characterized by excessive deoxygenation of tissues because (II) can
be seen on lips, bottom, oral cavity and tongue
a)both true, connected
b)both true, no connection
c)true, false
d)false, true
e)false, false

GERD symptoms?
a)headache
b)Dysphagia (?)
c)excessive sweating
d)weight loss
e)cough
(flera svar är rätt)

In chronic kidney disease for the C3(?) category the GFR is? (can barely see)
To which of the following does the gold classification apply
a)COPD
b)asthma
c)cystic fibrosis
d)H pylori infection
e)tuberculosis
a?

2018 resit

Orthostatic hypotension

Intermittent claudication is a symptom of the condition that affects peripheral arteries and
produces pain in the leg muscles (I). occurs after few hour when the person is in standing
position and stops shortly after beginning of exercise (II)
a)true true, correlated
b)true, true, no connection
c)true, false
d)false, true
e)false, false
c/e?

Indicate the characteristic features of COPD


a)wheezes
b)ronchi
c)pleural friction rub
d)alla bove
e) a and b
e?

Indicate the best investigation to estimate glomerular filtration rate


a)urine pH
b)24 hours protein excretion rate
c)plasma urea level
d)plasma erythropoietin level
e)creatinine clearance
e?

Bolt pattern respiration can be present in


a)metabolic acidosis
b)obesity
c)increased intracranial pressure
d)metabolic alkalosis
e)all correct
c?

Name the pupil abnormality on the picture.


a)miosis
b)mydriasis
c)argyll robertson pupil
d)anisocoria
e)healthy eyes

30 year old patient complains of chronic cough (lasting for 3 months) hemoptysis, 37C
temperature in random measurements, 5kg weight loss during last month, fatigue, which
tests will you order for the beginning
1.lung spirometry
2.microbiologic sputum analysis
3.chest X-ray
4. spirometry
5.tuberculosis skin test
6. lungbiopsi
a)1, 3, 4
b)1, 4, 6
c)2, 3, 5
d)1, 3, 6
e)2, 5, 6
c?

2016
Rapidly progressive renal failure with hematuria and proteinuria with increased blood
pressure and vasculitis type changes in the skin would indicate
a)interstitial renal disorder
b)urinary tract infection
c)glomerulonephritis
d)nephrolithiasis
e)diabetes insipidus
c?

The image below corresponds to…

a)left sided hydrothorax


b)right sided hydrothorax
c)situs inversus
d)right sided bronchiectasis
e)bilateral pneumonia
b?

Upon chest examination you find: diminished tactile fremitus over the right lung, lung borders
upon percussion - right scapular line 6th - intercostal space and 10th left scapular line,
diminished diaphragmatic excursion over the right and decreased vesicular sound over the
base of the right lung. The finding might correspond with
a)pulmonary edema
b)right sided exudative pleurisy
c)right sided bronchiectasis
d)b and c are correct
e)all answers are correct
b?

Alternating pulse indicate the correct statement


a)pulse with two clear peaks within one heart cycle
b)irregular pulse
c)pulse which amplitude differs from beat to beat
d)might be present in heart failure or atrial fibrillation
e) c and d are correct
e?

Which of the following findings are seen in a chronic renal failure stages IV/V
1)macrocytic anemia
2)normocytic anemia
3)hypocalcemia
4)azotemia
5)hyperphosphatemia
6)decreased parathormon level
7)hyperkalemia
a)1,3, 4, 5, 6, 7
b)2, 3, 4, 5, 7
c)2, 3, 5, 6
d) 1, 6
e)1, 7
b?

36 year old male was brought to ER with increasing dyspnea and low blood pressure after
falling from a ladder. Upon physical examination you find a wound in right anterior axillary
line, diminished vesicular sound and tactile fremitus on that side. What is the most probable
preliminary diagnosis
a)right side IInd rib fracture
b)right side pneumonia
c)left ventricle heart failure
d)liver cirrhosis
e)right side tension pneumothorax
e?

Which of the cranial nerve pathology would present as altered sensation of touch on one of
the sides of the face
a)accessory
b)glossopharyngeal
c)optic
d)trigeminal
e)facial
d?

Referring to the precious question what kind of procedure would you order to confirm the
diagnosis
a)carcinoembryonic antigen in blood
b)barium meal examination
c)CT of the abdomen
d)colonoscopy with biopsy
e)fecal occult blood test
d?

Which statement related to the increased tactile fremitus is true


a)may be caused by long consolidation with no bronchial obstruction
b)may be caused by obstructive bronchial secretion
c)may be caused by excess air in the lungs
d)may be caused by pleural effusion
e) a and d are true
a?

The rate and depth of breathing will increase with


a)metabolic acidosis
b)anxiety
c)pain
d)hypoxemia
e)all of above
e? - a? eller d? får mer oxygen inn da?, hmm jaknskej e idk
trodde pain var mindre hmm
hmm ja sant, de andre enn a er vel ikke deeper..eller vet ikke, anxiety er jo veldig overfladisk
breathing
For i hypoxia så vil det jo kreve mer krefter og oksygen å bruke accessory resp muscles, så
den blir mer shallow

A 35 year old male complains of substernal chest pain aggravated by inspiration and
relieved by sitting up. Lung fields are clear on auscultation and heart sounds are regular but
quite. Chest x ray shows a markedly enlarged cardiac silhouette. Indicate the true answer
about other elements of physical examination
a)there might be a blood pressure difference between upper and lower extremities
b)might be a blood pressure decrease during inspiration
c)there might ba a pulse deficiency on the peripheral arteries
d)the patient may present with a bruit over the carotid arteries
e)none of the above findings is expected in this patient
e?

A 68 year old woman who has had a previous hysterectomy presents with an 8 hour history
of cramping periumbilical pain. Each episode of pain lasts 3-5 minutes and then …? Over
several hours she develops nausea,vomiting and abdominal distension. She has been
unable to pass stool or flatus for the past 4 hours. What is the diagnosis?
a)acute hepatitis
b)acute pancreatitis
c)acute pyelonephritis
d)intestinal obstruction
e)irritable bowel syndrome
d?

A 32 year old woman presents with dysuria, urinary frequency and urinary urgency for 3
days. Urinalysis shows erythrocytes, leukocytes, and nitrate in her urine, without casts.
Indicate the possible preliminary diagnosis
a)pyelonephritis
b)nephrolithiasis
c)hydronephrosis
d)urinary tract infection
e)none of the above
d?

A 39 year old alcoholic presents with massive hematemesis and hypotension. Examination
reveals hemorrhoids and ascites. What is the most likely diagnosis explaining his condition
a)gastric ulcer
b)aortoenteric fistula
c)esophageal varices
d)colon polyp
e)adenocarcinoma of the colon
c?
A 31 year old man who works physically presents to the ER due to a chest pain. He does not
smoke and jogs 3 days a week. His father died of a heart attack in his 60es. The patient
developed chest pain gradually during the day that worsens with activity and resolve at rest.
His HR is 68 bpm, BP 120/70 mmHg, and RR 14. On examination lungs are clear and there
is no cardiac murmur. You palpate tenderness over the sternal border and the 3rd and 4th
rib. An ECG and chest X Ray shows normal. Indicate the possible preliminary diagnosis
a)costochondritis, intercostal neuralgia
b)myocardial infarction
c)pleurisy
d)acute ulcerative disease
e)GERD
a?

On a general examination a 70 year old patient has a mask like face, rigid movements and
walks small steps. When resting marked tremor of his arms, hands and head is observed. A
probable reason is
a)hypothyroidism
b)hemiparesis
c)aortic regurgitation
d)gout
e)parkinson’s disease
e?

A car accident victim, with head trauma, is brought to the ER. He moans and read with eye
paining and limb extension for pain. Indicate which scale do you use for assessing this
patient state
a)new york heart association scale
b)glasgow coma scale
c)canadian cardiology society scale
d)apache scale
e)none of the above
b?
Chvostek sign is a sign of
a)hypercalcemia
b)hypocalcemia
c)hypophosphatemia
d)hypercortisolemia
e)hyperuricemia
b?

A 36 year old woman complains of frequent headaches accompanied by abdominal pain,


nausea, weakness and palpitations. She had hypertension diagnosed a few months ago. A
mass located retroperitoneally in the upper abdominal cavity is seen on abdominal CT. What
is the most likely diagnosis
a)teratoma
b)pheochromocytoma
c)kidney cancer
d)ovayr cyst
e)acute pancreatitis and pancreatic cyst
b?

Which of the following pathologies is least likely to displace the cardiac apex
a)left ventricular dilation
b)left ventricular hypertrophy
c) right ventricular dilation
e)emphysema
d?

A 28 year old man comåöaints of coughing up small amounts of blood over the past 2-3
weeks and low grade intermittent fevers for the past 2 months, night sweats. He smoked for
3 years but stopped 2-3 years ago. He reports weight loss of 5-7 kg that does not attribute to
diet. His laboratory data are unremarkable with the exception of leukopenia and positive HIV
test. WHat is his most likely diagnosis?
a)bronchitis
b)bronchiectasis
c)lung cancer
d)tuberculosis
e)esophageal varices
c?
A 43 year old woman complains of fatigue and night sweats for 2 months. On physical
examination there is nontender lymphadenopathy including supraclavicular, epitrochlear ad
scalene nodes. Full blood count and chemistry studies including liver enzymes are normal.
Chest x-ray shows hilar lymphadenopathy. Which of the following is the best next stein in
evaluation
a)excisional lymph node biopsy
b)gastroscopy
c)toxoplasmosis IgG serology
d)CT of the neck
e)nose and throat culture
a?

A 50 year old woman comes to your office with a history of 2 episodes of coughing blood.
The quantity is 2-3 tablespoons of bright blood on each occasion. She complains of mild
shortness of breath, but she feel comfortable at rest. She has never smoked, denies weight
loss or fever and has no other medical problems. What would be the next step in her
management
a)referral to a pulmonologist for bronchoscopy
b)return to your clinic in the next several weeks to monitor the symptoms
c)chest x ray with an admission to the hospital for additional testing
d)complete vital signs and physical examination
e)PET scan of the whole body
d?

Lordosis, kyphosis and scoliosis are terms describing


a)posture
b)neurologic disorders
c)speech
d)vision
e)hearing
a?
Intermittent claudication us a symptom of the condition that affects peripheral arteries
(peripheral artery disease) and produces pain in the leg muscles (II) occurs after few hours
when the person is in standing position and stops shortly after beginning of exercises
a)true, true, connected
b)true, true, no connection
c)true, false
d)false, true
e)false, false
c?

diminished or delayed chest wall movement, narrowed intercostal spaces on affected side,
tachypnea, diminished tactile fremitus , apical impulse and trachea deviated to the affected
are- this is the best description of…
a)bronchiectasis
b)asthma
d)pleural effusion
d)atelectasis
e)COPD
d?

Pain provoked by tapping/pushing over the costovertebral margin, restlessness of the patient
who also complains of nausea and fever may suggest
a)prostatitis
b)renal artery occlusion
c)pyelonephritis
d)acute pancreatitis
e)none of the above
c?
LUNGS
Cough/Hemoptysis/Sputum

28.the following terms are related to sputum except for


a)albuminoid
b)rusty
c)prune juice
d)cranberry juice
e)moss-agate
A?
Moss-agate is green

Very sticky, dense sputum is in:


a)mucoviscidosis
b)bronchial asthma
c)dehydration
d)each
e)none
D

Brownish sputum is characteristic for:


a)pulmonary edema
b)lung tumor
c)gastric ulcer
d)tuberculosis
e)lobar pneumonia
E
Brownish sputum -> Pneumococcal infetion. Pneumonic inflammation causes lysis of RBC
hence the colour

Choose incorrect answer related to cough:


1.sudden, usually involuntary, expulsion of air from the lungs
2.onset of cough can be related to foreign body aspiration
3.can’t be as a symptom of respiratory tract obturation
4.is airway protective reflex
5.paroxysmal cough in asthma is connected with hematemesis

Correct is:
a)3
b)2, 3
c)1, 3, 5
d)5
e)2, 5
A

Choose incorrect answer:


a)reddish sputum can occur in hysterical people
b)brownish sputum is characteristic for pneumonia, pink for pulmonary edema
c)in case of gastric bleeding contrary to pulmonary bleeding sputum is light red mixed with
mucus or pus
d)pink (raspberry) is typical for lung tumors
e)purulent sputum is characteristic for bacterial infections
- A?

Reddish-brown sputum is characteristic for:


a)acute pulmonary edema
b)bronchial asthma
c)lung cancer
d)pneumococcal pneumonia
e)chronic bronchitis
D
Blood-tiged sputum is characteristic for:
a)bronchial asthma
b)bronchitis
c)pneumonia
d)acute pulmonary edema
e)tuberculosis
D
OBS: notice that book says acute pulmonary edema has clear/watery sputum but this picture
is from lecure so let’s go for D

Dry cough is characteristic for:


a)pleurisy
b)pulmonary infarction
c)left ventricle failure
d)foreign body aspiration
e)correct is answer a, b, c

7)Combine correct alternatives:


1. Raspberry sputum a) tumor
2. brown sputum b) haemophilus influenzae
3. green sputum c) pneumococcal infection

correct: 1-a, 2-c, 3-b

8)Which of the following is the most common cause of blood in the sputum:
a) Tuberculosis
b) Sublingual varicosities
c) Congestive heart failure
d) Pneumonia
e) Purulent discharge

ANSWER: b) sublingual varicosities

10) What is hemoptysis


a) vomiting blood
b) coughing up blood
c) bloody stool - coagulated
d) bloody stool – fresh
e) none of the above
B

Choose the most unlikely cause of hemoptysis:


a)tuberculosis
b)lung cancer
c)coagulation defects
d)bronchial adenoma
e)mediastinal tumor
E -> the expectoriation of blood/blood streaked sputum from pharynx, larynx,
trachea, bronchi or lungs

Hemoptysis doesn’t occur in:


a)pulmonary infarct
b)tuberculosis
c)thoracic aortic aneurysm
d)bronchiectasis
e)mitral stenosis
Can occur in all of them…
The most common causes of hemoptysis are:
a)bronchiectasis, lung cancer, tuberculosis
b)pulmonary infarction, viral pleurisy
c)Rendu-Olser’s disease
d)pulmonary infarction, mitral stenosis
e)acute inflammation of upper respiratory tract, chronic
bronchitis
A
But attention all of the alternative can cause hemoptysis

Cause of cough can be:


1.sinusitis
2.pharyngitis
3.bronchial asthma
4.pneumonia
ALL CORRECT

Cause of hemoptysis can be:


1.infections
2.tumors
3.chest trauma
4.heart defects
5.coagulation abnormalities

Correct is:
a)1, 2, 3, 5
b)2, 3, 5
c)2, 3, 4, 5
d)each
e)none
D

Lung cancer
A 50 year old lady with chronic voice hoarseness should be suspected of
a)cancer of larynx
b)arch of the aorta aneurysm
c)retrosternal goiter
d) a and b
e) a b and c
e
Other causes include esophageal cancer, hypo/hyperthyroidism, lung cancer, mitral stenosis

2. Small cell lung cancer is characteristic for


a. smokers
b. adenocarcinoma
c. Squamous cell carcinoma
d. In small airways
A

59.... Patient suffers from recurrent pneumonia, always occurring in the same place with
streaks of blood in sputum. Most likely diagnosis is:
a) tuberculosis
b) emphysema
c) pulmonary embolism
d) lung cancer
e) left ventricular heart failure
D

10. Lung cancer wrong statement


a. Tends to spread very early
b. 90 - 95 % arise from epithelial lining of small airways
c. Strongly related to smokers
d. Small cell lung cancer is the most common type
D
Adenocarcinoma is the most common type

44. The most aggressive type of lung cancer, with extremly bad prognosis is?
a) Small cell lung cancer
b) Adenocarcinoma
c) Squamous cell carcinoma
d) Large cell carcinoma
e) None of the above
A

19. Lung cancer risk factors – show the correct answer:


1. cigarette smoking is the most important risk factor of lung cancer
2. the risk of lung cancer does not increase with dose of tobacco
3. the risk is greater in persons who begin smoking in older age.
4. Only A and B true
5. A,B,C true
A

Abnormal breathing

Irregular interspersed periods of apnea in a disorganized sequence of breath are


characteristic
a)biot pattern of breathing
b)kaussmaul pattern of breathing
c)cheyne stokes pattern of breathing
d)tachypnea
e)hyperventilation
a
1. Kind of breathing that can occur during acidosis
a. Biot
b. Kussmaul
c. Cheyne stokes
d. No change in breathing
B
Biot = damage in medulla
Cheyne Stoke = CHF, damage to basal ganglia/thalamus

Kussmaul breathing doesn’t occur in:


a)diabetic ketoacidosis, diabetic coma
b)lobar pneumonia
c)methanol intoxication
d)uremia
e)can occur in each situation
B
All the other options can cause metabolic acidosis -> Kussmaul breathing. Lobar
pneumonia would rather cause dyspnea

Kussmaul breathing:
a)slow and deep breathing
b)fast and shallow breathing
c)occurs in diabetic coma
d)is due to hydrogen ions deficiency followed by respiratory center stimulation
e)decrease of breaths rate, shallow
C (more specifically diabetic ketoacidosis. Fast and deep breathing)

Kussmaul respiration occurs in:


a)pneumonia
b)bronchial asthma
c)diabetic coma
d)emphysema
e)bronchiectases
C

Orthopnoe is:
a)physiological dyspnea due to physical exercise
b)posture abnormality with flat feet
c)dyspnea due to RDS in past
d)dyspnea, where patient has to keep vertical position with chest stabilization (to use
accessory inspiratory muscles) to breathe
e)none is correct
D

Breathing pattern characteristic for neurotic patient:


a)fast, shallow
b)slow
c)Kussmaul
d)sighing
e)Cheyne-Stokes
D/A?

Orthopnoea is:
a)dyspnea due to physical exercise
b)ventilation abnormality due to mechanical problems
c)dyspnea due to cardiovascular problems
d)kind of dyspnea where respiration can be performed only in an erect posture, with hands
lean against stable object
e)dyspnea due brain tumors and inflammations
D (orthopnea = dyspnea when lying flat)

Irregular respiration pattern with unstable amplitude, volume, frequency of breathing


is:
a)Kussmaul’s respiration
b)Biot’s respiration
c)Chelmonsky’s respiration
d)Cheyne-Stoke’s respiration
e)Chwostek’s respiration
B

Choose incorrect answer related to dyspnea:


a)exertive dyyspnea may occur in cardiovascular disorders
b)in metabolic disturbances can occur Kussmaul’s respiration
c)onset of cardiac asthma is usually around midday and has renal origin
d)acute and chronic laryngitis may occur with expiratory dyspnea
e)the most frequent type of dyspnea is mixed (exipratory and inspiratory) dyspnea
- C

Condition, where breathing is possible only in vertical posture is called:


a)polypnoe
b)orthopnoe
c)thepopnoe
d)dyspnoe
e)hyperpnoe
- B

Choose incorrect answer:


a)dyspnea of cerebral origin occurs in encephalitis
b)Kussmaul’s respiration, cyclic slower and shallower and later faster and deeper
respiration, occurs in metabolic acidosis
c)orthopnoe can occur in bronchial asthma
d) in expiratory dyspnea are active expiratory accessory muscles, for example
sternocleidomastoid muscle
e)emotional dyspnea is related to some functional (not organic) cardiovascular abnormalities
Which answer related to Kussmaul’s respiration is correct:
a)deeper and faster breathing
b)characteristic for metabolic abnormalities, especially acidosis
c)occurs in diabetic coma, uremia, methanol poisoning
d)each
e)b, c
All are correct

Kussmaul respiration pattern:


a)slower and deeper respiration
b)faster and shallower respiration
c)occurs in diabetic coma and uremia
d)s related to acute cardiac arrest
e)slower and shallower respiration
C (fast and deep)

Irregular breathing consisting of gradual acceleration and deepening of breaths and


then slowing down and becoming shallow with the moments of apnea, we call
a)biot’s breathing
b)bradypnoe
c)hypopnea
d)cheyne stokes breathing
e) none from the above
d

Choose correct answer:


a)dyspnea in bronchial asthma usually occurs by night
b)in bronchial asthma is more difficult to inhale than to exhale air
c)Biot’s respiration (deep and fast breathing) occurs in uremia
d)in expiratory dyspnea there is expiratory accessory muscles involvement, for example
sternocleidomastoid muscle
e)barrel chest occurs in Marfan’s syndrome
- A

Cheyne-Stokes respiration:
1.is an effect of respiratory center disturbance
2. irregular breathing with intervals of apnea followed by slow and shallow and later fast and
3. deep sequence of respiration
4. periodic fast and shallow respiration with 10-30 seconds apnea intervals
5. can occur in brain tumors and poisoning
Correct answer is:
a)1
b)1, 2
c)3
d)1, 2, 4
e)1, 3
B

Cheyne-Stokes respiration:
a)is related to faster and deeper breathing
b)can be physiological in children
c)is due to respiratory center impairment in medulla oblongata
d)is cyclic pattern, where firstly is slow and shallow respiration and later faster and deeper
e)which gradually disappear
f)correct is c, d
F
Can be seen in children with immature respiratory systems, it is not physiologic

Expiratory dyspnea
a)is characteristic for bronchial asthma
b)is only due to lung flexibility decreasing
c)is related to bronchoconstriction, what leads to aerodynamic drag (resistance) increasing
d)none is correct
e)correct is a, c
- E

3.Types of respiration do not include:


a)Tachypnea
b)Cheyne-Stokes
c)Ataxic
d)Kussmaul
e)opisthotonus
E
Choose incorrect answer:
a)Cheyne-Stokes’ respiration can be physiological in elderly during sleep
b)inspiratory dyspnea may occur in chronic laryngitis, lung tumors, emphysema, tetanus
c)in some conditions pain in chest may be related to abdominal pathological process (i.e.
pain spread from stomach)
d)raspberry sputum suggests lung tumor occurrence
e)Kussmaul’s respiration can be caused by methanol intoxication

57. Cheyne-Stokes pattern of breathing may be characteristic for:


a) sleep apnea
b) ketoacidosis
c) metabolic alkalosis
d) metabolic acidosis
e) lung cancer
A

Pneumonia
3. The main cause of Community-acquired
pneumonia is/are:
a. E. coli
b. Streptococcus pneumoniae
c. Gram-positive bacteria
d. Answer b & c are correct
B

85 year old demented woman was admitted to hospital due to dehydration and loss of
appetite. After 5 days of hospitalization she developed fever, confusion, dyspnea. Upon
physical examination you hear dull percussion tone over the middle right lobe and rales over
the right lung with some bronchial sounds. You suspect
a)pulmonary embolism
b)bronchitis
c)pneumonia
d)tuberculosis
e)exacerbation of COPD
C
3)In pneumonia sputum could be:
a) bloody or rust coloured
b) green
c) foul-smelling and bad-tasting
d) currant-jelly
e) all are correct
E

6)Mendelson syndrome:
a) Is a opportunistic pneumonia
b) is a form of atelectasis
c) is a form of chemical pneumonia
d) is a form of atypical pneumonia
C
42. Which of the following is the least likely find...ing in a patient with typical
pneumonia?
a) Altered mental status
b) Herpes labialis
c) Slow onset fever (correct)
d) Chest pain
e) Increased fremitus over consolidation
C
Rapid onset of fever. Slow onset and not very high fever is seen in atypical pneumonia

Emphysema

60. MOST characteristic sign of emphysema is:


a) finger clubbing
b) wheezing sounds in parts of lungs
c) wheezing sounds all over lungs
d) pursed-lip breathing
e) hyperresonant sound upon percussion
D

5. Which type of emphysema is/are not associated with airflow obstruction?


a. Centriacinar emphysema
b. Panacinar emphysema
c. Para Septal emphysema
d. b and c
D

6. Symptoms of COPD are a combination of:


a. Emphysema
b. Pneumonia
c. Chronic bronchitis
d. a and c
e. a and b
D

Hyperresonant percussion tone:


1.occurs in emphysema
2.is characteristic for pleural effusion
3.occurs in atelectasis
4.is typical for bronchiectasis

Correct answer is:


a)1
b)2
c)1, 2
d)1, 2, 3
e)each
A
Typical for COPD. Pleural effusion causes dull sound on percussion.

Which physical sign is not typical for emphysema


a)breathing with pursed lips
b)...edge displaced downward
c)tactile fremitus diminished
d)tactile fremitus increased
e)diminished audibility of heart sounds
d
Tactile fremitus is increased in a very inflammed lung, ex in pneumonia

COPD
Which of the signs and symptoms is not typical for copd
a)chronic cough
b)dyspnea
c)hemoptysis
d)chest barrel
e)silent vesicular sounds
c

49. A patient with COPD has onset of dyspnea in:


a) Morning
b) Evening
c) Before midnight
d) Midnight
e) None of the above
A

18. Which sentence about COPD is false:


1. COPD usually takes years to become clinically significant
2. The diagnosis is usually made in middle-aged and older persons
3. Patients with COPD have reversible airway obstruction
4. The course of the disease is punctuated by periodic exacerbations
5. Usually affects smokers
C
One of the differences between COPD and asthma is that COPD has airway obstruction
minimal or not reversible, meanwhile in asthma it is reversible.

What is the most common cause of copd exacerbations


a)rupture of bullae
b)infections
c)increased number of cigarettes smoked per day
d)lung cancer
e)none of the above
b

Who's the wrong answer about asthma and COPD


a)in COPD radiological findings (thoracic x-ray) are usually normal
b)the onset of asthma usually occurs before the age of 20, while copd after 40 yo
c)in copd cough and sputum secretion are chronic mostly permanent symptom
d)in asthma, the lung function is normal in the time between symptomatic periods
e)the effect of using bronchodilators is much better in asthma than i copd
a

Pulmonary embolism

Pleural friction rub may be present in


a)tuberculosis
b)pneumonia
c)emphysema
d)a and b are true
e) a b and c
d? (I woudl go for only B/Elvira according to book/seminars)
Other causes of pleural friction rub are pulmonary embolism (most common) ,
pneumonia, pulmonary vasculitis and pleuritis). It is produced when inflamed parietal and visceral
pleurae move over one another

Chest abnormalities
7. Barrel chest will not be a common finding on physical examination in:
1. Cystic fibrosis
2. Chronic asthma
3. Emphysema
4. Arterial hypertension
5. chronic bronchitis
4

Pes excavatum means:


a)barrel chest
b)gibbus
c)rachitic chest
d)pigeon chest
e)funnel chest
E

29.Barrel chest is associated with?


a) Increased superior-inferior diameter
b) Increased Anterior-posterior diameter
c) Is also called pectus carinatum
d) May accompany aging and COPD
e) Answers b and d are correct
E

Which is not characteristic for barrel chest:


a)inspiratory position of chest
b)hollow of sternum
c)horizontal position of ribs
d)costal angle more than 90 degrees
e)increased anteroposterior diameter
B
Hollow of sternum goes with pectus excavatum/funnel chest

Pectus carinatum means:


a)barrel chest
b)flat chest
c)pectus piriformis
d)pigeon chest
e)funnel chest
D

A 29 year old female has a chest deformity


in which the sternum protrudes from the
thorax. Select the appropriate name for skeletal deformity
a)pectus excavatum
b)kyphosis
c)barrel chest
d)pectus carnitum
e)lordosis
d

Into your office comes a s69 year old male with 50 years history of intense smoking, with
chronic productive cough and extremely low physical exertion tolerance. What are the
fingerings you would expect to find upon physical examination?
a)funnel chest, exacerbated vesicular sound, dehydration
b)barrel chest, diminished vesicular sound, prolonged expiration, lowered lung borders, silen
heart tones, pulsation in epigastrium
c)barrel chest, loud heart tones, increased diaphragmatic excursion
d)barrel chest, prolonged inspiration, bronchial sound, silent heart tones, elevated lung
borders
e)funnel chest, ronchi, increased S1 over aorta, flat percussion sound over whole lungs
b?
Patient for suuuure has COPD
Auscultation
The adventitious sound crackles are
a)discontinuous
b)continuous
c)are heard more often on inspiration
d)caused by disrupted passage of air through bronchial tree
e)all except b are correct
e?

What is the correct order of physical examination of the lungs?


b. Auscultation, percussion, palpation, inspection
c. Inspection, palpation, auscultation, percussion
d. Auscultation, percussion, inspection, palpation
e. Inspection, auscultation, palpation, percussion
f. Inspection, palpation, percussion, auscultation
- F

68. Choose a false statement regarding adventitious breath sounds:


a) Wheezes are caused by rapid air flow through narrowed
or partially obstructed airways
b) Presence of wheezing may suggest asthma, emphysema,
presence of obstructive tumor, or congestive heart failure
c) Rhonchi are characterized by accumulation of
secretio...ns in larger airways, usually in trachea or large bronchi
d) Crackles or rales are often heard in typical pneumonia
e) None of the above statements are false*
- E

61. A collapsing arterial pulse and high pulse pressure are characteristic for which
valvular heart disease?
a) Aortic stenosis
b) Subaortic stenisos
c) Aortic regurgitation*
d) b and c
e) All of them
- C

Pleural friction rub


a)related to rubbing of pleural walls against each other
b)occurs during pleurisy
c)might be associated with pain during cough and deep breathing
d)occurs when there is increased amount of fluid in pleural space
e) a b and c are correct
e
Seen in ex pulmonary embolism, pneumonia, pleuritis, pulmonary vasculitis.

Pulmonary edema
Pulmonary edema may be caused by
1.myocardial infarction
2. cardiac tamponade
3. atrial fibrillation with rapid ventricular rate
4. an acute increase in blood pressure
5. pulmonary embolism
6. tricuspid regurgitation
7. mitral regurgitation with an acute onset
a)1, 2, 4, 6, 7,
b)1, 2, 3, 4, 5, 6, 7
c)1, 4, 5, 7
d)1, 2, 3, 4, 6
e)1, 3, 4, 7
e

Diaphragmatic excursion (lung mobility) may be limited in all of the mentioned states except
a)emphysema
b)pregnancy
c)massive ascites
d)pulmonary edema
e)rib fracture
d

Kidney
Types of urination
Dysuria = pain/discomfort when urinating
Pollakisuria = frequent urination at short intervals without increase in volume. DUe to
cystitis/reduced bladder capacity
Enuresis = involuntary discharge of urine (ex during night)
Aliguria = pain during urination (ex cystitis/urethritis)
Stranguria = slow painful urination (muscular spasm of urethra/bladder)
Polyuria = abnormally large amounts of urine
Oliguria = abnormally small amounts (below 0.5L/day in adult)
Anuria = below 100 ml urine/day
Pyuria = urine containing WBC or pus

12) Which definition is correct for polyuria:


a) release of abnormally large amounts of urine
b) release of abnormally small amounts of urine
c) pain or discomfort when urinating
d) slow, painful urination, caused by muscular spasm of the urethra and bladder
e) none of mentioned above
A

13) Frequent urgency or urination at short intervals without increase in daily volume
of urinary output, due to cystitis or reduced bladder capacity is called:
a) Nycturia
b) Stranguria
c) Pollakisuria
d) Anuria
e) Dysuria
C

Anuria means producing less urine than (OBS THEY MEASURE IN ML/HOUR)
a)100 ml/h
b)60 ml/h
c)650 ml/h
d)300 ml/h
e)5 ml/h
e (Anuria = below 100ml/day. 5x 24 = 120)

Frequent or permanent urination is called


a)stranguria
b)polyria
c)anuria
d)poollakisuria
e)oliguria
d

14) Pyuria is not usually due to:


a) Infectious proces of UT
b) Urinary tuberculosis
c) Drugs nephropathy
d) Fever
e) Glomerular disease
D? (Osäker… men tror D pga fever is not the cause of pyuria, however it can be a
symptom of the ongoing infection/disease?)

Proteinuria may be caused by


1)urinary tract inflammation
2)heart failure
3)physical exercise
4)fever
5)glomerulonephritis
The correct answers are
a)1, 2, 4, 5
b)5
c)2, 3, 5
d)1, 5
e)all are correct
E

Nephrotic syndrome
The most common cause of nephrotic syndrome in adults is
a)diabetic nephropathy, primary glomerulonephritis
b)diabetic nephropathy, secondary glomerulonephritis
c)hypertension nephropathy, primary glomerulonephritis
d)hypertension nephropathy, secondary glomerulonephritis
e)chronic infections, nephropathy caused by drugs
b (antar att de menar on 2nd place it is glomerulonephritis, not that it is a “secondary
cause”,bc it is a primary cause)

2) Indicate the possible nephrotic syndrome causes:


a. Diabetes mellitus
b. Rapidly Progressive Glomerulonephritis
c. Systemic Lupus Erythematosus (SLE)
d. Amyloidosis
e. All of the above
E
Nephrotic syndrome is characterized
by
a)hypoalbuminemia
b)proteinuria > 3.5g/24 hours
c)hematuria
d)oliguria
e) a and b are correct
E
16) Sudden onset of chills, fever, lumbar pain and a positive Goldflam sign are
characteristic for:
a) Prostatitis
b) Acute pyelonephritis
c) Cystitis
d) Urethritis
e) All above
E

Nephritic syndrome
1) Nephritic syndrome is characterized by:
a. Proteinuria, hypertension, hypoalbuminemia,
edema, hyperlipidemia and hyperlipiduria
b. Active urine sediment with massive proteinuria
(>3,5g/d)
c. Hematuria, proteinuria, hypertension, impairment of
renal function
d. Isolated proteinuria and systemic hypertension
e. Pain in the lumbar area, hematuria, bacteriuria
C

Indicate features of the acute nephritic syndrome


a)hypertension, renal function, impairment, hematuria, proteinuria <3.5g/d
b)proteinuria > 3.5 hypoalbumineamia, edema, hyperlipidemia, hyperlipidemia
c)dysuria symptoms, hematuria, fever
d)microalbuminuria, hypertension, polyuria
e)edema, nocturia, dyspnea, hypertension
a

Acute & Chronic renal failure


3) Indicate the possible causes of acute renal failure:
1. Sepsis
2. Acute glomerulonephritis
3. Malignant hypertension
4. Severe hemorrhage
5. Urethra obstruction by a large kidney stone

a. 1, 3, 5
b. 1, 2, 4
c. 3, 4, 5
d. 1, 2, 3
e. All of the above
E
Basically everything can cause ARF. Remember that the causes are prerenal (ex
hemorrhage), intrarenal (ex acute glomerulonephritis) and postrenal (ex urolithiasis)

7. Typical laboratory findings in chronic


renal failure are:
A. Low serum bicarbonate
B. Hypokalcemia
C. Macrocytic anemia
D. Elevated serum urea and creatinine
E. Hypophosphatemia
a) A,B,C,D
b) A,D,E
c) A,B,D
d) A,B,E
e) A,B,C,D,E
C

Which of the following symptoms are connected with


chronic kidney failure
1)anemia
2)hypophosphatemia
3)hyperkalemia
4)metabolic acidosis
5)metabolic alkalosis
6)weakness
7)skin itchiness
8)haematuria and polyuria
a)1, 3, 4, 6, 7
b)1, 3, 4, 5, 7, 8
c)2, 3, 4, 6, 7
d)2, 3, 5, 7, 8
e)1, 4, 5, 6, 7, 8
a

8. Typical laboratory findings in acute


renal failure are:
A. Hyperkalcemia
B. Acidosis
C. Hypernatremia
D. Elevated serum urea and creatinine
E. Hypophosphatemia
a) A,B,C,D
b) A,D,E
c) A,B,D
d) A,B,E
e) A,B,C,D,E
C

Nephrolithiasis
4) Indicate the sings, and symptoms of
acute nephrolithiasis:
a. Acute onset of proteinuria, edema and
abdominal pain
b. Steady decline of renal function with the
history of infectious endocarditis
c. Sudden onset of colicky pain located on the
side, radiating to the groin, hematuria, polyuria,
vomiting
d. Hematuria, proteinuria, fast decline in renal
function
e. Skin itching, nausea, fatigue, anorexia
C
The extreme colicky pain is the most characteristic presentation for nephrolithiasis

Renal stone can cause the following


a)positive goldflam sign
b)renal colic
c)hematuria
d)nausea and emesis
e) all of the above
e

18. Which sentences about nephrolithiasis are true:


1. The peak incidence is in the age group of 20 to 45 years olds
2. Patients with nephrolithiasis usually have hematuria
3. The incidence is higher in women than in men
4. It may be associated with vomiting and ileus
5. Fiber rich diet is a risk factor of nephrolithiasis

a) 1,2,4
b) 3,4,5
c) 5
d)1,3
e)1,2,3,4
a
Males are more frequently affected. Fiber rich diet can actually decrease the risk

Which of these conditions makes the patient produce large amounts of light yellow
urine with a high specific gravity
a)nephrogenic diabetes insipidus
b)nephrocalcinosis
c)uncontrolled diabetes
d)central diabetes insipidus
e)psychogenic polydipsia
C
High specific gravity = very concentrated urine

17. Which statement concerningapidly Progressive Glomerulonephritis is true:


a) heamaturia is a typical finding
b) renal failure develops over weeks to months
c) the main cause is obstructive uropathy
d) A and B are true
e) A ,B and C are true
D
Main causes are related to anti-glomerular basement membrane antibodies (idiopathic,
goodpasture’s syndrome, associated with other primary glomerular diseases)

From the list of symptoms choose those which are most characteristic for the acute
glomerulonephritis
a)proteinuria, erytrocyter, hypertension
b)ascites, hypertension, proteinuria
c)erythrocyte, proteinuria, hypotension
d)proteinuria, venous thrombosis, acute kidney failure
e)acute kidney failure, hypertension, acidosis
a

6. Characteristic symptoms for uremia are:


a) Hypervolemia, hypertension, anuria, ”fluid lungs”, nausea
b) Hypervolemia, hypertension, polyuria, microcytic anemia
c) Hypovolemia, polyuria, hyperkalemia, hyperkalcemia
d) hypervolemia, secondary hyperparathyroidism, polyuria
e) hypovolemia, GI bleeding, anuria, hypertension

1. List 4 causes of acute abdomen


- Hepatitis
- Ectopic pregnancy → colitis
- Pancreatitis
- Acute appendicitis

2. Signs for acute abdomen?


-> Murphy’s sign
-> Rovsing’s sign
-> Iliopsoas’s sign -> javorski`s sign
-> Cullen’s sign
-> Grey-Turner’s sign-> Blumberg’s sign

3. List 6 signs/symptoms of chronic liver failure.


-> clubbing of fingers
-> spider nevi
-> gynecomastia
-> hepatomegaly
-> testicular atrophy
-> palmar erythema
-> loss of axillary/pubic hair -> jaundice

4. Signs & symptoms of left ventricular failure.


-> nocturnal dyspnea -> fatigue,
-> orthopnea
-> cyanosis
-> tachycardia -> confusion

5. Signs & symptoms of right ventricular failure.


-> fatigue
-> weight gain
-> pitting edema
-> hepatomegaly
-> splenomegaly
-> Kussmaul sign (JVP)

6. Location of heart valves.


-> mitral: left 5th ICS in MCL
-> tricuspid: 4th ICS at left sternal border -> aortic: 2nd ICS at right sternal border
-> pulmonary: 2nd ICS at left sternal border

7. Witch of auscultation sounds of lung are physiological?


-> vesicular
-> bronchovesicular -> bronchial

8. Which of auscultation sounds of lungs are pathological?


-> crackles (pulmonary edema, pulmonary fibrosis) -> wheeze (asthma, COPD)
-> rhonchus (COPD, pneumonia, Cystic Fibrosis)

9. What is Horner syndrome & list symptoms and causes of it?


Horner syndrome is a combination of symptoms that arise when sympathetic trunk is
damaged.
-> unilateral ptosis
-> anhidrosis
-> miosis
-> pseudoenophthalmos

10. Symptoms of Superior Vena Cava Syndrome:


-> difficulty breathing.
-> headache.
-> facial swelling.
-> venous distention in the neck and distended veins in the upper chest and arms -> upper
limb edema
-> cough

11. In what valvular pathologies can detect diastolic murmur/systolic murmur?


Systolic murmurs:
-> aortic/ pulmonary stenosis
-> mitral valve prolapse
-> atrial septal defect
Diastolic murmurs:
-> aortic/pulmonic regurgitation -> mitral/ tricuspid stenosis

12. Findings upon chest examination in pneumonia.


Inspection:
-> sputum production -> cough
-> central cyanosis
-> fever
-> tachypnea Palpation:
-> Tactile fremitus ↑-> Chest expansion ↓Percussion:
-> dull
Auscultation:
-> bronchial breathing -> crackles, wheeze

2. Findings upon chest examination in pneumonia of lower lobe of right lung.


Inspection:
-> sputum production
-> cough
-> central cyanosis
-> fever
-> tachypnea
Palpation:
-> Tactile fremitus ↑
-> Chest expansion ↓
Percussion:
-> dull
Auscultation:
-> bronchial breathing
-> crackles, wheeze

13. Findings upon chest examination in COPD?


Inspection:
-> cough
-> dyspnoea
Palpation:
-> ↓ chest expansion bilaterally
Percussion:
-> hyper-resonant Auscultation:
-> wheeze, crackles
-> ↓ vesicular breaths

14. Findings upon chest examination in pneumothorax?


Inspection:
-> dyspnoea
-> chest pain
-> fatigue
Palpation:
-> ↓ tactile fremitus
-> ↓ unilateral chest expansion Percussion:
-> Hyper-resonant Auscultation:
->↓ vesicular breaths

15. Findings upon chest examination in bronchitis?


Inspection: -> cough
-> chest pain -> fatigue
-> fever
-> engagement of accessory respiratory muscles -> sputum
Palpation:
->
->
Percussion:
-> resonant Auscultation:
-> wheezing
-> bronchial sounds

16. List inferior borders of lungs.


Midclavicular line: RL 6th rib, LL 4th rib
Midaxillary line: RL 8th rib, LL 8th rib
Scapular line: RL and LL 10th rib
Paravertebral line: RL and LL 10th rib
17. What is CRP?
C-reactive protein, marker of inflammation. >3mg/L high risk

18. What are the causes of hepatomegaly?


-> cirrhosis
-> chronic liver failure -> viral hepatitis
-> alcohol abuse
-> Reye’s syndrome-> sarcoidosis

19. List symptoms of hyperthyroidism.


-> excessive sweating
-> heat intolerance
-> increased bowel movements. -> tremor
-> nervousness, anxiety
-> rapid HR
-> weight loss.
-> fatigue
-> diarrhoea

20. List symptoms of hypothyroidism.


-> fatigue
-> intolerance to cold -> constipation
-> dry skin
-> weight gain
-> puffy face.
-> hoarseness
-> muscle weakness -> hair loss

21. What are the causes of pneumothorax?


-> Cystic Fibrosis -> asthma
-> lung cancer -> trauma
-> smoking -> COPD

22. List 5 symptoms of emphysema.


-> persistent coughing
-> wheezing
-> excessive mucus production -> chest pain
-> general fatigue
-> loss of appetite

23. List signs of Grave’s disease.


-> exophthalmos
-> anxiety and irritability
-> a fine tremor of fingers
-> heat sensitivity
-> weight loss
-> enlargement of thyroid gland -> change in menstrual cycles -> muscle weakness

9. Signs of Grave’s disease (eyes).


-> Stellwag's sign : absence of normal blinking
-> Von Graefe`s sign : upper eye lid lags behind the eye ball as the patient is asked to look
downwards.
-> Moebius sign : inability converge the eye balls
-> Kocher’s sign

24. Causes of acute pancreatitis.


-> alcohol abuse -> trauma
-> gallstones
-> smoking
-> viral infections (fx mumps) -> drugs

25. Causes of chronic pancreatitis.


-> alcohol abuse
-> Cystic Fibrosis
-> autoimmune conditions
-> duct obstruction (fx by cancer)

26. Symptoms of pancreatitis.


-> upper abdominal pain
-> abdominal pain that feels worse after eating. -> fever
-> rapid pulse
-> nausea
-> vomiting
-> tenderness when touching the abdomen

27. Symptoms of dyspnoea:


-> nasal flaring
-> noisy breathing
-> very fast, shallow breaths
-> cyanosis
-> wheezing
-> chest pain.
-> accessory respiratory muscles involved
28. Abdominal signs + related diseases
-> PANCREATITIS:
Grey Turner's sign (bruising of the flanks)
Cullen's sign (superficial edema and bruising around umbilicus)
-> PANCREATIC CANCER
Courvoisier sign (enlarged palpable gallbladder)
-> APPENDICITIS:
McBurney's sign (deep tenderness at McBurney's point)
Iliopsoas’s sign (hyperextension of right hip causing abdominal pain)
Obturator’s sign (internal rotation of flexed right hip causing abdominal)
Rovsing’s sign (right lower quadrant pain with palpation of the left lower quadrant)
-> CHOLECYSTITIS
Murphy’s sign (abrupt interruption of inspiration on palpation of right upper quadrant)
Peritonitis
Blumberg's sign
Chelmonski sign
-> KIDNEYS DISEASE
Goldflame sign

29. Diabetic criteria:


-> random glucose level >200 mg/dL -> fasting glucose level >126 mg/dL -> 2hour glucose
level >200mg/dL -> haemoglobin A >6,5%
30. Examination of lymph nodes?
-> location
-> size
-> colour of skin -> tenderness -> fixation
-> consistency

8. What are the types of cyanosis + where can we find them + level of oxygen?
-> central cyanosis : at mucous, lips, conjunctiva
-> peripheral cyanosis : at peripheral skin, fingers
Deoxyhemoglobin >5mg/dL

31. Causes of central cyanosis?


-> hypoventilation -> bronchospasm
-> congenital heart disease -> sepsis
-> asthma

32. Causes of peripheral cyanosis?


-> cold exposure
-> low cardiac output
-> right-sided heart disease

33. Signs symptoms of hyperthyroidism (eye symptoms).


-> Stellwag's sign : absence of normal blinking
-> Von Graefe`s sign : upper eye lid lags behind the eye ball as the patient is asked to look
downwards.
-> Moebius sign : inability converge the eye balls Gifford's sign: Difficulty in eversion of the
upper lid
-> Kocher’s sign

34. Draw a graph of: Kussmaul breathing + Cheyne-stokes breathing.

35. Draw a graph of Biot’s respiration.

36. Draw a graph: intermittent fever + recurrent fever


37. Where can we measure pulse?
-> carotid artery
-> radial artery
-> femoral artery
-> popliteal artery
-> posterior tibial artery -> dorsalis pedis artery

38. List symptoms of aortic regurgitation.


-> dyspnea on exertion -> orthopnea
-> nocturnal dyspnea -> palpitations
-> angina pectoris
-> cyanosis (in acute cases) -> fatigue

39. How is the pulse in aortic stenosis?


7. List 4 causes of ascites.
-> cirrhosis
-> tuberculosis
-> heart failure
-> cancer

11. What can be seen during examination of patient with cardiac tamponade?
-> dyspnea
-> cough
-> chest pain
-> decreased heart sounds

2017

2011
Seminar group 1 questions

1. Thick skin on a physical examination could indicate:


a. Scleroderma
b. Myxedema
c. Ichtyosis
d. All of the above
2. Which of the following conditions do not produce a symptom of “wet” cough`?
a. COPD
b. Tuberculosis
c. Atelectasis
d. Tobacco abuse

3. The main cause of Community-acquired pneumonia is/are:


a. E. coli
b. Streptococcus pneumonia
c. Gram-positive bacteria
d. Answer b & c are correct

4. Pulmonary embolism usually arise from thrombi originating from:


a. Lower extremity deep veins
b. Pelvic veins
c. Upper extremity veins
d. Renal veins

5. Which type of emphysema is/are not associated with airflow obstruction?


a. Centriacinar emphysema
b. Panacinar emphysema
c. Paraseptal emphysema
d. b and c

6. Symptoms of COPD are a combination of:


a. Emphysema
b. Pneumonia
c. Chronic bronchitis
d. a and c
e. a and b

7. According to NYHA classification, in which class would a patient with marked limitation of
physical activity be in?
a. NYHA class I
b. NYHA class II
c. NYHA class III
d. NYHA class IV

8. Which symptom is less likely to appear in right-side heart failure?


a. Peripheral edema
b. Hepatomegaly
c. Anorexia
d. Orthopnea

9. Hepatojugular reflux is a sign of:


a. Increased venous pressure when applied pressure on the abdomen.
b. Decreased venous pressure when applied pressure on the abdomen.
c. Decreased atrial pressure when applied pressure on the abdomen.
d. Increased atrial pressure when applied pressure on the abdomen.

10. Common sign(s) of RS and LS heart failure is/are:


a. Nocturia
b. S3 gallop
c. hydrothorax
d. All of the above

Seminar group 2 questions

1)Indicate the FALSE answer. Decrease skin moisture:


a) Elders
b) Scleroderma
c) Hyperthyroidism
d) Vomiting
e) Diarrhea

ANSWER: c) Hyperthyroidism

2)Where should you look for cyanosis?


a) lips
b) nail beds
c) Eyes
d) A and B is correct
e) A, B and C is correct

ANSWER: d) A and B is correct

3)In pneumonia sputum could be:


a) bloody or rust coloured
b) green
c) foul-smelling and bad-tasting
d) currant-jelly
e) all are correct

ANSWER: e) all are correct

4)A woman presents to the hospital with excess fluid in her pericardium (pericardial effusion) which
of the following options is not a possible mechanism of her condition:
a) Inflammatory connective tissue disease
b) Cancer
c) Left ventricular hypertrophy
d) Viral infection
e) None of the above

ANSWER: c) left ventricular hypertrophy

5)Aortic Stenosis
a) creates a systolic murmur
b) creates a diastolic murmur
c) best heard at the in 2nd right intercostal space
d) A&C are correct
e) all are correct

ANSWER: d) A&C are correct

6)Mitral Stenosis
a) creates a systolic murmur
b) creates a diastolic murmur
c) best heard at the in 5th left intercostal space (apex)
d) A&C are correct
e) B&C are correct

ANSWER: e) B&C are correct

7)Which is NOT a sign of left congestive heart failure


a) pulmonary hypertension
b) left ventricle hypertrophy
c) dyspnea
d) pulmonary edema
e) peripheral edema

ANSWER: e) Peripheral edema

8)Which of the following is the most common cause of blood in the sputum:
a) Tuberculosis
b) Sublingual varicosities
c) Congestive heart failure
d) Pneumonia
e) Purulent discharge

ANSWER: b) sublingual varicosities

9)Name the option preferred for palpation of the apex beat:


a) Left lateral recumbent position
b) Sitting position
c) Supine position
d) Answers A & B
e) None of the above

ANSWER: d) A&B

10) What is hemoptysis


a) vomiting blood
b) coughing up blood
c) bloody stool - coagulated
d) bloody stool – fresh
e) none of the above

ANSWER: b) coughing up blood

Seminar group 3 questions

1)Pulsus diferensis is seen in:


a) aortic stenosis
b) Pericarditis
c) aoric regurgitation-correct
d) Tricuspid regurgitation

2)Sharp pain in precordial area, increases during respiratory movement or left side position is an
indication for:
a) angina pectoris
b) prinzmetals angina
c) myocardial infarct
d) percarditis-correct

3)Paradoxal pulse is seen


a) cardiac temponade
b) pulmonary embolism
c) asthma
d) all of the above- correct

4)which of the below can not be a cause of transudate


a) hypoalbuminema
b) congestive heart failure
c) pancreatitis-correct
d) chirrosis
5)In case of unbalanced respiratory acidosis arterial blood analysis show:
a) increased PH, decreased CO2, increased HCO3
b) decreased PH, increasedCO2, increased HCO3-correct
c9 decreased PH, decreased CO2, inc HCO3
d) decreased PH, INcreased CO2, decreased HCO3

6)Mendelson syndrome:
a) Is a oppertunistic pneumonia
b) is a form of atelectasis
c) is aform of chemical pneumonia- correct
d) is a form of atypical pneumonia

7)Combine correct alternatives:


1. Raspberry sputuma) tumor
2. brown sputumb) hemophilus influenzae
3. green sputumc) pneumococcal infection

correct: 1-a, 2-c, 3-b

Seminar group 4 questions

1. Kind of breathing that can occur during acidosis


a. Biot
b. Kussmalcorrect
c. Cheyne stokes
d. No change in breathing

2. Small cell lung cancer is characteristic for


a. smokerscorrect
b. adenocarcinoma
c. Squamous cell carcinoma
d. In small airways

3. kaiser fischer ring is characteristic for


a. Addisons
b. Wilsonscorrect
c. Horners
d. corneal arcus

4. Pancoast tumor can cause the following


a. Superior vena cava syndrome
b. Atelectasis
c. Hornerssyndromecorrect
d. Enlarged virchow's node

5. Unilateral exopthamlus is found in following


a. Ortibal tumor
b. Aneurysm
c.Orbital hematoma
d. All of the abovecorrect

6. Which malignancy is the most common in the virchow's node


a. Lung
b. GI Tractcorrect
c. Lung and GI tract
d. Glioblastoma

7. Which one is not a sign of Horners syndrome


a. Endopthalmus
b. Anhydrosis
c. Miosis
d. Exopthalmuscorrect

8. Not a sign of dyspne is


a. Nasal flarring
b. Retraction of ribs
c. Use of accessory muscle
d. Decubitus positioncorrect

9. In oliver Cardarelle sign the is no diagnos for


a. Mediastinal tumor
b. Aortic reguritation
c. Aortic aneurysm
d. Heart Failurecorrect

10. Lung cancer wrong statement


a. Tends to spread very early
b. 90 - 95 % arise from epithelial lining of small airways
c. Stronglu related to smokers
d. Small cell lung cancer is the most common type-correct

INTERNAL 2020 - REVIEW

Spørsmål Svar + forklaring

1. Hepatic failure is usually observed: Increased INR


Prolonged PT is one of the first signs of liver failure
(acc. to rocket).
Remember that the liver has a huge residual
capacity, so it takes a great deal of fibrosis/Cirrhosis
for it to fail.
Increased INR is due to failure in producing clotting
factors.

2. Hyperpigmentation in the palmar creases is Primary adrenal insufficiency (Addison’s disease)


characteristic for: Positive feedback increases ACTH levels → binds to
melanocortin receptors on melanocytes in the skin

3. Brown skin color can be found in: c? hyperpigmentation


Select one: Må være det ja
A. Addison-Biermer disease
Enig her, Cushing har de der abdominal striae på
B. Cushing's syndrome
siden often, men de er jo “local skin changes” hvsi
C. Addison's disease man kan si det/ Lou
D. Horner syndrome
E. Paget's disease

4. Cockeroft and Gault can be used to: A


Select one:
A. Calculate creatinine clearance
B. Calculate metoxycatecholamins in urine
C. Calculate pulse oximetry
D. Calculate FEV1/FVC index
E. None of the above

5. Which abnormalities are present in 2, 3


non-compensated respiratory acidosis :
1. increased pH
2. decreased pH
3. increased pCO2
4. decreased pCO2
5. increased HCO3-
6. decreased HCO3-
7. increased base deficiency
8. decreased base deficiency
Correct statements are:

6. Atrial fibrillation can be caused by: E


Select one:
A. Hyperthyroidism Common causes of atrial fibrillation:
- Hypertension
B. Coronary artery disease
- Heart failure
C. Electrolyte imbalance - Myocardial infarct
D. Heart valve dysfunction - Thyrotoxicosis
E. All of the above are correct - Alcohol-related heart disease
- Mitral valve disease
- Infection, e.g. respiratory, urinary
- Following surgery, especially cardiothoracic
surgery
- Electrolyte imbalance
7. The cause of pericardial effusion can be: 2, 3, 4, 5
1. uremia
2. tuberculous pericarditis
3. myocardial infarction
4. rheumatic disease
5. systemic lupus erythematosus
Correct statements are:

8. In which type of anemia are neurological B


symptoms most common? Pernicious anemia
Select one:
A. Folic acid deficiency
B. Vitamin B12 deficiency
C. Iron deficiency
D. A and c
E. B and c

9. Aortic valve is auscultated in the: B


Select one:
A. Left 5th intercostal space. Auscultation of heart valves:
2nd right: aortic
B. Right 2nd intercostal space.
2nd left: pulmonary
C. Left 4th intercostal space. 4th left: tricuspid valve
D. Left 2nd intercostal space. 5th left: mitral valve
E. Right 5th intercostal space
3rd left: ERB point

10. Hepatic encephalopathy is associated with: D


Select one:
A. An increased concentration of urea in the Defective hepatic urea cycle function causes
accumulation in ammonia, seen in acute liver failure
blood
B. Brain hypoperfusion Vil det ikke da være D?? - Anni
C. Increased level of albumins in the blood Haha joooo!
D. Increased concentration of ammonia in the
blood
E. Hypernatremia

11. Assign first-step diagnostic procedures used to A → 4?


diagnose the diseases listed below. B → 2?
(A) ileus C→1
D→5
(B) cholecystitis
E → 3?
(C) gastric ulcer
(D) rectal tumor
(E) hiatal hernia
1. gastroscopy
2. abdominal ultrasound
3. upper gastrointestinal tract X-ray with oral
contrast medium
4. abdominal X-ray
5. rectoscopy
12. Increased waist circumflex might be caused by E!
Select one: A → Ascites caused by hepatic congestion and
A. Heart failure pulmonary htn
B → Ascites caused by fibrosis causing sinusoidal htn
B. Liver cirrhosis
and hypoalbuminemia (by decreased liver function)
C. Ovarian cancer causing more fluid to move out to extravascular
D. Correct a and b space
E. Correct a, b and c
Hvis ovarian tumor er stor nok kanskje det kunne økt
waist circumference? eller - anni
- Jeg er enig med at Ovarian tumor også kan øke
waist cirumference, de er ofte sneaky, agressive og
kan blive ret store uden at de giver så mange andre
symptomer / Lou
REGEL = 6F

13. In a patient with chronic pulmonary heart 1, 2, 3, 4, 5, 6


disease caused by severe COPD (chronic
obstructive pulmonary disease) may occur: har en ikke increased hemoglobin her da, pga COPD?
1. Dyspnoea
2. Peripheral edemas
Det er basically cor pulmonale de spør om sant?
3. Fluid in the pleural cavities Vet ikke med de to siste
4. Enlarged liver
5. Enlarged spleen
6. Fluid in the peritoneal cavity
7. Atrial fibrillation lærte i morph at både RHF og LHF kan gjøre så det
8. Poliglobulia (=Polycythemia) blir pleural effusion da,
men det står jo bare may occur, ikke noe classical
The correct answers are:
symptoms, eller
altså jeg aner ikke, tenkte bare ut ifra at cor
pulmonale er RHF og den kan gi pleural effusion

14. Cushing syndrome is group of symptoms Hypoglycemia


caused by excessive levels of glucocorticoids.
Among symptoms below point the one that Signs in cushing syndrome:
- Central obesity, moon face, buffalo hump
DOES NOT suit to Cushing syndrome: - Mood change (depression, lethargy,
irritability, psychosis)
- Irregular menses, hirsutism, erectile
dysfunction
- Acne, abdominal striae
- Increased BP, increased glucose levels
- Recurrent achilles tendon rupture
- Hyperglycemia
- Hypokalemia
- Hypernatremia

15. It is the easiest to differentiate between B. Percussion


hydrothorax and pneumothorax upon:
A. Auscultation Vocal fremitus increases in hydrothorax and
B. Percussion decreases in pneumothorax, men vet ikke om det er
letteste måten å skille de?
C. Vocal fremitus
D. Chest movement Tror percussion er beste måten, det er vel
E. Is not possible hyper-resonant på pneumothorax og dull på
hydrothorax? - anni
Enig, Lou

16. Hypertension can be caused by: B?


1. Cushing's syndrome
2. Hypothyroidism 4 → primarily affect cortisol but can also impair
3. Renal diseases aldosterone synthesis, low aldosterone would rather
cause hypotension.
4. Adrenal insufficiency
Enig her, Lou
5. Hyperthyroidism
Correct are:
Select one:
A. 1, 2, 3, 4, 5
B. 1, 2, 3, 5
C. 1, 3, 5
D. 3, 5
E. 1, 2, 4

17. Secondary hypertension can occur in all B?


diseases except :
Select one: Enig her, Liver diseases → Portal hypertension og
A. Kidney diseases evt. low albumin levels. Og low albumin vil jo kunne
gi hypotension og edema. / Lou
B. Liver diseases
C. Suprarenal gland diseases
D. Pituitary gland diseases
E. Aortic diseases

18. In which pathological states is fever ever is one


of the major symptoms: Hmm, jeg kunne have sagt alle her.
I.Tumour of a kidney Tumor of kidney: yes (morph)
II.Pneumonia Pneuomnia: yes
III.Lymphoma Lymphoma: yes, fever and night sweats
Infectious endocarditis: tror jeg bestemt, men kan
IV.Infectious endocarditis
slå op
V.Lupus erythematosus Lupus erythematosus:
Select one:
A. All above are correct
B. Correct II, III, IV
jeg tror også alle, SLE er jo the big imposter så den
C. Correct II, IV, V
har jo basically every single symptom in the book
D. Only II i s correct
E. Correct I, II

19. Polyuria occurs in: 1, 2, 5?


1. Decompensated diabetes tror 3 også: “In hyperparathyroidism, the
2. In antidiuretic deficiency syndrome commonest symptoms relate to hypercalcaemia:
3. In primary hyperparathyroidism in polyuria, polydipsia, renal stones, …”
hypercalcemia
4. Congenital syndromes of calcium, other “Polyuria also occurs when the kidneys cannot
electrolytes and amino acids reabsorption concentrate urine. Causes may be extrarenal, e.g.
disorder in kidney coils diuretic drugs; hyperglycaemia with glycosuria
5. Renal glycosuria causing an osmotic diuresis; lack of arginine
vasopressin (AVP) from the pituitary gland in cranial
diabetes insipidus, or failure of aldosterone
secretion by the adrenal gland in Addison’s disease.”

20. Which diseases can cause the superior vena 1, 2, 5?


cava syndrome:
1) Lung cancer Lung cancer: yes, esp. tumor located in the apex
2) Lymphoma Lymphoma: yes, esp. the central/mediastinal ones
Aortic aneurysm: if located in the ascending +
3) Acute pancreatitis
thoracic part (?)
4) Gastritis
5) Aortic aneurysm

21. Normal range of arterial carbon dioxide partial C


pressure is:
Select one:
A. below 25 mmHg
B. 25-35 mmHg
C. 35-45 mmHg
D. 45-55 mmHg
E. 55-65 mmHg

22. 1st degree atrioventricular block (latent): B


Select one:
A. it is characterized by the stimulation of the
ventricles from a replacement pacemaker
B. does not cause subjective clinical symptoms
C. cannot be recognized by ECG
D. Only the answers A and B are true
E. Only the answers B and C are true

23. Upon physical examination you diagnosed D → Pancoast tumor


patient having symptoms of the Horner's
syndrome. Which of the additional examination
you will perform in this patient as a first choice
procedure?
Select one:
A. Gastroscopy
B. Head CT scan
C. Visual field examination
D. Thorax CT scan
E. Gastroscopy

24. In arterial blood gases pH 7.21, pCO2 35mmHg , C. Non-compensated metabolic acidosis
HCO3 15mmol/l. Correct diagnosis is:
ph=low
Select one:
CO2=normal
A. Compensated metabolic acidosis
HCO3=low
B. Compensated respiratory acidosis
C. Non-compensated metabolic acidosis
D. Non-compensated respiratory acidosis
E. No correct result.

25. Select the true sentence: E. The correct answers are a and b

A. Nausea and vomiting may accompany


inferior wall myocardial infarction
B. Anginal pain is relieved by nitroglycerin
C. Exacerbation of angina is dependent upon
body position
D. The correct answers are a and c
E. The correct answers are a and b

26. Murmur over thyroid gland is associated with: C. Increased blood flow to the gland
Select one:
A. Calcification of thyroid gland
B. Hashimoto disease
C. Increased blood flow in the gland.
D. Anemia
E. Single thyroid gland nodule

27. Choose the characteristics of angina: 1,2, 4, 6, 7?


1. Pain is burning, choking, crushing
2. It is usually located retrosternally Er enig her, men usikker på 1, fordi pain er så
3. Pain is triggered by a change in body subjektivt.
I Mcleod, angina pain: “Constricting, heavy”
position or pressure
Jeg tror godt at 1 kan være riktig, ettersom det skal
4. It can radiate to the neck, lower jaw, like an elephant sitting on my chest, det vil jeg tro er
epigastric, left arm både choking og crushing, at de føler at de ikke kan
5. Acute, stinging pain associated with puste fordi noe sitter på brystet deres - anni
breathing
6. Relieves after taking nitroglycerin s.l. 3 fits with pericardial pain which is may be aggravte
usually within 1-3 min. by postrual shift.
7. It i s caused by physical exertion or
emotional stress and soothes at rest

28. While auscultating a patient, a systolic murmur C. Mitral regurgitation


with puncture maximum in the area of apex was
found. It radiates to the left armpit. What kind of Tricuspid and mitral valves should be closed during
diagnosis can we suspect? systole, if there is a systolic murmur heard most
loudly at the apex of the heart it would indicate that
Select one: the issue is located in the mitral valve.
A. Aortic regurgitation
B. Aortic stenosis
C. Mitral regurgitation
D. Mitral stenosis
E. Ventricular septal defect

29. In the urinalysis, the presence of the following E


may indicate infection, with the exception of:
Select one:
A. Protein
B. Leukocytes
C. Ketone bodies
D. Nitrates
E. All of above may indicate infection

30. Causes of secondary hypertension include: - Renovascular disease (stenosis)


- Pheochromocytoma
- Primary aldosteronism
- Cushing syndrome
- Coarctation of the aorta
- Adult polycystic kidney disease
List fra MC s. 50/ Lou

Secondary hypertension fra oxford:


- Renal disease: glomerulonephritis,
polyarteritis nodosa, systemic sclerosis,
chronic pyelonephritis, PCKD, renovascular
disease
- Endocrine disease: Cushing, Conn
syndrome, pheochromocytoma,
acromegaly, hyperparathyroidism
- Others: coarctation of aorta, pregnancy,
liquorice :), several drugs (steroids, MAOI,
contraceptives, cocaine, amphetamine etc.)

31. Paralytic ileus may be a consequence of: - Abdominal trauma + surgery


- medication-based (opiods!)
- electrolyte imbalances:
hypokalemia, hyponatremia, hypomagnesia
- pancreatitis
- diffuse peritonitis
- mesenteric ischemia (↓blood flow)
Her har jeg været på google :-)
Oxford sier:
- abdominal surgery
- pancreatitis (or any localized peritonitis)
- spinal injury
- hypokalemia, hyponatremia, uremia
- peritoneal sepsis
- drugs (TCAs)

32. Asthma and COPD are differentiated by: eosinophils morph hehe

33. Haemoptysis can be caused by: - Respiratory tract infections:


Pneumonia, Bronchitis (acute+chron),TB
- Lung tumors
- Pulmonary embolism
- Bronchiectasis
- Pulmonary oedema (pink frothy sputum,
det ligner sådan næsten bobler fra
skumbad)
Liste taget fra Macleods/ Lou

34. In the peripheral blood morphology laboratory Leukocytosis: ↑WBC count


test you found neutrophil leukocytosis. In which
clinical situation does it occur most typically?
1.HIV infection
2.Treatment with cytostatic agents
3.Trauma
4.within treatment with glucocorticosteroids
Neutrophils are increased in:
5.in aplastic anemia - Bacterial infections
6.in acute appendicitis - inflammation (MI, polyarteritis nodosa)
7.in bacterial pneumonia - Myeloproliferative disorders
8.in Addison's disease - Drugs (steroids)
Select one: - Disseminated malignancy
A. Correct answers 1,3,4,5,7 - Stress (e.g. trauma, surgery, burns,
hemorrhage, seizure)
B. Correct answers 2,3,4,7
C. Correct answers 3,4,6,7 SÅ C. 3, 4, 6, 7? eller hva tror du
D. Correct answers 3,6,7,8 Enig med C - Lou
E. All answers are correct

35. Patient with a massive ascites might present: E. All of the above
Select one:
A. Increased abdomen circumflex
B. Dull sound upon percussion of the abdomen
C. Sign of fluid wave
D. Decreased peristalsis during auscultation
E. All of the above

36. Which of the following is an acute complication B Iatrogenic hypoglycemia


of diabetes? → Too high dose of insulin administered
Select one:
A. Diabetic retinopathy All of the others are complications seen in DM but
none of them are acute
B. Iatrogenic hypoglycemia
C. Diabetic foot syndrome
D. Diabetic nephropathy
E. Peripheral polyneuropathy

37. Total respiratory failure can be diagnosed upon B. PaO2 lower than 60 mmHg, PaCO2 higher than 45
blood-gases results of arterial blood in the case of: mmHg.
Ville ha’ gået for B her - Lou
Select one:
A. PaO2 lower than 60 mmHg, PaCO2 lower than 45
mmHg Respiratory failure:
B. PaO2 lower than 60 mmHg, PaCO2 higher than 45 Type 1 -partial
mmHg LOW pO2 < 50 mmhg, no CO2 retention
C. PaO2 higher than 60 mmHg, PaCO2 lower than 45
Type 2- complete respiratory failure
mmHg
LOW PO2 < 50mmhg + HIGH pCO2 > 60mmhg
D. PaO2 higher than 60 mmHg, PaCO2 higher than
45 mmHg
E. None of the above

38. Paralytic ileus may be a consequence of: E. answers A, B and C are correct
(look for our amazing list above :-) )
Select one:
A. peritonitis
B. pancreatitis
C. hypokalemia
D. A and B are correct
E. answers A, B and C are correct

39. The absence of peristalsis is typical in: E?


Select one: Er lidt enig på denne - Lou
A. Paralytic ileus.
B. Chronic constipation. Jeg tenker kanskje bare A? de andre er vel bare
decreased og ikke absent? i chronic constipation så
C. Hypothyroidism.
vil du vel ha bowel sounds foran der stool har
D. A and B are correct. accumulated i tarmen eller? jeg er ikke helt sikker,
E. All of the above. men føler bare at alltid når vi har snakket om absent
peristalsis så er det i paralytic ileus

Jeg ville også bare tatt A. tuva

40. Oral glucose tolerance test consists of


following stages:

41. Dysphagia may occur during which state: Esophageal cancer (Plummmer-Vinson syndrome)
Stomach cancer
Sjögren
Zenker diverticulum
- Tatt fra powerpoint - tuv

42. Systolic murmur heard during auscultation of E?


left carotid artery might be caused by: Radiation to the carotids er vel most commonly
caused by aortic stenosis?
Select one: - Enig, Lou
A. Aneurysmal dilatation
B. Mitral valve stenosis kan det ikke være c? at det er narrowing av aorta
C. Narrowing caused by atherosclerosis feks? eller kanskje ikke idk
D. Pulmonary valve stenosis
hadde også tatt E - tuv
E. None of the above

43. Physical examination performed on a patient A?


revealed differing blood pressure measurements on
the left and right upper limbs: 170/105 mmhg, Tenker D og kan være riktig så isfåfall E, men B vil vel
120/80mmHg respectively. The potential cause heller føre til pulse deficit?
- Er enig her (Lou), ja det tror jeg du har ret i.
could be:
kan det være at dissection skjer etter
Select one: brachiocephalic trunk og før left subclavian? derfor
A. Atherosclerosis of right subclavian artery det kan være differing BP på upper limbs? og isåfall
B. Aortic dissection kan den stemme eller - anni
C. Cervical rib syndrome
D. Technical/measurement error så E?
E. All above correct

44. A 55-year old woman presents to the Kunne det være A her?
Emergency Department with jaundice that she Hun passer med Female og over 40, sikkert også fat
noticed 2 days ago. Her medical history ;-). Også past history.
includes an episode of cholelithiasis treated
Jeg læste det godt, men hun kan godt have en ny
with a cholecystectomy 4 years ago. The most
sten intra hepatic ducts? de er vel ikke borte?
likely diagnosis is:
hahaha, men får man jaundice av bile stones? kan
A. Recurrence of cholelithiasis. det være obstruction
B. Bile duct obstruction.
C. Acute hepatitis B infection. står at gallstones are formed in the gallbladder på
D. Paracetamol poisoning. google, så vet ikke om hun kan få det da
E. Pancreatic head cancer.
jeg tror at om det hadde vært i hepatic duct så
hadde de skrevet hepatolithiasis eller? siden
cholelithiasis i definisjon betyr stones in gallbladder,
og gallstones er produced in gallbladder

jeg tror D - siri, men hepatitis kan jo også gi jaundice


45. Among the typical signs of cholangitis are: 1, 2, 3, 4
1. Fever
2. Right upper quadrant pain 4? usikkker, tror ikke det
3. Jaundice
4. Hepatomegaly
5. Constipation
Correct answers are:

46. Point basic symptoms of lobular pneumonia B?


in chest examination:
Select one:
A. sharp vesicular murmur, multiple dry
rales, wheeze.
B. fine rales, pleural friction rub, dull
percussion sound, sharp vesicular murmur
C. bronchial murmur, crepitations, dull
percussion sound
D. wheeze, dry rales, stridor
E. pleural friction rub, weak vocal fremitus,
vesicular murmur

47. Frequent or permanent urination is called: E


Select one:
A. Oliguria No no no tror det er C!! - tuva, enig- siri :)
B. Stranguria
Polyuria is a condition where the body urinates
C. Pollakisuria
more than usual and passes excessive or abnormally
D. Anuria large amounts of urine each time you urinate
E. Polyuria → dette betyr jo basically bare at du tisser STØRRE
VOLUM MED URIN hver gang du tisser, ikke
nødvendigvis at du tisser med increased frequency!

Pollakiuria is also known as benign idiopathic


urinary frequency. It refers to frequent daytime
urination in children with no specific cause.

48. The reasons for thyroid enlargement (goiter) E


include:

Select one:
A. hypothyroidism
B. thyroiditis
C. thyroid neoplasms
D. iodine deficiency
E. all of the above

49. Cyanosis is the bluish discolouration of the Hypoxemia, low oxygen saturation of the Hb
skin and mucous membranes caused by:
-It is caused by accumlation of dexoygenated
hemoglobin in a level of >5mg/dl

50. The reason for haematuria is: 1, 2, 3, 5?


1. kidney stone
2. bladder cancer Vil si’ 5 også, det sagde hun lab.dig damen på
3. glomerulonephritis seminar at physical acc. can cause microscopic
hematuria.
4. beetroots
5. physical activity

51. The cause of microcytic anemia may be a C. iron


deficiency of:
Select one:
A. vitamin B12
B. folic acid
C. iron
D. coagulation factors
E. all of the above

52. (I) A chest x-ray is a basic imaging test that can A. Parts I and II are true and they are logically
confirm an exacerbation („attack") of asthma, connected.
because (II) it visualizes atelectasis of inferior lobes
of the lungs.
Select one:
A. Parts I and II are true and they are logically
connected.
B. Parts I and II are true, but they are not logically
connected.
C. Part I is true, part II is false.
D. Part I is false, part II is true.
E. Parts I and II are both false.

53. Normocytic anemia: D


Select one:
A. May be the result of acute bleeding
B. May occur in the course of cancer
C. Can occur in renal failure
D. Correct a, b and c
E. Correct b and c

54. (I) Thrombocytopenia is present in liver true true cause effect? - siri
cirrhosis (II) because due to portal
hypertension and hypersplenism platelets are jeg tror ikke cause & effect, platelets blir vel ikke
being destroyed. ødelagt pga portal hypertension (??), men T/T
hvertfall - tuven

55. Agranulocytosis is: Implies that granulocytes (WBCs with neutrophil,


basophil, or eosinophil granules) have stopped being
made, leaving the patient at risk of fatal infections.

56. A typical clinical picture of acute pancreatitis - Cullens sign


includes: - vomiting
- pain radiating to the back osv..

57. Gastric or duodenal ulceration might be E?


caused by: ja
Select one:
A. Chronic treatment with nonsteroidal
anti-inflammatory drugs
B. Chronic treatment with nonsteroidal
anti-inflammatory drugs in combination with
corticosteroids
C. Helicobacter pylori infection
D. A and c are correct
E. A, b and c are correct

58. During physical examination of a patient with A.


Pneumothorax all symptoms might be present except:
Vil den ikke være decreased i pneumothorax - jo tror
Select one: det.
A. Increased vocal fremitus
B. Hyper resonant sound upon percussion
C. Deviation of trachea
D. Decreased breathing sounds
E. Signs of dyspnea.

59. Causes of secondary hypertension include: E


A. polycystic kidney disease
B. renal artery stenosis
C. Cushing's syndrome
D. Correct B and C
E. All answers are correct

60. Physical examination of a patient with Emphysema - pink puffer !


pulmonary emphysema might reveal: Patient history:
- Smoking, dyspnea upon exertion

Physical examination:
- Barrel chest (hyperinflation)
- Dyspnea, use of accessory respiratory muscles
(scalenes, pectoralis major, abdominal muscles)
- Clubbing of fingers, pursed lips (fish mouth)
- Bilateral reduction in chest wall movement
- Prolonged expiration, stridor, wheezing
- Hyper-resonant sound upon percussion,
diminished vesicular sounds upon auscultation

61. (I) Abdominal ultrasound is the radiological F, T?


method of choice in suspected mechanical
bowel obstruction because (II) ultrasound
shows free intraabdominal fluid well.
62. What are the symptoms of hypothyroidism? Symptoms:
- Periorbital edema
- Husky voice
- Goitre
- Bradycardia
- Carpal tunnel syndrome
- Menorrhagia
- Constipation
- Low metabolic rate, weight gain
- Dry skin and hairloss
- Sensitivity to cold
- Lethargy, mental impairment, depression

63.Type 1 diabetes is caused by: C?


Select one:
A. Chronic pancreatitis Destruction of b-cells
B. Insulin resistance
enig - tuv
C. Autoimmune reaction
D. Destruction of α cells i n pancreas
E. None of the above

64. While auscultating the patient's heart you C


notice a completely irregular heartbeat. Select the
arrhythmias or disturbances of heart's electrical
conduction that can lead to such irregular
heartbeat:
Select one:
A. Atrial fibrillation
B. Atrial flutter with alternating conduction block
C. A & B are correct
D. Extrasystoles
E. None of the above

65. ST segment elevation occurs in: D. 1,2,3


1. Variant angina (Prinzmetal angina)
2. Myocardial infarction
3. Pericarditis
4. Digoxin toxicity
5. Hypokalemia
Select one:
A. only answer 2nd is correct
B. only answer 1st and 2nd is correct
C. the correct answers are 1, 2 and 4
D. the correct answers are 1, 2 and 3
E. all of the above answers are correct

66. Jaundice can be a sign of: E?


Select one:
A. Pancreatic head tumor
B. Hemolytic anemia
C. Chronic right-sided heart failure
D. A & B are correct
E. A, B & C are correct

67. A characteristic phenomenon in the urinalysis A. ketone bodies


in patients with anorexia nervosa is the
presence in the urine of:

68. Hypersplenism is: D. symptoms of increased accumulation and


sequestration of blood cells in the spleen

70. (I) In iron deficiency anemia there is high level F/T


of reticulocytes in peripheral blood because (II) as
result of iron deficiency there is impaired
erythropoiesis in bone marrow.

71. A typical symptom of aortic stenosis is - angina pectoris (ischemic pain), fainting,
congestive heart failure, shortness of
breath, lightheadedness, dizziness

72. Symptoms of pulmonary embolism - cyanosis,


- dyspnea
-

73. Diminution of inferior lung borderparal A. Pleural effusion

74. Which of the following statements describe Alle er rett in my opinion - tuv
Kussmaul breathing:
er det most characteristic for meningitis? er ikke det
1. is characterized by deep and rapid breathing heller Biots? -siri
2. occurs in the ketoacidosis
jo er forsåvidt enig i det
3. is caused by the stimulation of the
respiratory center by an excess of hydrogen
ions in the blood
4. is also the most characteristic for meningitis

75. Systolic murmur is present during auscultation E. 1 and 4


of heart valves in the case of:
1. Aortic stenosis
2. Mitral stenosis
3. Aortic regurgitation
4. Mitral regurgitation
5. Tricuspid valve stenosis

Select one:
A. 1 and 2
B. 2 and 3
C. 3 and 4
D. 2 and 5
E. 1 and 4

76. Pulse deficit is characteristic for: C?

Select one:
A. Heart tamponade
B. Aortic coarctation
C. Atrial fibrillation
D. Subclavian steal syndrome
E. Buerger syndrome

77. Select the causes of pulmonary edema: C 1,2,3, 4, 5, 6,7 (?) og 8? alle?- siri
1. Paroxysmal atrial fibrillation with rapid ventricular
response
2. Intravenous fluid overhydration of a patient with
impaired left ventricular systolic function
3. Intravenous fluid overhydration of a patient with
preserved left ventricular systolic function but with
significantly impaired left ventricular diastolic
function
4. Cardiac tamponade
5. Dissection of the ascending aorta with cardiac
tamponade
6. Myocardial infarction
7. Significant mitral regurgitation secondary rupture
of chordae tendineae in the course of infective
endocarditis
8. Pulmonary embolism

78. 73-year-old men with shortness of breath for D.1,4 pneumonia vil gi cough, cancer vil gi
the past few weeks came to the clinic. He denied hemoptysis
cough and hemoptysis. His past medical history
include hypertension, coronary heart disease (ACS
in 2012) and nicotinism (40 pack years of smoking).
On physical examination, blood pressure of 142/86
mmHg, regular heart rate of 78 bpm, chest
percussion revealed dull sound at the base of the
right lung, and auscultation diminished vesicular
sound, clear heart sound with no murmurs.
Peripheral edema was noted. Based on these
clinical findings, the most probable diagnosis is:
1.Heart failure
2.Lung cancer
3.Pneumothorax
4.Chronic obstructive pulmonary disease
5.Pneumonia
Select one:
A. 1, 4, 5
B. 1,2
C. 1,2,4
D. 1,4
E. All of the above

79. Pulsation over lower precordium is B?


characteristic for: A da? ser litt sånn ut
Yes, enig i den!
Select one:
A. right ventricle enlargement
B. left ventricle enlargement
C. aneurysm of ascending aorta
D. aortic stenosis
E. mitral regurgitation

80. Patient complaints about having E??


exercise-induced dyspnea that increases for years. agree
He also claims to have a productive cough with
expectoration of sputum (especially i n the
morning). Patient used to be an active smoker.
Which tests should you perform to confirm the
most probable cause of the above symptoms?

Select one:
A. Chest X-ray
B. Bronchofiberoscopy
C. Coronarography
D. Plethysmography
E. Spirometry

81. Anginal chest pain, dyspnoea on exertion, D. Aortic stenosis


dizziness, fainting, blurred vision (dark spots),
systolic murmur in the second intercostal space
radiating to the carotid arteries suggest:
Select one:
A. critical stenosis of the common carotid artery
B. coronary artery disease
C. bacterial endocarditis
D. aortic stenosis
E. coarctation of the aorta

82. Thyrotoxicosis can be a symptom of: E. all of the above


Select one:
A. Graves' disease
B. Toxic thyroid adenoma
C. subacute thyroiditis (De Quervain's disease).
D. pituitary adenoma secreting TSH
E. all of the above answers are correct
83. Characteristic symptoms for cardiac Paradoxal pulse
tamponade is/are

84. Hematuria can be caused by: e?


A. Tuberculosis
B. Neoplastic process i n urinary bladder, ser sånn ut iflg dr.google
C. Thrombocytopenia
D. A and B are correct
E. All of the above are correct

85. Hepatomegaly occurs in:

86. Which of the following sentence/s is / are fully B. 1 and 2


correct:
1. Hypoglycemia could be caused by insulin
overdose, physical activity or alcohol consumption
2. Gastroscopy is crucial diagnostic procedure in
peptic ulcer perforation
3. Positive Chelmonski sign is typical for peptic
ulcer perforation.
Select one:
A. only 1
B. 1 i 2
C. 2 i 3
D. only 3
E. all are correct

87. Complications of asthma include: D. 2 and 5


1. Pulmonary edema
2. Acute respiratory failure
3. Amyloidosis
4. Pulmonary fibrosis
5. Asthmatic state
The correct answers are:
Select one:
A. 1 and 2
B. 2 and 3
C. 3 and 4
D. 2 and 5
E. 1 and 4

88. A dehydrated patient might present with 1, 2, 5


1) hypotension
2) altered mental status
3) increased skin turgor
4) bradykardia
5) thick saliva

89.

90.

You might also like